Anda di halaman 1dari 94

PENULISAN KATA

Penulisan kata mengatur penulisan (1) kata dasar, (2) kata turunan, (3) pemenggalan kata, (4)
kata depan, (5) partikel, (6) singkatan, (7) angka dan bilangan, (8) kata ganti, serta (9) kata sandang.
Aturan-aturan ini mengikuti EYD (terbaru edisi V, 2022).

A. Kata Dasar
Kata dasar ditulis secara mandiri. Dalam tes seleksi masuk PTN, soal-soal yang bersinggungan
dengan aturan penulisan kata dasar seperti penulisan kata baku/tidak baku dan penulisan kata
serapan.

Kata baku adalah kata yang sudah ditentukan dalam kaidah kebahasaan (KBBI dan EYD). Untuk
itu, penggunaan kata baku harus sesuai dengan kaidah tersebut.
Contoh:
Kata Tidak Baku Kata Baku
antri antre
cendikia cendekia
ekstrim ekstrem
frustasi frustrasi
hipotesa hipotesis
ijasah ijazah
komplek kompleks
notulen notula
fikir pikir
sekedar sekadar

Kata serapan adalah kata yang berasal dari unsur bahasa selain bahasa Indonesia. Kata serapan
dapat terbentuk melalui proses adopsi, adaptasi, penerjemahan, atau kreasi. Pada proses adopsi,
sebuah kata diserap tanpa mengubah bentuknya.
Contoh:
bus film plaza
data internet radio

Pada proses adaptasi, sebuah kata diserap dengan cara mengadaptasi penulisan dan
pelafalannya ke dalam bahasa Indonesia.
Contoh:
active aktif (c → k), (ive → if)
athlete atlet (th → k)
check cek (ch → c), (ck → k)
central sentral (c → s)
extra ekstra (x → ks)
photo foto (ph → f)
Ramadhan Ramadan (dh → d)
stress stres (penghilangan konsonan ganda di akhir kata)
system sistem (y → i)
university universitas (ity → itas)
variant varian (penghilangan konsonan di akhir kata)
Pada proses penerjemahan, sebuah kata diterjemahkan atau dicarikan padanan kata dalam
bahasa Indonesia.
Contoh:
de jure berdasarkan hukum online daring/dalam jaringan
de facto berdasarkan fakta overlap tumpang tindih
inactive takaktif psychologist ahli psikologi

Pada proses kreasi, konsep sebuah kata diambil dan diterjemahkan atau dicarikan padanan kata
dalam bahasa Indonesia. Meskipun sekilas sama dengan proses penerjemahan, proses kreasi
lebih bebas dalam penerjemahan katanya.
Contoh:
baby sitter pramusiwi start up perusahaan rintisan
contact person narahubung stand up comedy komedi tunggal
gadget gawai talkshow gelar wicara

B. Kata Turunan
Terdapat tiga bentuk kata turunan: (1) kata berimbuhan, (2) bentuk terikat, (3) bentuk ulang, dan
(4) gabungan kata. Kata berimbuhan merupakan kata yang mendapatkan imbuhan (awalan,
sisipan, akhiran, serta gabungan imbuhan). Penulisan kata berimbuhan ditulis serangkai antara
kata dasar dengan imbuhannya.
Contoh:
berlari gelegar duniawi keteladanan
diberi jemari jabatan pembangunan
menulis kinerja tulisan perbaikan

Kata yang mendapatkan bentuk terikat juga termasuk kata turunan. Penulisan kata ini
dirangkaikan antara kata dasar/berimbuhan dengan bentuk terikatnya atau dirangkaikan
dengan tanda hubung, kecuali bentuk terikat maha.
Contoh:
dwiwarna pascakemerdekaan anti-Barat anti-mainstream Maha Esa
ekstrakurikuler prajabatan non-WNI pasca-reshuffle Maha Kuasa
inframerah tunawicara pro-Jokowi pra-reshuffle Maha Agung

Bentuk ulang merupakan kata yang mendapatkan perulangan. Bentuk ulang ditulis dengan
menggunakan tanda hubung (-) di antara unsur-unsurnya.
Contoh:
anak-anak mencari-cari porak-poranda cas-cis-cus
buku-buku menerka-nerka sayur-mayur dag-dig-dug
kura-kura mondar-mandir tunggang-langgang was-wes-wos

Gabungan kata merupakan hasil penggabungan beberapa kata. Penulisan gabungan kata dapat
dipisah, dirangkai, atau dirangkaikan dengan tanda hubung (-).
Contoh:
duta besar acapkali buku-sejarah baru
ibu kota bagaimana buku sejarah-baru
simpang lima sukarela
C. Pemenggalan Kata
Pemenggalan kata merupakan pemotongan kata berdasarkan suku kata dengan tujuan dapat
dituliskan dan dilafalkan dengan baik.
Contoh:
la-ri pan-dai
ber-la-ri mu-sya-wa-rah

D. Kata Depan
Kata depan atau preposisi berfungsi untuk membantu Menyusun kalimat yang menunjukkan
suatu tempat atau lokasi.
Contoh:
di Di mana tempat tinggalmu? Saya tinggal di Jakarta.
ke Ke mana kamu akan pergi? Saya akan pergi ke luar kota.
dari Dari mana kamu datang? Saya dari Surabaya.

E. Partikel
Partikel adalah kata tugas yang tidak memiliki fungsi tertentu. Partikel memiliki makna
gramatikal namun tidak memiliki makna leksikal. Artinya, partikel memiliki makna tertentu jika
digunakan dalam kalimat, tetapi tidak memiliki makna jika berdiri sendiri. Penulisan partikel
dapat dirangkai dan dipisah.
Contoh:
-lah Bacalah suratku nanti!
-kah Apakah kamu membaca surat itu?
-tah Apatah guna bersedih hati?
pun Apa pun makanannya, minumnya jangan lupa.
Meskipun sibuk, ia tetap sempat berkabar.
per Harga mangga itu Rp20.000,00 per kilo.

F. Singkatan
Singkatan adalah bentuk pendek dari suatu kata. Singkatan dapat ditulis berdasarkan huruf
awal setiap kata, suku kata setiap kata, dan atau gabungan huruf awal dengan suku kata.
Contoh:
Singkatan nama orang A.H. Nasution Abdul Haris Nasution
Singkatan gelar H. Ahmad Haji Ahmad
Singkatan sapaan Suman Hs. Suman Hasibuan
Singkatan pangkat Kol. Bambang Kolonel Bambang
Singkatan nama diri BIN Badan Intelijen Negara
Singkatan bukan nama diri KTP kartu tanda penduduk
Singkatan dua huruf a.n. atas nama
no. nomor
Singkatan tiga huruf dll. dan lain-lain
Singkatan ukuran/takaran kg kilogram
Akronim nama diri Bulog Badan Urusan Logistik
Akronim bukan nama diri iptek ilmu pengetahuan dan teknologi
G. Angka dan Bilangan
Angka merupakan lambang bilangan seperti 1, 2, 3, dst; sedangkan bilangan merupakan nama
angka yang dinyatakan dengan huruf seperti satu, dua, tiga, dst.
Contoh:
Angka Arab 0, 1, 2, 3, 4, 5, 6, 7, 8, 9
Angka Romawi I, II, III, IV, V, VI, VII, VIII, IX, X, L, C, D, M
Angka dalam teks Di lemari itu tersimpan 25 naskah kuno.
Di antara 72 anggota yang hadir, 52 orang setuju, 15 orang
tidak setuju, dan 5 orang abstain.
Angka ukuran/nilai 0,5 meter 5 liter 5 tahun
1% 1 persen Rp1.000,00
Angka dan bilangan besar Sebanyak 500 ribu dosis vaksin telah didistribusikan.
Angka untuk alamat Jalan Kartika I No. 15
Angka untuk bagian kitab Bab II, Pasal 3, halaman 13
Angka dan akhiran Penyanyi itu tenar pada tahun 90-an.
Bilangan dalam teks Mereka menonton drama itu sampai tiga kali.
Bilangan utuh tujuh tujuh belas tujuh puluh satu
Bilangan pecahan setengah
Bilangan tingkat abad ketujuh
Bilangan untuk pembilang Telah terima uang sebesar Rp1.000.000,00 (satu juta rupiah)
untuk pembayaran perbaikan kompor.
Bilangan nama geografi Kelapadua Simpanglima Tigaraksa

Pemilihan angka dan bilangan pada teks


Penulisan penulisan angka atau bilangan pada teks sesuai dengan EYD V diatur sebagai berikut.
kuantitas yang dinyatakan satu kata ditulis Hari ini adalah hari ulang tahun adik ke
dengan bilangan sebelas.
kuantitas yang dinyatakan lebih dari satu Dia memiliki kucing sebanyak 12 ekor di
kata ditulis dengan huruf rumahnya.

H. Kata Ganti
Kata ganti atau pronomina merupakan kata yang digunakan untuk menunjuk subjek atau objek
tanpa harus mengulang penyebutan namanya.
Contoh:
-ku Rumah itu milikku.
ku- Rumah itu sudah kujual.
-mu Apakah itu bukumu?
-nya Apkah itu benar miliknya?
kau Kau urus saja dirimu!

I. Kata Sandang
Kata sandang merupakan kata tugas yang membatasi nomina.
Contoh:
si Paket itu sudah sampai di rumah si penerima.
Dalam cerita itu, si Pitung berhasil menolong penduduk.
sang Harimau itu marah sekali kepada sang Kancil.
Kita harus berserah diri kepada Sang Pencipta.
1. Penggunaan huruf kapital yang benar Jateng akan menertibkan Radio Siaran
terdapat pada kalimat … Pemerintah Daerah.
(A) Kita harus berusaha menggunakan
Bahasa Indonesia yang baik dan benar. 4. Kalimat berikut ditulis dengan huruf
(B) Pada Bulan Agustus, ia akan berangkat kapital yang benar, kecuali …
ke Amerika. (A) Bila berkunjung ke Makassar, Ibu Kota
(C) Dimana banyak terdapat Suku Jawa? Provinsi Sulawesi Selatan, jangan lupa
(D) Pegunungan yang membentang di mampir di toko cendera mata.
daratan Sumatra itu bernama Bukit (B) Sepanjang perjalanan, kapal yang kami
Barisan. tumpangi ke Kepulauan Krakatau
(E) Dengan gembira, masyarakat terus diguncang gelombang.
menyambut Hari Lebaran. (C) Kebebasan dapat Anda rasakan saat
bersepeda di jalan Lingkar Luar
2. Huruf kapital dipakai dengan huruf Jakarta.
pertama petikan langsung di bawah ini (D) Perjalanan kami lanjutkan ke Desa
semuanya benar, kecuali … Pandes Sempokan.
(A) Adik bertanya, “Kapan kita pulang?” (E) Di antara dominasi warna terakota
(B) Bapak menasihatkan, “Berhati-hatilah, keramik dan gerabah yang memenuhi
Nak!” sisi jalan, kami menemukan Toko
(C) “Kemarin engkau terlambat,” katanya. Hasta Kreasi yang menjual patung
(D) “Besok pagi,” kata ibu, “dia akan keramik.
berangkat”
(E) “apa yang kau makan itu?” kata Ibu. 5. Penulisan nama dan jabatan yang tepat
terdapat pada kalimat …
3. Penulisan huruf kapital yang mengikuti (A) Ayahnya adalah seorang Gubernur.
EYD terdapat dalam kalimat berikut … (B) Siapa Bupati yang baru dilantik itu?
(C) Seminggu yang lalu Kolonel Haryono
(A) Senin malam banjir lahar kembali meninggal dunia.
terjadi di aliran Kali Putih dan sempat (D) Brigjen Sumadi baru dilantik menjadi
menerobos areal dekat pasar Jumojoyo. Mayor Jenderal.
(B) Setelah sejak lama diserahkan ke DPR (E) Presiden baru saja melantik beberapa
RI pemerintah melalui Mendagri akan Menteri.
memberikan penjelasan mengenai
rancangan Undang-Undang 6. Pemakaian huruf miring atau garis bawah
Keistimewaan DIY di harapan Komisi dibenarkan, kecuali untuk …
II. (A) Satu-satunya majalah sastra di
(C) Enam partai kecil di Salatiga akan Indonesia adalah Horison.
menggetarkan Pemilihan Umum (B) Jika akan berkemah, kita juga
Kepala Daerah (Pemilukada) kota memerlukan kompas.
Salatiga yang digelar pada 8 Mei 2013 (C) Politik devide et impera pernah
mendatang. merajalela di sini.
(D) Sebagaimana diungkapkan oleh Jassin, (D) Siapakah pengarang Layar Terkembang?
surat dari PSSI yang berisi penegasan (E) Huruf pertama kata abad adalah a.
bahwa Piala AFF bukanlah gratifikasi
itu telah diterima KPK.
(E) Dalam usaha menegakkan aturan dan
menghilangkan kesan tidak adil
Komisi Penyiaran Indonesia Daerah
7. Pemakaian huruf miring atau garis bawah (E) Setelah harga minyak naik pada
dibenar kecuali untuk …. Oktober 2005, impor minyak terus
(A) nama orang atau nama instansi atau menurun menjadi dua belas juta barel
lembaga pada Desember 2005, 8,2 juta barel
(B) menegaskan bagian kata-kata, atau pada Januari 2006, dan 6,4 juta barel
kelompok kata untuk impor Februari 2006.
(C) menuliskan kata nama-nama ilmiah
(D) menuliskan ungkapan asing, kecuali 11. Penulisan kata bilangan dalam kalimat
yang disesuaikan ejaannya berikut benar, kecuali …
(E) menuliskan nama buku dan majalah (A) Penulisan bilangan tiga dua pertiga
yang dikutip dalam karangan adalah 3
2
3
(B) Pada halaman 1.024 tertera nomor
8. Penulisan kata bilangan yang benar telepon 760130.
terdapat pada kalimat … (C) Tamu undangan yang hadir dalam
(A) 15 orang tewas dalam kecelakaan itu. perhelatan itu diperkirakan mencapai
(B) Kami memerlukan 10 (sepuluh) buah lima ratus orang.
pos pegawai. (D) Ulang tahun ke-20 adalah peristiwa
(C) Untuk keperluan itu, kami harus penting bagi orang Jepang.
menyiapkan uang 1.000 an. (E) Ia telah membaca 2 bab dari 5 buku itu.
(D) Dua ratus lima puluh orang diundang
pada pertemuan malam itu. 12. Penulisan bilangan dalam kalimat yang
(E) Anna menonton drama itu sampai tiga sesuai dengan aturan EYD adalah …
kali. (A) Anda dapat membeli buku ini seharga
Rp50.000,00 di toko saya.
9. Penulisan bilangan sesuai ejaan yang tepat (B) Sebanyak sembilan puluh enam
terdapat pada kalimat … kelurahan di Jakarta rawan demam
(1) Lima belas anak mengikuti lomba itu. berdarah dengue (DBD).
(2) Kedua anak itu akrab. (C) Pinjaman asosiasi pembangunan
(3) Ia anak kedua dari keluarga Arman. internasional biasanya diberikan
(4) 256 menghadiri pertemuan itu. dengan jangka waktu tiga puluh lima
tahun, masa tenggang sepuluh tahun,
10. Penulisan bilangan yang salah terdapat dan tingkat suku bunga tetap 0,75%
dalam kalimat … per tahun.
(A) Tugu Siger yang mempunyai tinggi 50 (D) Ratusan orang yang berunjuk rasa
meter dibangun pada tahun anggaran mengaku perwakilan warga Dayak
2005/2006. dari 13 kabupaten dan kota di
(B) Dalam Alquran pada halaman 1260 Kalimantan Timur.
termuat firman Tuhan yang (E) Anda perlu memiliki sedikitnya 2
menyampaikan perintah untuk kecerdasan utama untuk bisa menjadi
berbuat baik kepada sesama. seorang manajer yang andal.
(C) Rapat akan diadakan pada pukul
13.00-15.00 di Gedung 5.
(D) Dana luncuran Anggaran dan
Pendapatan Belanja Negara tahun
2000/2001 sebesar Rp15,5 triliun baru
terlaksana sekitar 30%.
13. Penulisan unsur gabungan kata yang 18. Penulisan kata-kata berikut benar,
dipakai dalam kombinasi di bawah ini kecuali ….
yang benar adalah kecuali …. (A) akhirat, akta, akbar
(A) antibiotik (B) akhlak, ikhlas, amendemen
(B) dasa warsa (C) akidah, akhir, lembap
(C) dekameter (D) akseptabel, akseptor, akselerasi
(D) pascasarjana (E) faksimili, aktifitas, piranti
(E) rumah tangga
19. Kalimat yang ditulis secara tepat
14. Kata gabung yang ditulis secara tepat adalah …
adalah …. (A) Harga buku yang difotokopi ini,
(A) modal linear, orang tua, olahraga Rp 15.000,- per jilid.
(B) persegi Panjang, bumi putera, olah (B) Harga buku yang di-photocopi ini,
raga Rp 15.000,- per jilid.
(C) puspa warna, radio aktif, saptapesona (C) Harga buku yang difotokopi ini,
(D) pimpang empat, rumah sakit, darma Rp 15.000,00,- per jilid.
siswa (D) Harga buku yang diphotocopy ini,
(E) acap kali, bea siswa, kaca mata Rp 15.000,- per jilid.
(E) Harga buku yang difotokopi ini
15. Penulisan unsur serapan yang berikut ini Rp15.000,00 per jilid.
benar, kecuali ….
(A) sistem, apotek, atlet 20. Kata bercetak miring dalam kalimat
(B) akwarium, kwitansi, sintesis berikut ini termasuk kata baku, kecuali …
(C) analisis, hipotesis, sintesis (A) Hari ini akan dilakukan gladi resik
(D) formal, tradisional, rasional upacara wisuda.
(E) trotoar, dresoar, repertoar (B) Mereka menghadapi masalah yang
16. Penulisan gabungan kata di bawah ini sangat kompleks.
benar semuanya, kecuali …. (C) Dalam menghadapi masalah
(A) tanggung jawab, kerjasama, kelompok, tidak boleh terlalu ekstrim.
tandatangan, terima kasih (D) Orang muda harus bekerja keras untuk
(B) semipermanen, mancanegara, mengembangkan kariernya.
pascasarjana, saptakrida (E) Pemimpin itu memiliki karisma yang
(C) acap kali, adakalanya, barangkali, tinggi.
bagaimana
(D) mata ajar, meja tulis, orang tua, 21. Penulisan kata bercetak miring yang benar
simpang empat terdapat pada kalimat …
(E) kacamata, dukacita, saputangan, (A) Para mahasiwa diwajibkan membayar
halalbihalal SPP persemester.
(B) Rumah itu terpaksa di jual karena perlu
17. Penulisan gabungan kata ini baku, uang.
kecuali …. (C) Pertandingan basket antarmahasiswa
(A) mitrabestari, matahari, tatabahasa berlangsung seru.
(B) bilamana, saputangan, darmabakti (D) Siapakah diantara Anda yang bersedia
(C) peribahasa, sumber daya, syahbandar membantu kami?
(D) halalbihalal, acap kali, cita rasa (E) Sekali pun kami belum pernah ke sana.
(E) dukacita, sukaria, tunarungu
22. Penulisan gabungan kata yang benar 25. Penulisan kata serapan yang sesuai dengan
terdapat dalam kalimat … aturan EYD adalah …
(A) Senjata yang digunakan militer (A) Perlu adanya standarisasi dalam mutu
memanfaatkan infra merah. produksi sabun.
(B) Tawuran antarsiswa itu berhasil (B) Agaknya telah terjadi marginalisasi
diselesaikan dengan damai. peran terhadap kelompok tertentu.
(C) Perusahaan itu membutuhkan pekerja (C) Perubahan positif sangat diharapkan
yang serba bisa. entah dalam bentuk material ataupun
(D) Tujuan daripada seminar itu mencari non material.
jalan ke luar dari krisis. (D) Keterampilan managerial yang tinggi
(E) Kerjasama siswa saat ujian dilarang. sangat diperlukan bagi setiap
pemimpin.
23. Penulisan kata yang tidak tepat terdapat (E) Respon masyarakat terhadap rencana
pada kata-kata bercetak miring dalam perbaikan kampung sangat baik.
kalimat …
(A) Warga kompleks membangun gedung 26. Salah satu penyebab sepinya penonton di
pertemuan secara swadaya. stadion dalam kejuaran sepak bola adalah
(B) Pada saat pascapanen, harga gabah kwalitas pertandingan yang belum baik.
sering merosot. Dalam hal ini pihak pelatih dan ofisial juga
(C) Setiap bus antarprovinsi harus dalam bertanggung jawab membentuk team yang
kondisi laik jalan. andal.
(D) Para tunawisma di bawah jembatan
layang akan ditertibkan. Kata yang penulisannya tidak sesuai
(E) Banyak pelajar melakukan kerjasosial dengan EYD adalah ….
untuk mengisi liburan. (A) pihak dan sepak bola
(B) stadion dan andal
24. Penulisan gabungan kata yang sesuai (C) kwalitas dan team
dengan EYD dalam kalimat di bawah ini (D) stadion dan ofisial
adalah … (E) pihak dan bertanggung jawab
(A) Setiap Hari Raya Idul Fitri keluarga
besar itu selalu mengadakan acara 27. (1) Air selalu menjadi atraksi yang sangat
halalbihalal. menarik, bahkan sejak zaman kuno
(B) Jika ingin membaca, ayah selalu berbagai keajaiban dunia berada di tepian
mengenakan kaca mata. air. (2) Opera House Sidney (ikon
(C) Adik saya selalu membawa sapu Australia), Patung Liberty (ikon Amerika
tangan hijau kesayangannya kalau Serikat), dan Merlion (ikon Singapura)
sedang pergi ke luar kota. berada di wilayah teluk dan merupakan
(D) Mereka memberikan bantuan secara simbol nasional ketiga Negara tersebut
suka rela kepada para korban bencana yang sangat melekat di ingatan orang di
alam itu. seluruh dunia. (3) Dapat dicatat pula
(E) Mereka turut berdukacita atas bahwa di wilayah Nusantara berkembang
meninggalnya salah satu teman dua pola besar, yaitu wilayah tepian air
mereka. atau pesisir dan daratan di pedalaman
yang subur. (4) Sunda Kelapa dan
Jembatan Ampera menjadi ikon Jakarta
dan Palembang meskipun keduanya
sekarang terus berjuang mengembalikan
kharismanya. (5) Selain atraksi, maka
sungai yang dianggap sebagai salah satu (D) Penulis beranggapan bahwa praktek
elemen kehidupan juga digunakan untuk perladangan, sebagai contoh,
berbagai ritual, misalnya Sungai Gangga merupakan suatu keahlian yang
(India) hingga hari ini berperan penting didasari pengalaman tradisional dan
dalam kehidupan spiritual bangsa India. sama sekali tidak primitif.
(E) Makin banyak lulusan sekolah
Dalam bacaan di atas terdapat kata yang menegah atas yang berminat
tidak baku, yaitu …. mendalami bidang studi akutansi.
(A) zaman (kalimat 1)
(B) tepian (kalimat 1) 30. Pembentukan kata serapan yang betul
(C) simbol (kalimat 2) terdapat dalam kalimat …
(D) kharisma (kalimat 4) (A) Di dunia olahraga yang dipentingkan
(E) ritual (kalimat 5) adalah sportivitas.
(B) Sikap yang kaku dari seorang
28. Kata bercetak miring dalam kalimat pemimpin menimbulkan watak arogan
berikut ini baku kecuali … dan otoritair.
(A) Pertemuan itu dihadiri para (C) Majelis juga mengusulkan agar
rohaniawan dari berbagai negara. kalangan eksekutip dan legislatip peka
(B) Heroisme di mana pun tidak pernah terhadap aspirasi rakyat.
terlepas dari situasi sosial politik. (D) Mereka juga memprotes kwalitas beras
(C) Membuat masyarakat gemar yang mereka terima.
berolahraga bukanlah pekerjaan yang (E) Rapat itu gagal karena yang hadir
mudah. tidak memenuhi korum.
(D) Untuk memercayai seseorang
diperlukan kehati-hatian. 31. Penulisan kata yang bercetak miring dalam
(E) Untuk memecahkan masalah tersebut, kalimat berikut yang sesuai dengan EYD
diperlukan daya analisis yang tinggi. adalah …
(A) Hasil analisa peneliti itu sangat
29. Penggunaan kata serapan yang tepat berguna bagi masyarakat dan
terdapat dalam kalimat … perkembangan.
(A) Dari aspek bahasa, kata kampanye (B) Dalam sistim pendidikan yang baik,
berasal dari bahasa Inggris, yaitu to proses pendidikan dapat
campaign yang berarti ‘serangkaian diselenggarakan di dalam maupun di
aktivitas yang direncanakan atau suatu luar kelas.
operasi militer’ (Evison, 1983) (C) Konduite merupakan salah satu syarat
(B) Sebagai penyusun buku ini kami untuk naik pangkat.
berharap buku ini dapat menggugah (D) Pendapatnya sangat ekstrim sehingga
generasi muda perempuan bahwa para peserta rapat tidak
dalam berkeluarga membina karir menyetujuinya.
masing-masing adalah suatu (E) Tehnik penulisan ilmiah harus dapat
kemungkinan. dipahami oleh para mahasiswa di
(C) Dengan menerbitkan buku ini kami perguruan tinggi.
ingin menyumbangkan sesuatu yang
konkrit tentang aspirasi perempuan
Indonesia sebagai manusia dan
anggota masyarakat yang sedang
membangun.
32. Penulisan kata serapan yang benar (E) Komunikasi merupakan syarat
terdapat dalam kalimat … esensial bagi terciptanya prasyarat
(A) Setiap wanita WNI yang menikah tumbuhnya kegiatan yang terbuka
dengan pria WNA harus siap yang tidak elit.
menanggung konsekwensi bahwa
status kewarganegaraan anaknya akan 35. Penulisan kata serapan yang sesuai dengan
mengikuti suaminya. aturan EYD adalah …
(B) Kita perlu segera mengkonkritkan (A) Perlu adanya standarisasi dalam mutu
gagasannya itu. produksi sabun.
(C) Penduduk Pasifik yang kebanyakan (B) Agaknya telah terjadi marginalisasi
tinggal di pulau karang adalah peran terhadap kelompok tertentu.
kelompok yang paling beresiko. (C) Perubahan yang positif sangat
(D) Kami mengusulkan agar salah satu di diharapkan entah dalam bentuk
antara kita bersedia mengkoordinir
material ataupun non material.
pelaksanaan kegiatan ini.
(D) Keterampilan managerial yang tinggi
(E) Dia berhasil memopulerkan jenis
sangat diperlukan bagi setiap
olahraga berbahaya itu di negaranya.
pemimpin.
(E) Respon masyarakat terhadap rencana
33. Penulisan kata serapan dalam kalimat-
perbaikan kampung sangat baik.
kalimat berikut ini benar, kecuali yang
terdapat pada kalimat …
36. Kata serapan yang salah penulisannya
(A) Setiap pagi kumandang azan itu
membangunkan saya. terdapat dalam kalimat …
(B) Muna bekerja di laboratorium itu (A) Pembangunan fisik tanpa
sebagai bakteriolog. pembangunan mental seumpama
(C) Orang yang mengusahakan bank bangunan tanpa pondamen.
disebut banker. (B) Basket adalah olahraga favoritnya.
(D) Keamanan di ibu kota merupakan (C) Perjanjian antara dua pihak disebut
barometer keamanan di Indonesia. perjanjian bilateral.
(E) Sebelum meninggal, N sempat (D) Gadis itu menjadi sangat ideal untuk
menyelesaikan otobiografinya. menjadi peragawati.
(E) Aku jarang pergi ke bioskop.
34. Bentuk kata serapan yang benar terdapat
dalam kalimat… 37. Setiap kata ditulis secara benar dalam
(A) Radio pada mobil merupakan asesoris kalimat …
yang digemari oleh banyak konsumen. (A) Di samping itu, ia juga diminta untuk
(B) Tarian itu dianggap akseptebel untuk menyelesaikan laporan proyek.
ditampilkan di depan tamu.
(B) Di antara sepuluh orang, hanya tiga
(C) Pada jaman penjajahan di kota Malang
orang yang berani masuk ke dalam
terbit banyak koran, koran-koran
arena pertandingan.
tersebut tidak dapat terbit secara
(C) Kerjasama di antara mereka sudah
kontinyu.
diantisipasi oleh kelompok lawannya.
(D) Pematuhan terhadap prinsip kerja
(D) Ia tidak pernah lupa bersyukur ke
sama dan prinsip kesopanan akan
hadirat Tuhan Yang Maha Kuasa.
melahirkan komunikasi yang bonafide
sekaligus melahirkan pula wacana (E) Dari dua puluh sembilan korban, lima
yang wajar. di antaranya adalah anak pejabat.
38. Kata bercetak miring dalam kalimat 41. Penulisan kalimat berikut yang tidak
berikut ini termasuk kata baku, kecuali … sesuai dengan kaidah EYD, kecuali…
(A) Masalah konkret yang sedang kita (A) Dunia kita sekarang memang jauh
hadapi sekarang ini adalah bencana lebih komplek dibandingkan dengan
alam. masa yang lalu karena, tehnologi yang
(B) Imunisasi haemopilus influenza makin canggih telah membantu orang
digunakan untuk menguatkan daya lebih efisien dan cerdas, juga lebih
tahan tubuh. terbuka.
(B) Kini di era perkembangan teknologi
(C) Semoga Tuhan Yang Maha Pengasih
informasi, kita dengan mudah bisa
mengabulkan doa kami.
diteropong dan meneropong
(D) Varieties padi unggul tahan wereng
pendapat, pemikiran, emosi,
sekarang jarang didapati.
kedalaman etika, bahkan tatakrama
(E) Jenis daya tahan tubuh yang berupa kita lewat status facebook, YM, juga
kolostrum banyak dijual bebas di berbagai forum diskusi di berbagai
apotek. situs di internet.
(C) Gus Dur telah berpulang, kata
39. Bentuk kata yang digunakan dalam rohaniawan itu. Tetapi ajarannya
kalimat-kalimat di bawah ini tidak baku, tentang demokrasi dan toleransi
kecuali … beragama masih perlu dikembangkan
(A) Kita harus dapat memanfaatkan waktu dalam masyarakat kita yang multi
seselektif mungkin. kultural.
(B) Kemampuan masing-masing pemain (D) Meskipun sering kontroversial,
menunjukkan kemampuan yang pengaruh Gus Dur terbukti masih
berarti. sangat kuat dan diikuti oleh ribuan
(C) Polda Jabar turut mensukseskan WTJ santrinya.
2005. (E) Presiden SBY mengatakan, dalam
(D) Perda itu kurang efektif untuk penyusunan draft teks baru
memberantas perjudian yang ada di mengakomodasikan masukan para
kepala negara pada pertemuan
Jawa Barat.
Konfrensi Perubahan Iklim PBB harus
(E) Pada babak Kwalifikasi pertama,
dicantumkan keseimbangan kemitraan
Alonso mencatat waktu 1.29 menit.
yang adil antara negara maju dan
berkembang dalam pengelolaan hutan.
40. Penulisan kalimat yang tidak sesuai
dengan EYD adalah … 42. Kalimat yang penulisannya mengikuti
(A) Artikel yang berjudul “Sastra dan EYD adalah …
Kekuasaan” dimuat dalam majalah (A) Sepuluh tahun yang lalu Keluarga
Horison. Satrio tinggal di jalan RE Martadinata,
(B) Ketika antre di tempat itu, Anda harus No. 5.
siap fisik dan mental. (B) Sebagian besar dari kita beranggapan
(C) Acara akan berlangsung dari pukul bahwa kebudayaan Timur lebih halus
07.30 s.d. 11.30. jika dibandingkan kebudayaan Barat.
(D) Halal bihalal dilakukan di sebuah (C) Ia mengakui ke dalaman wawasan
gedung mewah. tokoh masyarakat kampung Naga itu.
(E) Alat pandang-dengar digunakan pada (D) Pengusaha besar itu menghadiahi atlit-
acara tersebut. atlit berprestasi internasional dengan
bea siswa untuk belajar keluar negeri.
(E) Beberapa tokoh anti rasisme dari
beberapa negara sepakat untuk
menanda tangani suatu kesepahaman.
43. Kalimat yang ditulis sesuai dengan aturan 45. Kata-kata yang bercetak miring dalam
EYD adalah … kalimat berikut yang penulisannya
(A) Tiga puluh enam orang terperangkap mengikuti EYD adalah …
dalam longsoran itu. (A) Hakikat hidup adalah mengabdi
(B) Di sana-sini terdengar tangis yang kepada Tuhan Yang Maha Kuasa.
memilukan. (B) Analisa para peneliti itu ditentang oleh
(C) Penanganan musibah itu diambil alih kaum birokrat.
oleh pemerintah pusat. (C) Harga obat Apotek Sari Waras lebih
(D) Kata nominator tidak berarti “yang murah dibandingkan dengan apotek
dinominasikan.” lainnya.
(E) Cara penanganan masalah tersebut (D) Surat kuasa tersebut telah disyahkan
menimbulkan ketidakpuasan. oleh pejabat yang berwenang.
(E) Orang-orang yang disebut sebagai
44. Kalimat berikut ini yang seluruhnya ditulis para normal biasanya memiliki
sesuai dengan aturan ejaan yang baku kelebihan-kelebihan tertentu.
adalah …
(A) Hampir setiap buku Bahasa Indonesia
membahas masalah ejaan.
(B) Dalam tulisannya masih banyak
terdapat kesalahan ejaan, walaupun ia
sudah tahu aturan ejaan yang baku.
(C) Kata-kata baku dalam teks ilmiah
penulisannya perlu digaris-bawahi
atau dicetak miring.
(D) Kata universitas sebagai nama diri
ditulis dengan huruf awal kapital.
(E) Setiap siswa perlu memiliki buku
Pedoman Ejaan Yang Disempurnakan.
TANDA BACA

Tanda baca memiliki fungsi yang penting dalam wacana. Aturan-aturan penulisan tanda baca
diatur dalam EYD (terbaru edisi V, 2022). Tanda baca meliputi tanda titik (.), tanda koma (,), tanda titik
koma (;), tanda titik dua (:), tanda hubung (-), tanda pisah (−), tanda tanya (?), tanda seru (!), tanda
ellipsis (…), tanda petik (“…”), tanda petik tunggal (‘…’), tanda kurung ((…)), tanda kurung siku ([…]),
tanda garis miring (/), dan tanda apostrof (‘).

A. Tanda Titik (.)


digunakan pada akhir kalimat pernyataan • Kondisi kebahasaan di Indonesia
membutuhkan penanganan yang tepat.
• Kondisi kebahasaan di Indonesia yang
diwarnai oleh bahasa standar dan
nonstandar, ratusan bahasa daerah, dan
ditambah beberapa bahasa asing
membutuhkan penanganan yang tepat
dalam perencanaan bahasa.
digunakan di akhir angka/huruf dalam I. Kondisi Kebahasaan di Indonesia
daftar/perincian/tabel/bagan A. Bahasa Indonesia
1. Kedudukan
2. Fungsi
memisahkan angka jam, menit, dan detik • Ia berjanji akan datang pukul 10.00.
• Nyatanya ia datang pukul 10.05.20.
• Ia menghabiskan 01.15.20 jam untuk
mengerjakan sertaus butir soal latihan.
memisahkan bilangan ribuan /kelipatannya • Indonesia memiliki lebih dari 13.000 pulau.
yang menunjukkan jumlah • Anggaran lembaga itu mencapai
Rp225.000.000.000,00.

B. Tanda Koma (,)


memisahkan unsur-unsur dalam perincian • Ibu memasak nasi, sayur bayam, dan ayam
goreng.
• Mohon melengkapi surat pengajuan dengan
melampirkan
(1) fotokopi KTP,
(2) fotokopi BPJS kesehatan, dan
(3) fotokopi BPJS ketenagakerjaan.
digunakan setelah kata penghubung dalam • Ini bukan milik saya, melainkan milik kakak
kalimat majemuk pertentangan saya.
• Adiknya sedang berenang, sedangkan
kakaknya sedang bermain gim.
• Ayah ingin membeli mobil baru, tetapi
tabungannya belum cukup.
memisahkan anak kalimat yang mendahului • Karena memperoleh penghargaan, ia dipuji
induk kalimat banyak orang.
• Jika diundang, saya akan datang.
digunakan setelah kata penghubung • Ia anak yang ramah dan baik. Oleh karena itu,
antarkalimat banyak orang yang menyayanginya.
• Ia suka menulis sejak kecil. Jadi, tidak heran
jika ia menjadi penulis terkenal.
digunakan sebelum/sesudah kata seru dan • Oh, begitu rupanya.
kata sapaan • Wah, beruntung sekali, Anda!
• Hati-hati di jalan, ya, Kak!
memisahkan petikan langsung dari dari bagian • Kata nenek, “itu berbahaya!”
kalimat lain • “Lebih baik nikmati pahitnya belajar,
daripada pahitnya ketidaktahuan,” kata
guruku.
digunakan di antara nama dan alamat, bagian- • Siti Sayyidah, Jl. H Dimun Raya II/17,
bagian alamat, tempat dan tanggal, serta nama Kelurahan Sukamaju, Kecamatan Cilodong,
tempat dan wilayah Kota Depok.
• Medan, 10 April 2022
• Kembangan, Jakarta Barat, DKI Jakarta
digunakan sesudah salam pembuka, salam • Dengan hormat,
penutup, dan nama jabatan penanda tangan • Hormat saya,
surat • Kepala Sekolah SMA Tunas Bangsa,
Drs. Agung Setiaji, M.Pd.I.
digunakan untuk memisahkan nama dan gelar • Aina Mulyana, M.Pd.
• H. Anies Rasyid Baswedan, S.E., M.P.P.,
Ph.D.
digunakan sebelum desimal atau antara rupiah • Panjang rambutnya 0,52 m.
dan sen • Harga pertamax mengalami penurunan
menjadi Rp13.900,00.
digunakan untuk mengapit keterangan • Semua karyawan, baik laki-laki maupun
tambahan/aposisi perempuan, diwajibkan bekerja di kantor per
bulan depan.
digunakan di belakang keterangan yang • Dalam rapat itu, perubahan peraturan
terdapat pada awal kalimat perusahaan dibahas.
• Dalam rapat itu, dibahas perubahan
peraturan perusahaan.

C. Tanda Titik Koma (;)


digunakan sebagai pengganti kata hubung • Adiknya sedang berenang; kakaknya sedang
dalam kalimat majemuk setara bermain gim.
• Gitar dipetik; seruling ditiup; angklung
digoyang.
digunakan memisahkan perincian berupa frasa • Syarat mengikuti ujian, yaitu
verbal (1) berkewarganegaraan Indonesia;
(2) berbadan sehat; dan
(3) berijazah minimal SMA/sederajat.
digunakan memisahkan perincian yang sudah • Agenda rapat ini meliputi
menggunakan tanda koma (1) pengisian daftar hadir, pembukaan, dan
sambutan;
(2) laporan pertanggungjawaban, laporan
anggaran, dan penyusunan program
kerja; serta
(3) pembagian tanda mata dan penutupan.
digunakan untuk memisahkan sumber-sumber • Kasus perencanaan bahasa di Indonesia
kutipan dianggap sebagai salah satu yang paling
berhasil (Fishman, 1974; Moeliono, 1985;
Samuel, 2008; Wardhaugh dan Fuller, 2015).
D. Tanda Titik Dua (:)
digunakan pada akhir pernyataan lengkap • Ibu membeli sayur-mayur: bayam, jagung,
yang diikuuti perincian/penjelasan kangkung, dan daun papaya.
• Kami membutuhkan peralatan tukang:
martil, gergaji, dan alabangka.
digunakan setelah kata/frasa yang Ketua : Ahmad Wijaya
memerlukan pemerian Wakil Ketua : Deni Simanjuntak
Sekretaris : Siti Aryani
Bendahara : Aulia Arimbi
digunakan dalam dialog pada naskah drama Ibu : “Apakah kamu baik-baik saja, Nak?”
Andika : “Baik, Bu. Ibu bagaimana?”
Ibu : “Ibu juga.”
digunakan di antara jilid/nomor dan halaman, • Surah Ibrahim: 2–5
surah dan ayat, serta judul dan anak judul • Matius 2: 1–3
• Dari Pemburu ke Terapeutik: Antologi
Cerpen Mastera
memisahkan angka jam, menit, dan detik • Ia berjanji akan datang pukul 10:00.
• Nyatanya ia datang pukul 10:05:20.
• Ia menghabiskan 01:15:20 jam untuk
mengerjakan sertaus butir soal latihan.
digunakan menuliskan rasio yang menyatakan • Skala peta ini adalah 1:100.
perbandingan • Perbandingan jumlah peserta ujian laki-laki
dan perempuan adalah 3:2.

E. Tanda Hubung (-)


menandai bagian kata yang terpenggal bagian Perkembangan bahasa adalah proses berkem-
baris bangnya kemampuan seseorang untuk mema-
hami dan mengucapkan kata.
menyambung unsur bentuk ulang • anak-anak • warna-warni
• melambai-lambai • sayur-mayur
menyambung tanggal, bulan, dan tahun; • 22-2-2022
menyambung huruf dalam kata yang dieja; • l-i-t-e-r-a-s-i
serta menyatakan skor pertandingan • 2-1
memperjelas hubungan bagian kata atau • me-ngaji, meng-gaji, ber-evolusi, be-revolusi
ungkatan • tiga-puluh lima-ribuan, tiga-puluh-lima
ribuan
• mata-kaki, daun-telinga
merangkaikan unsur yang berbeda antara • se-Asia • ke-1
huruf kapital dan non kapital serta antara huruf • ber-KTP • tahun 2000-an
dan angka • ciptaan-Nya • S-1
merangkai unsur bahasa Indonesia dengan • mem-blow up • di-ruqyah
unsur bahasa daerah/asing/slang • meng-ghosting • di-slepet
menandai imbuhan yang menjadi objek • Imbuhan ber- pada kata belajar mengalami
bahasan alomorf.
menandai dua unsur yang merupakan satu • kakak-adik
kesatuan • Konferensi Asia-Afrika
F. Tanda Pisah (—)
mengapit keterangan/penjelasan dalam • Kemerdekaan bangsa itu—saya yakin akan
kalimat tercapai—diperjuangkan oleh bangsa itu
sendiri.
• Soekarno-Hatta—Proklamator Kemerdekaan
RI—diabadikan menjadi nama jalan di
beberapa kota di Indonesia.
Memisahkan antara bilangan/tanggal/tempat • Tahun 2019—2022 • Senin—Jumat
• Tanggal 5—10 April 2022 • Depok—Bekasi
G. Tanda Tanya (?)
mengakhiri kalimat tanya • Kapan Hari Pendidikan Nasional
diperingati?
• Siapa pencipta lagu "Indonesia Raya"?
menyatakan bagian kalimat yang diragukan • Monumen Nasional mulai dibangun pada
tahun 1961 (?).
• Di Indonesia terdapat 740 (?) bahasa daerah.

H. Tanda Seru (!)


mengakhiri ungkapan yang menggambarkan • Wow, indahnya!
kekaguman, kesungguhan, emosi yang kuat, • Bukan saya pelakunya!
seruan, atau perintah • Merdeka atau mati!
• Pergi!

I. Tanda Ellipsis (…)


menunjukkan bagian yang hilang/tidak • Penyebab kemerosotan … akan diteliti lebih
ditunjukkan lanjut.
• Ungkapan yang sesuai untuk
menggambarkan ilustrasi tersebut adalah ….
menulis ujaran yang tidak selesai dalam dialog Ayah : “Jangan menangis, laki-laki tidak
boleh menangis!”
Bari : “Tapi, Yah …”
Ayah : “Ah, tak usah tapi-tapi!”
menandai jeda panjang dalam tuturan yang • Maju ... jalan!
dituliskan • Kamera ... siap!
• Satu, dua, ... tiga!

J. Tanda Petik (“…”)


mengapit petikan langsung • "Merdeka atau mati!" seru Bung Tomo
dalam pidatonya.
• Menurut Pasal 31 ayat (1) Undang-Undang
Dasar Negara Republik Indonesia Tahun
1945, "Setiap warga negara berhak mendapat
pendidikan."
mengapit judul puisi, lagu, artikel, naskah, bab • Puisi "Pahlawanku" terdapat pada halaman
buku, pidato/khotbah, tema/subtema dalam 125 buku itu.
kalimat • Marilah, kita menyanyikan lagu "Maju Tak
Gentar"!
• Saya sedang membaca "Peningkatan Mutu
Daya Ungkap Bahasa Indonesia" dalam
buku Bahasa Indonesia Menuju Masyarakat
Madani.
• Makalah "Pembentukan Insan Cerdas
Kompetitif" menarik perhatian peserta
seminar.
• Perhatikan "Hubungan Antarklausa" dalam
buku Tata Bahasa Baku Bahasa Indonesia.
• Ceramah subuh minggu lalu di Masjid
Istiqlal berjudul "Hikmah dan Tujuan
Berpuasa Ramadan".
• Kongres Bahasa Indonesia XI bertema
"Menjayakan Bahasa dan Sastra Indonesia".
Mengapit istilah ilmiah yang memiliki arti Dilarang memberikan "amplop" kepada
khusus petugas!
K. Tanda Petik Tunggal (‘…’)
mengapit pertikan dalam petikan lain • Tanya dia, "Kaudengar bunyi 'kring-kring'
tadi?"
• "Kita bangga karena lagu 'Indonesia Raya'
berkumandang di arena Asian Games," kata
Ketua KONI.
mengapit makna, padanan, penjelasan, atau • retina 'dinding mata sebelah dalam'
ungkapan kata • tadulako 'panglima'
• lockdown 'karantina wilayah'

L. Tanda Kurung ((…))


mengapit keterangan tambahan, seperti Banyak pemengaruh (influencer) yang
singkatan atau padanan kata asing mendapat apresiasi karena konten yang
membangun.
mengapit keterangan atau penjelasan yang Keterangan itu (lihat Tabel 1.1) menunjukkan
bukan bagian utama kalimat arus perkembangan baru pasar dalam negeri.
mengapit kata yang keberadaannya dapat Pesepak bola kenamaan itu berasal dari (Kota)
dimunculkan atau dihilangkan Padang.
mengapit huruf atau angka sebagai penanda • Faktor produksi menyangkut (a) bahan
perincian yang ditulis ke samping atau ke baku, (b) biaya produksi, dan (c) tenaga
bawah kerja.
• Dia harus melengkapi berkas lamarannya
dengan melampirkan
(a) daftar riwayat hidup,
(b) ijazah terakhir, dan
(c) surat keterangan kesehatan.

M. Tanda Kurung Siku ([…])


mengapit huruf/kata/kelompok kata sebagai • Asmara[n]dana merupakan salah satu
koreksi naskah asli yang ditulis orang lain tembang macapat budaya Jawa.
• Dia sedang menyelesaikan [h]utangnya di
bank..
mengapit keterangan dalam kalimat penjelas Persamaan kedua proses itu (perbedaannya
yang terdapat dalam tanda kurung dibicarakan di dalam Bab II [lihat halaman 35–
38]) perlu dibentangkan di sini.
N. Tanda Garis Miring (/)
digunakan dalam nomor surat, nomor pada • Nomor: 7/PK/II/2022
alamat, dan penandaan masa tahun • Jalan Kramat III/10
• Tahun ajaran 2022/2033
digunakan sebagai pengganti kata dan, atau, • Semua organisasi harus memiliki AD/ART.
serta setiap • Pilih salah satu moda transportasi
darat/laut!
• Harga kain itu Rp75.000,00/meter.
mengapit huruf/kata/kelompok kata sebagai • Asmara/n/dana merupakan salah satu
koreksi naskah asli yang ditulis orang lain tembang macapat budaya Jawa.
• Dia sedang menyelesaikan /h/utangnya di
bank.

O. Tanda Apostrof (‘)


menunjukkan penghilangan bagian kata atau • Dia 'kan datang. ('kan = akan)
bagian angka tahun • Diriku s'lalu dimanja. (s'lalu = selalu)
• Tahun ‘90-an ('90 = 1990)
1. Penggunaan tanda baca yang salah (E) Berbagai persoalan yang harus
terdapat dalam kalimat … diselesaikan meliputi persoalan di
(A) Membaca buku hendaklah di tempat bidang politik, sosial, dan ekonomi
yang terang. serta keamanan.
(B) Dia tidak dapat membaca buku, karena
lampu padam. 5. (1) Kegiatan Program Nasional
(C) Supaya mata tetap sehat, membaca Pemberdayaan Masyarakat (PNPM) telah
buku di tempat yang terang. memberdayakan masyarakat miskin untuk
(D) Bacalah buku ini di tempat yang mengakses modal untuk usaha hanya
terang. dengan syarat, antara lain, membuat
(E) Rina, guru adik, sakit. kelompok swadaya masyarakat. (2) Di
beberapa wilayah perkotaan maupun
2. Penggunaan tanda baca yang benar pedesaan banyak bermunculan usaha
terdapat pada … pemula dengan modal antara
(A) Ibu Rully tidak setuju, karena gagasan Rp.500.000,00 hingga Rp.2.000.000,00. (3)
itu tidak masuk akal. Para pelaku usaha pemula ini kebanyakan
(B) Di Ciwalk terdapat: sepatu, baju, tas, hanya untuk bertahan hidup, bukan untuk
dan perabot rumah tangga. berwirausaha yang mampu memberi nilai
(C) Surat biasa, surat kilat, ataupun surat tambah. (4) Kondisi ini wajar, karena para
kilat khusus memerlukan prangko. pelaku usaha pemula ini pada awalnya
(D) Oleh karena itu kita harus rajin belajar. ingin keluar dari jeratan kemiskinan
(E) Joko, sahabat Wido mendapat melalui peluang yang dapat mereka
beasiswa. lakukan. (5) Mereka melakukan usaha
mandiri dengan modal kecil hanya secara
3. Penulisan tanda baca yang betul terdapat naluri, dan mengikuti tren yang
pada kalimat … berkembang. (6) Pemerintah, sebenarnya
(A) Jakarta Ibu Kota Republik Indonesia sangat terbantu dengan keinisiatifan
berhawa panas. masyarakat yang melakukan kegiatan
(B) Jakarta, Ibu Kota Republik Indonesia produktif.
berhawa panas.
(C) Jakarta, Ibu Kota Republik Indonesia, Dalam paragraf di atas terdapat kalimat
berhawa panas. yang mengandung kesalahan pemakaian
(D) Jakarta Ibu Kota Republik Indonesia, tanda baca, kecuali ….
berhawa panas. (A) kalimat (1) dan (3)
(E) Jakarta, Ibu Kota, Republik Indonesia, (B) kalimat (2) dan (4)
berhawa panas. (C) kalimat (3) dan (5)
(D) kalimat (5) dan (6)
4. Pemakaian tanda baca yang tepat terdapat (E) kalimat (2) dan (3)
dalam kalimat …
(A) Berbagai persoalan yang harus
diselesaikan meliputi persoalan di
bidang politik, ekonomi, sosial dan
keamanan.
(B) Berbagai persoalan yang harus
diselesaikan meliputi persoalan di
bidang politik, sosial, ekonomi, dan
keamanan.
(C) Berbagai persoalan yang harus
diselesaikan meliputi persoalan di
bidang politik, sosial dan ekonomi
serta keamanan.
(D) Berbagai persoalan yang harus
diselesaikan meliputi persoalan di
bidang politik, sosial ekonomi dan
keamanan.
6. Penerapan EYD berikut ini salah, kecuali … (D) Daur ulang terbukti lebih ramah
(A) Polisi menyita sejumlah uang palsu lingkungan daripada penguburan atau
pecahan Rp.100.000, Rp.10.000, dan pembakaran sampah, tetapi hal ini
Rp.1.000. tidak disadari oleh sebagian besar
(B) Berdasarkan asumsi harga minyak masyarakat.
rata-rata sebesar 100 dollar AS per (E) Sejauh ini, Eropa telah memimpin
barel, inflasi meningkat dari 5,3 persen dunia dalam bahan-bahan kemasan
menjadi 7,6 persen. daur ulang.
(C) Harga rata-rata minyak 71,5 dolar AS
per barel, lebih tinggi jika 9. Pemakaian tanda baca yang betul terdapat
dibandingkan dengan harga rata-rata pada …
tahun sebelumnya sebesar 67 dolar AS (A) Indra rajin dan pandai, karena itu,
per barel. nilainya bagus.
(D) Kami memerkirakan ekonomi akan (B) Kemarin, semua orang hadir kecuali
tumbuh 6,4 persen pada 2008 dan 6,5 Anda.
persen pada 2009. (C) Sari ingin bersekolah tetapi ibu tidak
(E) Diperkirakan nilai tukar rupiah mampu membiayainya.
terhadap dolar melemah menjadi (D) Kemarin ia datang, dan ia pergi.
Rp10.000,00 per dolar pada akhir tahun (E) Jakarta, Ibu Kota RI, berhawa panas.
2008.
10. Kalimat-kalimat berikut ini tidak dapat
7. Kalimat yang benar pemakaian tanda digunakan dalam karya ilmiah karena
bacanya adalah … penulisan tanda bacanya tidak mengikuti
(A) Mereka bekerja terus, walaupun EYD kecuali …
malam telah larut. (A) Penggunaan obat tradisional termasuk
(B) Lalu lintas padat sekali, karena itu jamu, yang makin marak dewasa ini
kami datang terlambat. adalah kenyataan yang patut
(C) Kami membutuhkan tangga, gergaji, disyukuri.
dan palu. (B) Kendati pemakaian jamu cukup marak
(D) Anak-anak, pemuda dan orang tua dan industri jamu juga berkembang,
hadir dalam upacara itu. sungguh mengherankan bahwa
(E) Rupanya dia tidak tahu bahwa besok ternyata produksi tanaman obat
libur. beberapa tahun belakangan stagnan.
(C) Dalam catatan Kepala Badan POM
8. Untuk membentuk kalimat yang baku, Indonesia memiliki 30.000 jenis
tanda baca (,) perlu digunakan kecuali tumbuhan.
pada kalimat … (D) Penurunan angka produksi menjadi
(A) Komputer dan barang elektronik ironi, kalau kita bandingkan dengan
lainnya, mengandung bagian tak pasar yang sesungguhnya
berbahaya dalam penggunaan sehari- membentang luas.
hari tetapi jadi beracun jika tak (E) Yang cukup menggembirakan, adalah
dibuang dengan tepat. kenyataan bahwa beberapa rumah
(B) Pabrik-pabrik yang akan dibangun sakit sudah memasukkan obat
harus memiliki prasarana untuk tradisional dalam terapi kepada
pengumpulan e-waste, tetapi ada pasien.
pabrik yang tidak mengindahkan
syarat tertentu.
(C) Beberapa negara, termasuk Cina, telah
melarang impor sampah elektronik.
11. Pemakaian tanda baca di bawah ini benar, 14. Penerapan tanda baca kalimat ini benar,
kecuali … kecuali …
(A) Karena sering tidak masuk bekerja, (A) Karena krisis ekonomi, banyak buruh
pegawai itu mendapat peringatan pabrik yang di-PHK.
keras dari atasannya. (B) LKIR diikuti oleh siswa SMU se-Jawa
(B) Ibu pergi ke toko membeli barang- Timur.
barang kebutuhan dapur termos, gelas, (C) Berhati-hatilah terhadap sinar-X.
dan kompor. (D) Kabinet yang sudah mantap sudah
(C) Dalam tulisan tangan atau tikan, huruf tentu tidak akan di-reshuffle lagi.
atau kata yang akan dicetak miring (E) Upahnya hanya Rp.15.000,00/hari.
diberi satu garis di bawahnya.
(D) Ia menyukai musik, teater, dan jenis- 15. Penggunaan tanda penghubung (-) yang
jenis kesenian lainnya. tepat terdapat dalam kalimat …
(E) Laki-laki itu pintar, tetapi sombong. (1) Ani menduduki peringkat ke-12.
(2) Bulan ini akan diadakan musyawarah
12. Kalimat berikut tidak menggunakan tanda gubernur se-Indonesia.
baca yang tepat, kecuali … (3) Ia harus segera memperpanjang KTP-
(A) Telah dikemukakan bahwa, bahan nya.
bakar merupakan energi yang sangat (4) Para siswa berkejar-kejaran di
penting dalam kehidupan masyarakat. halaman.
(B) Meskipun belum sempurna, uraian
yang disampaikannya sangat menarik. 16. Pemakaian tanda baca yang benar terdapat
(C) Investasi yang kita berikan ini, tidak dalam kalimat-kalimat di bawah ini,
seluruhnya berupa uang. kecuali …
(D) Mutu pelayanan harus selalu (A) Dari segi nalar, nalar ilmu atau logika,
ditingkatkan, supaya tidak cara pengobatan es telapak kaki sulit
mengecewakan pelanggan. dimengerti.
(E) Kerusakan lingkungan yang parah di (B) Camat Kota Subang Rumanda, S.H.,
Dataran Tinggi Dieng, boleh jadi akan M.Si. meminta Bupati Subang segera
menimpa kawasan lereng Gunung mengganti Lurah Sukamelang Dayat.
Ungkaran. (C) Lia, warga Kampung Adiarsa Pusaka,
RT 01/RW 08, Kelurahan Adiarsa,
13. Penulisan kalimat yang seluruhnya sesuai Kecamatan Karawang Timur, tewas
dengan aturan EYD adalah … tertabrak mobil.
(A) Kata feedback ‘balikan’ sudah sering (D) Dalam hal tertentu, antara lain oleh
digunakan sebagai istilah. rasa takut, resah, cemas, dan atau
(B) Bang Kadir sering disebut sebagai demam, suhu tubuh manusia dapat
‘pahlawan’, ia sendiri tidak tahu berubah.
sebabnya. (E) Risiko budi daya udang masih tinggi
(C) Karena postur tubuhnya Galih karena udang sering terkena virus
mendapat julukan “Si Gendut”. penyakit whitespot “bintik putih”.
(D) Kalau tidak diizinkan saya tidak akan
pergi.
(E) Esai Ratna yang berjudul Kesehatan
Reproduksi dimuat di “Tempo”.
17. Pemakaian tanda baca yang betul terdapat 21. Bung Komar sering disebut pahlawan, ia
pada kalimat … sendiri tidak tahu sebabnya.
(A) Celana “jengki” sudah tidak popular Kata pahlawan pada kalimat di atas
lagi di Indonesia. seharusnya …
(B) Rate of inflation, laju inflasi, di (A) Dimulai dengan huruf kapital
Indonesia memang agak tinggi pada (B) Diapit dua tanda petik
bulan Oktober 2000. (C) Dalam tanda kurung
(C) Surat yang dikirimkan itu No: (D) Ditulis dengan huruf miring
124/PP/Pes/I/2001. (E) Digarisbawahi
(D) Ali bertanya: “Di mana kaubeli buku?”
(E) Ia bertempat tinggal di Jalan 22. Penggunaan tanda baca yang salah
Diponegoro IV-5. terdapat dalam kalimat …
(1) Membaca buku lebih baik di tempat
18. Pemakaian tanda baca yang benar terdapat yang terang.
pada kalimat … (2) Dia tidak dapat membaca buku.
(A) Sahlan, mengucapkan terima kasih Karena lampu padam.
atas bantuan Putut. (3) Supaya mata tetap sehat, kami
(B) Semua peserta, yang tidak membawa membaca buku di tempat yang terang.
kartu peserta, harus melaporkan diri (4) Bacalah buku ini di tempat yang
kepada panitia. terang.
(C) Ratulangi tinggal di Jalan Mawar/5,
Pati, Jawa Tengah. 23. Semua penggunaan tanda baca berikut
(D) Ia harus pulang sekarang, karena sakit. adalah betul, kecuali …
(E) Sahabat saya Amin tinggal di (A) Kecepatan kendaraan yang disarankan
Manokwari. adalah antara 50-60 km/jam.
(B) Cita-cita Arifin−warga Desa
19. Penggunaan tanda baca yang salah Masbagik, Lombok Timur−untuk
terdapat dalam kalimat … mengadu nasib sebagai TKI di
(A) Membaca buku hendaklah di tempat Malaysia kandas sudah.
yang terang. (C) Meskipun sudah diperingatkan
(B) Dia tidak dapat membaca buku, karena berkali-kali, mereka berangkat juga.
lampu padam. (D) “Perjalanan kapal feri tidak bisa
(C) Supaya mata tetap sehat, kita harus menentang arus”, ujar seorang petugas
membaca buku di tempat yang terang. di Bakauheni.
(D) Bacalah buku ini di tempat yang (E) “Siapa yang harus bertanggung jawab
terang! atas kematiannya?” tanya salah
(E) Rina, guru adik, sakit. seorang wartawan.

20. Penggunaan tanda baca dengan benar


terdapat pada kalimat …
(A) Mereka tidak setuju, karena gagasan
itu tidak masuk akal.
(B) Kami memerlukan kertas, mistar, dan
lain-lain.
(C) Di toko itu terdapat: pakaian anak-
anak, alat olahraga, dan perabot dapur.
(D) Bapaknya orang Bali; sedangkan
ibunya orang Minang.
(E) Hindun, sahabat Aminah menjadi
juara kelas di bidang seni tari.
24. Pemakaian tanda baca yang betul terdapat (E) Menambahkan tanda koma (,) setelah
pada kalimat-kalimat di bawah ini, kecuali kata lain (kalimat 2)

(A) Perjalanan mereka memerlukan waktu 27. Penggunaan tanda baca dalam kalimat
2.25.30 jam berikut ini sesuai dengan aturan EYD
(B) Surat biasa, surat kilat, ataupun surat kecuali …
khusus memerlukan prangko. (A) Di jalur Losari−Cirebon belum terlihat
(C) Pasangan baru yang menempati adanya perbaikan, meskipun
rumah mewah itu memerlukan kerusakannya tergolong parah.
perabot rumah tangga, seperti kursi, (B) Banyak daerah nyaris tidak berdaya,
meja, dan almari. terutama dalam mengelola keuangan
(D) Ayah menyelesaikan pekerjaannya di daerah.
kamar meja; sedangkan saya dan adik (C) Ketika acara seminar berlangsung,
mengerjakan tugas sekolah di ruang Anda bertugas sebagai notulis.
tamu. (D) Kita sekarang memerlukan peralatan
(E) Persamaan kedua proses ini sekolah: sepatu, tas, dan alat tulis
(perbedaannya dibicarakan di dalam (E) Hak asasi manusia adalah hak yang
Bab II) perlu diungkapkan di bawah dimiliki oleh setiap manusia sejak lahir
ini. yang tidak boleh dilanggar, dibatasi,
ataupun dikurangi oleh siapa pun.
25. Kalimat yang menggunakan ejaan yang
benar adalah … 28. Pemakaian tanda baca yang tidak tepat
(A) Studi ilmu sosial, di luar negeri; tidak terdapat dalam kalimat …
banyak bedanya dengan ekspor kayu (A) Jika terjadi pelanggaran kode etik,
dari Indonesia. advokat yang terbukti melakukan
(B) Dicari segera; Kepala Pabrik, Kepala pelanggaran akan mendapat sanksi
Gudang, dan Kepala Administrasi. dari organisasi profesi advokat.
(C) Artikel tersebut terdapat dalam (B) Apakah semua organisasi advokat:
majalah GAUNG No. 123 tahun 1992. Peradi, KAI, dan Peradin bersedia
Hlm. 12-34. menjalankan kode etik?
(D) Setelah memukul gong, Kepala Negara (C) Wadah tunggal organisasi profesi
lalu bertepuk tangan. advokat dan memudahkan sosialisasi,
(E) Mata pelajaran Bahasa Indonesia monitoring, dan evaluasi Kode Etik
dianggap sama sulitnya dengan mata Advokat Indonesia yang notabene
pelajaran Matematika dan Fisika. dirumuskan PERADI dalam
pelaksanaannya.
26. (1) Pada setiap adanya pembaruan perihal (D) Secara empiris, memang ada kode etik
kurikulum, pihak dinas pendidikan profesi yang dijalankan oleh beberapa
dituntut untuk secara cepat melakukan organisasi profesi, misalnya Kode Etik
penyesuaian terhadap para pelaksana di Guru Indonesia.
lapangan. (2) Akan tetapi, kenyataan (E) Menjaga profesionalisme advokat
berbicara lain yakni umumnya merupakan komitmen pemerintah;
penanganan pengembangan tenaga dan warga negara wajib ikut
pelaksana di lapangan berjalan sangat mendorong penegakan hukum di
lambat. Indonesia.
Ejaan pada kalimat (1) dan kalimat (2) di
atas menjadi benar jika diperbaiki dengan
cara …
(A) Menghilangkan tanda koma (,) setelah
kata kurikulum (kalimat 1)
(B) Menulis kata dinas pendidikan
(kalimat 1) dengan huruf kapital
(C) Mengubah kata perihal (kalimat 1)
menjadi hal
(D) Menghilangkan tanda koma (,) setelah
kata akan tetapi (kalimat 2)
29. Penggunaan tanda baca koma (,) yang 30. Pemakaian tanda koma yang tepat
tepat terdapat dalam kalimat … terdapat pada kalimat …
(A) Beberapa mahasiswa yang lolos tes (A) Pertumbuhan apartemen yang
dan memenuhi persyaratan, akan menjamur, diikuti dengan
mendapat beasiswa dari universitas permukiman padat di bawah garis
terkenal di Tokyo. ambang luap air (gala) yang kumuh,
(B) Para mahasiswa yang akan mengikuti merupakan gejala pertumbuhan kota
tes kemampuan bahasa Jepang secara yang tak terencana.
gratis, diharapkan mendaftarkan diri (B) Jika Indonesia baru memiliki 15 hasil
di ruang kantor. kebudayaan yang ditetapkan menjadi
(C) Pada ahli matematika merasa kagum, warisan budaya, Jepang dan Korea
ketika mengetahui perhitungan lebah Selatan, sudah memiliki masing-
yang sangat cermat. masing 43 dan 37 buah jenis
(D) Seorang pedagang di pasar kebudayaan.
mengatakan, sambil melayani (C) Menurut pakar diet klinis, Emilia E.
pelanggannya, bahwa kenaikan harga Achmadi, M.S., jika teh pemanis
cabai dan bawang merah sudah buatan diminum satu kali saja, efek
berlangsung selama tiga tahun. negatifnya, tidak akan signifikan.
(E) Para penyelenggara negara yang (D) “Kita harus bisa membuktikan bahwa
dalam batas waktu tertentu tidak tenun memang milik masyarakat, dan
melaporkan harta kekayaannya ke memiliki keterkaitan dengan
KPK, akan dikenakan sanksi yang kehidupan mereka”, jelas
berupa denda atau hukuman. Wamendikbud Bidang Kebudayaan.
(E) Oleh karena itu, data-data yang
dibawa seperti sejarah, hasil penelitian,
dan pemetaan, menjadi unsur yang
kuat, agar tenun NTT terpilih sebagai
warisan budaya dunia
MORFOLOGI

Morfologi atau merupakan ilmu yang berkaitan dengan proses pembentukan kata. Proses morfologi di
antaranya: (1) afiksasi, (2) komposisi, (3) reduplikasi, dan (4) penyerapan kata. Afiksasi membentuk
kata dengan cara memberikan imbuhan/afiks. Komposisi membentuk kata dengan cara
menggabungkan dengan kata lain sehingga akan terbentuk kata majemuk. Reduplikasi membentuk
kata dengan cara pengulangan. Penyerapan kata membentuk kata dengan cara menyerap bahasa lain
ke dalam bahasa Indonesia.

Afiksasi adalah proses pembentukan kata dari morfem menjadi kata bentukan yang lebih kompleks.
Morfem adalah satuan gramatikal terkecil. Morfem dapat berupa kata maupun imbuhan. Morfem yang
berupa kata disebut dengan morfem bebas. Morfem yang berupa imbuhan disebut dengan morfem
terikat.

Morfem Bebas Morfem Terikat Jumlah Morfem


bersepeda sepeda ber- 2
pertanggungjawaban tanggung jawab per-, -an 4

A. Awalan (Prefiks)
Awalan atau prefiks adalah imbuhan yang berada di depan kata.
Contoh:
me(N)- lawan > melawan, bawa > membawa, hapus > menghapus,
cukur > mencukur, sapu > menyapu, pel > mengepel
pe(N)- lamar > pelamar, bobol > pembobol, kasih > pengasih,
tanam > penanam, sambut > penyambut, cek > pengecek
ber- tamu > bertamu, renang > berenang, ajar > belajar
per- alat > peralat, ramal > peramal
ter- baik > terbaik, ramai > teramai
di- makan > dimakan
se- ikat > seikat
ke- luar > keluar

B. Sisipan (Infiks)
Sisipan atau infiks adalah imbuhan yang berada di tengah kata.
Contoh:
-ah- basa > bahasa
-el- gembung > gelembung
-em- getar > gemetar
-er- gigi > gerigi
-in- kerja > kinerja

C. Akhiran (Sufiks)
Akhiran atau sufiks adalah imbuhan yang berada di belakang kata.
Contoh:
-an petik > petikan -i insan > insani
-kan kerja > kerjakan -is jurnal > jurnalis
-i tanam > tanami -asi afiks > afiksasi
-nya niat > niatnya -isasi sosial > sosialisasi
-kah sudah > sudahkah -iah alam > alamiah
-lah ini > inilah -al struktur > struktural
-man seni > seniman -at muslim > muslimat
-wan rupa > rupawan -ah muslim > muslimah
-wati peragawati -in hadir > hadirin
-wi dunia > duniawi -ik magnet > magnetik
D. Gabungan Awalan dan Akhiran (Konfiks)
ke-an baik > kebaikan
pe(N)-an lapor > pelaporan, aman > pengamanan, dingin > pendinginan,
bersih > pembersihan, sebar > penyebaran, cor > pengecoran
per-an adab > peradaban, rasa > perasaan
ber-an gugur > berguguran
ber-kan mandi > bermandikan

E. Gabungan Imbuhan (Simulfiks)


me(N)-kan rugi > merugi > merugikan
me(N)-i tanam > menanam > menanami
memper-kan tahan > pertahankan > mempertahankan
memper-i baik > perbaiki > memperbaiki
member-i sama > bersama > membersamai
ber-kan nama > bernama > bernamakan
di-kan antar > diantar > diantarkan
keber-an pihak > berpihak > keberpihakan

Istilah-istilah dalam morfologi


• Afiksasi : pengimbuhan kata
• Alomorf : variasi perubahan bunyi bahasa yang terjadi pada afiksasi
• Morfem : satuan gramatikal terkecil
• Morfem bebas : morfem yang dapat berdiri sendiri (berupa kata)
• Morfem terikat : morfem yang terikat dengan morfem lain (berupa imbuhan)
• Morfologi : ilmu yang mempelajari tentang pembentukan kata
• Proses morfologi : proses pembentukan kata
1. Proses morfologis bentuk kata 6. Kata yang bercetak miring dalam kalimat
keberuntungan yang benar adalah …. berikut tidak baku, kecuali …
(A) untung > beruntung > keberuntungan (A) Indonesia sanggup mensejajarkan diri
(B) untung > keuntungan >
dengan negara lain.
keberuntungan
(C) untung > untungan > keuntungan > (B) Kita harus mensukseskan pemilu yang
keberuntungan akan datang.
(D) untung > keuntungan > beruntung > (C) Pemerintah berusaha menyetabilkan
keberuntungan harga semen.
(E) untung > beruntung > keuntungan > (D) Kamu harus menyesuaikan diri
keberuntungan dengan lingkungan.
(E) Pak lurah memrakarsai pembangunan
2. Kata keterbatasan terdiri atas ….
(A) 1 morfem bebas, 1 terikat di desa ini.
(B) 1 morfem bebas, 2 terikat
(C) 1 morfem bebas, 4 terikat 7. Konfiks pe-an yang berarti ‘alat melakukan
(D) 2 morfem bebas, 2 terikat perbuatan’ terdapat pada kalimat …
(E) 1 morfem bebas, 3 terikat (A) Penyatuan kedua wilayah itu bukan
masalah yang mudah.
3. Dalam kelompok kata sepuluh halaman
(B) Penciuman anjing sangat tajam.
folio terdapat ….
(A) 3 morfem bebas, 1 terikat (C) Angka pengangguran semakin hari
(B) 3 morfem bebas, 2 terikat semakin melonjak.
(C) 3 morfem bebas, 4 terikat (D) Pengumpulan dana itu diinisiasi oleh
(D) 3 morfem bebas, 3 terikat Pak RT.
(E) 3 morfem bebas tanpa morfem terikat (E) Penghentiannya sementara pegawai di
perusahaan itu terjadi sudah cukup
4. Pembentukan kata-kata berikut sesuai
dengan kaidah bahasa Indonesia baku lama.
kecuali ….
(A) mengkliring, mencap, mengkop 8. Imbuhan me-kan yang menyatakan makna
(B) mengklakson, mengeset, mengicau melakukan sesuatu untuk orang lain
(C) mengeklik, mengelem, mengelas terdapat pada kalimat …
(D) mengelap, mengkhitan, mengilap (A) Beliau menugaskan pekerjaan itu
(E) mengklarifikasi, mengklise,
kepada saya.
mengontrol
(B) Kepala sekolah telah memberikan
5. Dia terpaksa menyederhanakan kesempatan kepadamu.
masakannya pada akhir bulan. (C) Jono menerbangkan pesawat itu
sampai ketinggian 30.000 kaki.
Pola pembentukan kata menyederhanakan (D) Kamu harus segera membuatkan Pak
sama dengan pola pembentukan kata …
Budi laporan itu.
(E) Rini menghempaskan badannya ke
(A) memperjuangkan
(B) melakukan atas tempat tidur.
(C) mengesampingkan
(D) menyediakan
(E) membacakan
9. Penetapan Angka Kredit (PAK) tahap 1 11. Sufiks –i yang berfungsi mengubah kata
tahun 2009 yang merupakan hasil dari kerja intransitif menjadi kata kerja transitif
penilaian Daftar Usul Penetapan Angka terdapat pada kalimat …
Kredit (DUPAK) yang diajukan oleh 25 (A) Tentara itu menembaki musuh yang
pustakawan dan 4 calon pustakawan ada.
sudah diterbitkan. (B) Aji menangisi kepergian kekasihnya.
(C) Rosi memukuli kepalanya dengan
Imbuhan pe-an pada kalimat berikut keras.
semakna dengan imbuhan pe-an pada kata (D) Eko memandangi wanita cantik itu.
bercetak miring dalam kalimat tersebut, (E) Arie mencabuti bulu kakinya.
kecuali …
(A) Pembukaan peringatan Hari Krida 12. Pasangan kata me-kan dan me-i yang tepat
Pertanian (HKP) yang ke-37 tahun 2009 terdapat dalam kalimat …
menurut rencana akan dilaksanakan (A) Hendarto memindahkan kursi itu
secara serentak di seluruh Indonesia kemudian mendudukinya.
pada hari Senin, tanggal 22 Juni 2009. (B) Ruli selalu menurutkan nasihat itu
(B) Penghargaan diberikan kepada para sehingga menyenangi hati ibunya.
mahasiswa berprestasi. (C) Joko menyeberangkan jalan raya
(C) Penyusunan KTSP (Kurikulum setelah menjalani tugas.
Tingkat Satuan Pendidikan) (D) Husni menghindarkan lubang di jalan
memerlukan waktu yang tidak sedikit. untuk menyelamati kendaraannya.
(D) Berdasarkan Kurikulum 2006, (E) Mereka beranggapan bahwa Ani
penilaian kognitif, psikomotorik, dan meringankan badannya dengan selalu
afektif menjadi satu nilai dalam rapor. meminum jamu setiap hari.
(E) Sekarang sudah tersedia penyaringan
untuk menjernihkan air sehingga kita 13. Fungsi ber- pada kalimat Ia bernafsu
tidak perlu khawatir lagi. hendak memukul saya sama dengan fungsi
ber- pada kalimat …
10. Dalam perjalanan itu kami mendatangi (A) Aku berpikir maka aku ada.
beberapa tempat wisata. (B) Bertanam padi merupakan pekerjaan
Kata perjalanan pada kalimat di atas ayahku.
mempunyai makna yang sama dengan (C) Indonesia berambisi memenangi
kata pada kalimat berikut, kecuali … pertandingan itu.
(A) Perbaikan rumah itu memakan waktu (D) Setiap hari ia bersepeda ke kantor.
tiga bulan. (E) Bel tanda masuk sudah berbunyi.
(B) Beberapa orang warga mengikuti
pertandingan bulu tangkis.
(C) Para pendaki gunung itu membawa
perbekalan yang cukup.
(D) Persahabatan mereka sudah berjalan
selama sepuluh tahun.
(E) Pertemuan kedua sahabat itu sangat
mengharukan.
14. Imbuhan ke-an pada kalimat (E) Tini pergi ke dokter gigi untuk
Matanya kemasukan debu mencabutkan giginya yang sakit.
mempunyai makna yang sama dengan
imbuhan pada kalimat-kalimat berikut, 18. Ulama itu mempertemukan Hasan dan
kecuali … Azizah.
(A) Kemarin, kakak saya kecurian dompet
yang berisi sejumlah uang Proses pembentukan kata
(B) Imbanglah kedudukan ketika Warsidi mempertemukan dalam kalimat di atas
memasukkan bola ke gawang lawan sejalan dengan proses pembentukan kata
(C) Beberapa orang keracunan makanan berimbuhan memper-kan dalam kalimat …
dari pesta pernikahan itu (A) Patih Gajah Mada berupaya
(D) Anto mempunyai masalah kelebihan mempersatukan kerajaan-kerajaan
berat badan Nusantara.
(E) Karena kejatuhan kelapa, kepalanya (B) Atasan Anda tidak mempersalahkan
benjol Anda atas peristiwa yang terjadi
kemarin.
15. Kata berawalan ter- yang berjenis kata (C) Mengapa Andi selalu
benda terdapat pada kalimat … mempertanyakan nilai-nilai
(A) Ternyata dia tidak menghendaki kuliahnya?
kehadiran kami. (D) Saya mohon Anda tidak
(B) Tersenyumlah Jono dengan penuh mempersamakan saya dengan dia.
kebanggaan. (E) Wartawan memperjuangkan orang
(C) Apa yang terjadi maka terjadilah. yang tertindas dengan kata-kata.
(D) Terpidana mati itu tidak pernah
menyesal atas perbuatannya. 19. Kata berimbuhan dipakai secara tepat
(E) Terlambat sudah ia menyadari dalam kalimat …
kesalahannya selama ini. (A) Banyak bangunan di sepanjang jalan
baru dikontrakan untuk usaha.
16. Prefiks ber- dalam berhias mempunyai arti (B) Pada acara ajang pencarian bakat di
refleksif. Yang tidak semakna dengan itu sebuah televisi, peserta dapat
terdapat dalam bentuk …. mempertunjukkan bakat yang unik
(A) bersolek dan menarik.
(B) bercermin (C) Setiap laporan bacaan yang
(C) bercukur dikumpulkan harus diketik dengan
(D) bertinju spasi 2.
(E) berpakaian (D) Ketika kebaran terjadi, para penguin
apartemen diharapkan memerhatikan
17. Kata berimbuhan digunakan secara tepat jalur evakuasi yang telah disiapkan
dalam kalimat … oleh pengelola.
(A) Mandor di pabrik itu membawahi (E) Pemerintah melarang nelayan mencari
sepuluh kuli yang rajin. ikan dengan cara membom.
(B) Kepala sekolah menugasi para guru
untuk menghadiri rapat.
(C) Anita telah mengirimi surat kepada
bapaknya.
(D) Tujuan kegiatan ini adalah
mengolahragakan para karyawan.
20. Aku bagaikan kejatuhan bulan. 23. Pemuda itu memperistri kemenakan
kenalannya.
Kata kejatuhan dalam kalimat tersebut
berstruktur sama dengan kata berimbuhan Arti imbuhan memper- pada kata
ke-an dalam kalimat berikut, kecuali … memperistri sama dengan arti memper-
(A) Pak Dahlan kebakaran jenggot pada kata …
mendengar namanya disebut-sebut. (A) Diduga orang itu memperalat
(B) Tetangga saya tidak mengira temannya.
kedatangan tamu tak diundang tadi (B) Oknum polisi itu sering mempersulit
malam. warga.
(C) Mata adikku kemasukan debu (C) Galian kabel itu mempersempit jalan.
sehingga memerah. (D) Pemda akan mempertinggi tanggul
(D) Tiada yang risau akan kematian penahan banjir.
koruptor yang menyengsarakan (E) Jono memperoleh sepeda baru dari
negara. Presiden Jokowi.
(E) Baru saja ia melepas kepergian
suaminya. 24. Pemakaian bentuk –nya pada kalimat-
kalimat berikut tepat, kecuali …
21. Imbuhan ber-an pada kata Mereka (A) Budi sudah pergi ke sekolah sehingga
berhadapan di atas ring mempunyai sepedanya sudah tidak ada di rumah.
makna yang sama dengan imbuhan ber-an (B) Siswa itu tertabrak mobil. Hidung dan
di dalam kalimat berikut, kecuali … telinganya mengeluarkan darah.
(A) Rumah Adit berdekatan dengan (C) Semestinya para pejabat memberikan
rumah Sari. contoh yang baik kepada rakyatnya.
(B) Joni dan Jono akhirnya bersalaman di (D) Yang diparkir di halaman kampus itu
acara itu. adalah mobilnya dosen fakultas teknik.
(C) Kami berkenalan sepuluh tahun yang (E) Pendapatannya tidak cukup untuk
lalu. kebutuhan hidup selama sebulan.
(D) Kedua pasangan itu bertatapan muka
dengan penuh rasa cinta. 25. Kata dengan imbuhan me- yang tidak
(E) Ayah saya berpandangan sebaliknya. menyatakan kerja terdapat pada kalimat …
(A) Dia tidak mengakui perbuatannya
22. Makna afiks pada kata terbawa pada selama ini.
kalimat Maaf, bukumu kemarin terbawa (B) Penduduk desa itu banyak yang
oleh saya, sama dengan … merotan.
(A) Namanya tercantum dalam daftar (C) Para penerjun payung telah mendarat
siswa yang lulus. dengan selamat.
(B) Anjing itu berjalan terseok-seok setelah (D) Gentur mengontrak rumah di daerah
ditabrak motor. Kalisari.
(C) Sepedanya tersembunyi di ruang (E) Kami datang menjelang pesta dimulai.
bawah tanah.
(D) Ani menangis terisak-isak.
(E) Dia tertidur di ruang tamu ketika
menunggu ibunya.
26. Wisata bawah laut di perairan Wakatobi, 29. Reduplikasi yang bermakna berbalasan
Sulawesi Tenggara, akan menjadi terdapat dalam kalimat berikut …
keunggulan kelautan yang terintegrasi. (1) Mereka melompat-lompat kegirangan.
(2) Hobi anak gadis itu jahit-menjahit.
Kata keunggulan pada kalimat tersebut (3) Bekerjalah, jangan duduk-duduk saja.
sama dengan bentuk kata yang dicetak (4) Pertemuan itu diakhiri dengan tukar-
miring pada kalimat … menukar bingkisan.
(1) Dia berharap ekspor ini akan
memberikan keuntungan nyata.
30. Bentuk ulang yang menyatakan arti
(2) Hal itu diharapkan dapat
menyerupai terdapat pada …
meningkatkan kesejahteraan rakyat.
(1) sayur-mayur
(3) Wakatobi diharapkan dapat menjadi
(2) berkelip-kelip
kekayaan dunia.
(3) cubit-cubitan
(4) Pertemuan itu dihadiri tokoh
(4) kucing-kucingan
keuangan dan finansial.

27. Pada malam Minggu diselenggarakan 31. Fungsi pengulangan bermarah-marahan


pertunjukan musik di panggung utama. pada kalimat Kedua gadis itu sedang
bermarah-marahan, juga terdapat pada
Kata pertunjukan mengalami pola kalimat …
pembentukan yang sama dengan kata (1) Kedua musuh itu sedang mencari jalan
berimbuhan per-an dalam kalimat berikut, untuk berbaik-baikan.
kecuali … (2) Anak di bawah umur belum boleh
bercinta-cintaan.
(A) Peragaan busana kali ini menampilkan (3) Kedua anak itu sedang berkejar-
busana pengantin kejaran.
(B) Saya selalu berupaya memberikan (4) Pemuda-pemuda itu sering terlihat
perhatian kepada orang tua bermabuk-mabukan di lepau tuak.
(C) Ani mengagumi perlakuan Ria kepada
sahabatnya yang cacat 32. Makna pengulangan pada kata tergila-gila
(D) Persamaan hak kaum perempuan dalam kalimat Banyak orang tergila-gila
dengan pria terus diperjuangkan
pada pertandingan sepak bola, sama
(E) Perbuatan Rio terhadap temannya sulit
dengan makna proses pengulangan pada
dimaafkan.
kalimat-kalimat berikut, kecuali …
(1) Ia membetul-betulkan jawaban yang
28. Makna men-kan pada kalimat “Pemusik
salah.
Jazz, Benny Likumahuwa,
(2) Peserta ujian dilarang berpindah-
mempromosikan Ambon sebagai kota
pindah tempat duduk.
yang aman dan memiliki pesona” sama
dengan makna men-kan pada … (3) Kami saling bersurat-suratan.
(A) Penerbit memublikasikan buku baru (4) Jangan bawa-bawa kami dalam
Dewi Lestari. masalah ini.
(B) Dengan terpaksa, dia membukakan
pintu untuk pengamen itu.
(C) Nilai rupiah saat ini sangat
menggembirakan.
(D) Yuanita memilih mematikan lampu.
(E) Mereka menegaskan bahwa cara
tersebut akan dihentikan.
33. Makna reduplikasi pada kata mobil- (2) Kalian boleh makan sekenyang-
mobilan dalam kalimat Ketika kecil, kami kenyangnya di rumahku.
sering membuat mobil-mobilan dari kulit (3) Sehari-harinya dia membantu keluarga
jeruk bali, terdapat pula pada kalimat … kami di rumah.
(A) Mereka memiliki kebun buah-buahan (4) Penyelam itu berusaha mencapai
yang sangat luas. lubuk sungai sedalam-dalamnya.
(B) Suara tembak-tembakan terus
terdengar selama sepuluh menit. 37. Bangunan di perumahan baru itu kecil-
(C) Cincin ini kuberikan sebagai kenang- kecil.
kenangan untukmu.
(D) Kami terkejut melihat orang-orangan Arti pengulangan pada kecil-kecil sama
untukmu. dengan arti pengulangan pada kata
(E) Kalian harus menguasai tari-tarian bercetak miring di bawah ini, kecuali …
dengan sempurna. (A) Dia mengayuh perahunya cepat-cepat.
(B) Barang dagangannya murah-murah.
34. Anak-anak biasanya menyukai mobil- (C) Akibat banjir itu penduduk di sana
mobilan. sakit-sakit.
(D) Tanaman di kebunnya subur-subur.
Kata ulang pada kalimat di atas sama (E) Tentu saja cantik-cantik karena mereka
maknanya dengan kata ulang di bawah ini artis.

(1) Sering terjadi kucing-kucingan antara 38. Perulangan yang mempunyai arti saling
pedagang kaki lima dan satgas tibum. terdapat pada kalimat …
(2) Warna kehijau-hijauan biasanya (A) Kecil-kecil dia sudah dapat membantu
menyejukkan mata. orang tuanya.
(3) Kelakuannya itu kebarat-baratan. (B) Rumah-rumah di daerah kami sedang
(4) Langit-langit rumah itu telah rusak dipugar.
berat. (C) Siapa-siapa yang datang tentu
disapanya.
35. Makna reduplikasi pada kata tumbuh- (D) Ibu menggali lubang untuk tanaman
tumbuhan dalam kalimat Kebun Raya itu dalam-dalam.
Bogor dipenuhi dengan tumbuh- (E) Mereka berpandang-pandangan sejak
tumbuhan tropis terdapat pada kalimat … tadi.
(A) Anak yang akan disunat dinaikkan ke
atas singa-singaan. 39. Kata ulang yang mengandung arti
(B) Mobil-mobilan itu digerakkan dengan menyangatkan terdapat pada kalimat …
tenaga surya. (A) Ayah bekerja segiat-giatnya untuk
(C) Buku ini kuberikan kepadamu sebagai mencukupi kebutuhan keluarga.
kenang-kenangan. (B) Ibu Hasan mondar-mandir mencari
(D) Kami terkejut melihat orang-orangan anaknya yang terlambat pulang dari
di depan pintu rumah mereka. sekolah.
(E) Kalian harus menguasai tari-tarian (C) Mereka menggeleng-gelengkan
dengan sempurna. kepalanya karena jengkel sekali.
(D) Ia berpukul-pukulan dengan si Dul.
36. Pengedar obat terlarang harus dihukum (E) Para undangan bersalam-salaman
seberat-beratnya. kemudian pulang.

Fungsi perulangan pada kata seberat-


beratnya terdapat juga pada kata bercetak
miring dalam kalimat …
(1) Sebentar-sebentar orang itu menengok
ke samping kanan.
40. Pemakaian bentuk dalam kalimat berikut 44. Kata ulang yang digunakan secara tepat
salah, kecuali dalam kalimat … terdapat dalam kalimat …
(A) Syamila selalu tolong-menolong dalam (A) Banyak guru-guru hadir dalam
setiap kesempatan. pertemuan itu.
(B) Serombongan penari-penari Bali telah (B) Para murid-murid berbaris dengan
datang sore ini. rapi.
(C) Syahida dan Syamila saling bantu- (C) Segenap anggota-anggota Korpri wajib
membantu menyelesaikan tugasnya. apel.
(D) Dalam menghadapi kesulitan, kita (D) Pemuda-pemuda sibuk menghadapi
wajib tolong-menolong. peringatan hari Proklamasi.
(E) Peristiwa itu kait-mengait antara yang (E) Semua mobil-mobil harus diparkir di
satu dengan lainnya. tempat yang telah disediakan.

41. Kalimat yang menggunakan kata ulang 45. Penggunaan kata ulang kata kerja yang
semu adalah … tepat pada kalimat di bawah ini adalah …
(A) Ia tidur di tikar-tikar pandan. (A) Tamu itu mengetuk-ngetuk pintu
(B) Kedua remaja itu berkejar-kejaran. beberapa kali.
(C) Ia berlari di taman mengejar kupu- (B) Ade berlari-lari kecil melihat ibunya
kupu. datang.
(D) Rumah-rumah itu hancur diterjang (C) Kedua kelompok massa itu
banjir. mengumpulkan batu-batu.
(E) Ia mondar-mandir di depan kantor (D) Persoalan-persoalan itu berkaitan satu
guru. dengan lainnya.
(E) Mereka sehari-hari saling berkirim-
42. Penggunaan kata ulang yang sudah tepat kiriman SMS.
terdapat pada kalimat …
(A) Serombongan pelukis-pelukis
mengadakan pameran di kota kami.
(B) Ibu sudah mengajar kami hal masak-
memasak sejak sekolah dasar.
(C) Dalam menghadapi kesulitan kita
wajib tolong-menolong.
(D) Pikirkan hal itu sangat masak-masak
supaya tidak menyesal akhirnya.
(E) Adik selalu menyebut-nyebut nama
ibu sewaktu ia sakit.

43. Perulangan yang menyatakan makna


berulang kali adalah …
(A) Pikirkanlah hal itu baik-baik.
(B) Mereka berjemur-jemur di pantai.
(C) Dari tadi ia memanggil-manggil nama
adiknya.
(D) Selesai berdiskusi mereka berjalan-
jalan.
(E) Tertawa-tawa saja anak itu dari tadi.
SEMANTIK

Semantik atau tata makna merupakan ilmu yang berhubungan dengan makna kata. Beberapa cara
untuk memahami makna dapat dilakukan berdasarkan (1) jenis-jenis makna, (2) perubahan makna,
dan (3) hubungan makna.

A. Jenis-Jenis Makna
Jenis-jenis makna antara lain (a) leksikal, (b) gramatikal, serta (c) denotasi dan konotasi.
(a) Makna leksikal
Makna leksikal merupakan makna kata secara bebas, makna ini dapat dilihat pada kamus.
rumah ‘bangunan untuk tempat tinggal’

(b) Makna Gramatikal


Makna gramatikal merupakan makna kata yang timbul karena proses ketatabahasaan seperti
pembentukan kata, frasa, atau kalimat.
perumahan ‘kumpulan beberapa buah rumah; rumah-rumah tempat tinggal’
rumah sakit ‘gedung tempat merawat orang sakit’

(c) Makna Denotasi dan Konotasi


Makna donotasi merupakan makna kata sebenarnya, sedangkan makna konotasi
merupakan makna kata bukan sebenarnya atau kiasan.
Denotasi Konotasi
bunga ‘bagian tumbuhan yang akan menjadi buah’ bunga ‘gadis’
meja hijau ‘meja yang berwarna hijau’ meja hijau ‘pengadilan’

B. Perubahan Makna
Perubahan makna mengacu pada perubahan makna kata tertentu berdasarkan berbagai
penyebab. Setidaknya terdapat enam perubahan makna kata dalam semantik, yaitu (a) peyorasi,
(b) ameliorasi, (c) perluasan, (d) penyempitan, (e) sintesia, dan (f) asosiasi.
(a) Peyorasi
Peyorasi adalah perubahan makna menjadi lebih rendah dari yang makna sebelumnya.
Contoh:
tunarungu ‘tidak dapat mendengar’
tuli ‘tidak dapat mendengar’ (lebih rendah daripada tunarungu)

(b) Ameliorasi
Ameliorasi adalah perubahan makna menjadi lebih tinggi, lebih hormat, atau lebih disukai.
Contoh:
jomlo ‘tidak memiliki pasangan’
tunasmara ‘tidak memiliki pasangan’ (lebih terhormat daripada jomlo)

(c) Perluasan
Perluasan adalah perubahan makna yang cakupan maknanya lebih luas.
Contoh:
kepala ‘bagian tubuh di atas leher’
kepala ‘bagian tubuh di atas leher, pemimpin, ketua’ (meluas)
(d) Penyempitan
Perluasan adalah perubahan makna yang cakupan maknanya lebih sempit.
Contoh:
pembantu ‘orang yang membantu’
pembantu ‘orang yang membantu mengurus rumah tangga’ (menyempit)

(e) Sinestesia
Sinestesia adalah perubahan makna karena adanya pertukaran tanggapan dua indera.
Contoh:
Kata-katanya manis. (tajam merupakan tanggapan indera pengecap)
Pandangannya dingin. (dingin merupakan tanggapan indera peraba)

(f) Asosiasi
Asosiasi adalah perubahan makna karena adanya persamaan sifat
(benda/rasa/warna/tempat/waktu/peristiwa)
Contoh:
Tekadnya untuk lanjut kuliah seperti bara api. (asosiasi benda)
Bagai mendapatkan durian runtuh. (asosiasi peristiwa)
Bagai melukis di permukaan air. (asosiasi tempat)

C. Hubungan Makna
Terdapat lima hubungan makna dalam semantik, yaitu (a) sinonim, (b) antonim, (c) hiponim, (d)
hipernim, dan (e) polisemi.
(a) Sinonim
Sinonim adalah hubungan makna yang memiliki persamaan/padanan.
Contoh:
sulit = sukar bertemu = berjumpa
bagus = baik senang = bahagia

(b) Antonim
Antonim adalah hubungan makna yang saling berlawanan.
Contoh:
sulit >< mudah sulit >< mudah
bagus >< jelek senang >< sedih

(c) Hipernim Contoh:


Hipernim adalah kata yang memiliki Hipernim Hiponim
makna lebih umum. hewan kucing
anjing
(d) Hiponim bunga mawar
Hiponim adalah kata yang memiliki melati
makna lebih khusus.

(e) Polisemi
Polisemi adalah kata/frasa yang memiliki lebih dari satu makna.
Contoh:
Kepala1 sekolah meminta kami menundukkan kepala2 saat mengheningkan cipta.
kepala1 ‘pemimpin, ketua’
kepala2 ‘bagian tubuh di atas leher’
1. Hati-hati membawa barang ini. Jangan 4. Dalam kalimat berikut ini terdapat kata
sampai jatuh, ya! yang mengalami perluasan makna, yaitu

Kata jatuh yang dipakai dalam arti seperti (A) Sejak dulu hingga sekarang profesi
pada kalimat di atas terdapat pada … guru identik dengan keprihatinan dan
(A) Nilainya jatuh gara-gara tidak kemelaratan.
memperhatikan petunjuk soal ujian. (B) Banyak tabib terkenal diundang untuk
(B) Nama Institusi kita tidak akan jatuh menyembuhkan penyakit aneh sang
hanya karena kasus ini. putri.
(C) Selamat bertanding pembalap itu tidak (C) Dewasa ini banyak bermunculan LSM
memedulikan berapa kali jatuh yang memperjuangkan nasib
bangun di medan yang becek. perempuan.
(D) Sejak lama dia jatuh hati dengan (D) Dalam lima tahun bengkel yang
kebudayaan Indonesia. mereka kelola telah menghasilkan
(E) Sudah lama ia jatuh sakit. pemasukan tidak kurang dari lima
puluh juta sebulan.
2. Penggunaan makna terukir yang (E) Peran yang tidak pernah absen dalam
bermakna leksikal terdapat dalam kalimat setiap pementasan Srimulat adalah
… jongos.
(A) Kenangan terindah semasa di SMA
terukir di hatinya. 5. Kata yang mengalami perubahan makna
(B) Masih terukir dengan sangat jelas di karena asosiasi terdapat dalam kalimat …
ingatannya peristiwa letusan Gunung (A) Persentase sarjana yang belum
Merapi itu. mendapatkan pekerjaan semakin
(C) Suasana hiruk-pikuk di tempat bertambah.
pengungsian terukir dengan baik pada (B) Para pegiat perempuan semakin berani
lukisannya. memperjuangkan hak-haknya.
(D) Masa remaja yang penuh cerita terukir (C) Karena kesantuannya, dia disebut
begitu mendalam di hatinya. sebagai pedeta.
(E) Jejak-jejak ombak serta alur air terlihat (D) Sesekali dia melontarkan kritikan
terukir di atas karang. pedas terhadap kebijakan atasannya
yang dianggap keliru.
3. Kata berlayar yang dahulu hanya dipakai (E) Waktu melaksanakan tugas
dalam kalimat seperti “Nelayan tradisional jurnalistiknya, dia selalu berusaha
berlayar mencari ikan setiap malam” kini menolak pemberian amplop dalam
juga dipakai dalam kalimat seperti setiap peristiwa yang diliputnya.
“Marilah berlayar bersama kapal pesiar
kami.” Kata itu mengalami ….
(A) penyempitan makna
(B) perluasan makna
(C) perubahan makna yang membaik
(D) perubahan makna yang memburuk
(E) perubahan makna menjadi makna
kiasan
6. Mendiknas mengaku, banjir yang melanda 10. Meskipun mutiara air tawar menyerbu
Jakarta dan sekitarnya memberi dampak pasar domestik dengan harga murah
yang cukup signifikan terhadap sektor seperti yang ditawarkan, Cina memukul
pendidikan. Makna signifikan dalam usaha yang ada.
wacana tersebut adalah ….
(A) hebat atau luar biasa Makna istilah domestik pada kalimat
(B) penting atau berarti tersebut adalah ….
(C) tepat dan sesuai (A) hal kebutuhan pokok rumah tangga
(D) pasti dan yakin (B) berhubungan dengan masalah dalam
(E) parah atau merugikan negeri
(C) bersifat menentukan kemauan dalam
7. Komersialisasi jabatan sudah membudaya negeri
di Indonesia. (D) berpengaruh kuat terhadap masalah
pribadi
Kata komersialisasi jabatan dalam kalimat (E) pasar tradisional yang ada di dalam
itu mengandung makna …. negeri
(A) menjualbelikan jabatan
(B) memperdagangkan jabatan 11. Kata eksekusi dalam Eksekusi
(C) menyalahgunakan jabaan pembongkaran rumah di bantaran sungai
(D) mencari keuntungan dengan itu mendapat perlawanan dari penduduk
menyalahgunakan jabatan setempat mempunyai makna ….
(E) berbuat jahat dengan (A) penghapusan hukuman
menyalahgunakan jabatan (B) mengurangkan hukuman
(C) penguatan hukuman
8. Sumber […] minyak di Blok Ambalat (D) pembatalan keputusan hukuman
memicu pertentangan dengan negara (E) pelaksanaan keputusan hukuman
tetangga Malaysia.
12. […] sektor pengangkatan dan komunikasi
Kata yang tepat untuk mengisi bagian yang bagi Kabupaten Wonogiri sebesar Rp229,8
rumpang dalam kalimat itu adalah …. miliar atau 10,2% dari total kegiatan
(A) eksploitasi ekonomi.
(B) eksplorasi
(C) eksplikasi Kata yang tepat untuk melengkapi kalimat
(D) eksplosi di atas adalah ….
(E) eksaminasi (A) distribusi
(B) kontribusi
9. Segala hal yang dilakukan selama ini (C) kuantitas
sebenarnya tidak konsisten. (D) komoditas
(E) aktivitas
Kata konsisten dalam kalimat tersebut
berarti ….
(A) taat asas
(B) ada gunanya
(C) berbahaya
(D) penting
(E) tepat
13. Pendidikan merupakan proses Padanan kata yang tepat dalam bahasa
pendewasaan bangsa yang akan menjadi Indonesia untuk kata stigmatisasi pada
modal utama pembangunan. Karena itu, kalimat kedua dalam paragraf tersebut
pemerintah Indonesia […] anggaran adalah ….
belanja yang sangat besar di bidang (A) petanda
pendidikan. (B) penanda
(C) pertanda
Kata yang tepat untuk mengisi bagian (D) penandaan
kosong pada kalimat di atas adalah …. (E) proses penandaan
(A) memanipulasi
(B) mengakomodasi 17. Sebelum mengikuti seleksi, setiap peserta
(C) menginvestasikan wajib mengisi formulir pendaftaran yang
(D) mengasuransikan telah disediakan oleh panitia.
(E) mengalkulasi
Kata yang tepat untuk menggantikan kata
14. Kelompok Usaha Mandiri Al-Hidayah formulir dalam kalimat tersebut adalah …
hanya menghasilkan kerajinan tangan peci (A) angket
rajut. Setelah beberapa lama berlangsung (B) kisi-kisi
pemasarannya mulai tersendat. Sebagai (C) borang
solusinya diperlukan […] produksi. (D) daftar isian
(A) kreativitas (E) biodata
(B) diversifikasi
(C) inovasi 18. Sampai saat ini, India belum meratifikasi
(D) peningkatan Perjanjian non-Proliferasi Nuklir (NPT).
(E) penambahan
Padanan yang tepat untuk kata
15. Arti kata ambang dalam kalimat meratifikasi dalam kalimat tersebut adalah
Pencemaran udara di beberapa tempat di ….
Yogyakarta sudah berada di ambang (A) mengumumkan
membahayakan kesehatan adalah …. (B) mengesahkan
(A) tingkat (C) memperbaiki
(B) batas (D) merevisi
(C) baku (E) melaksanakan
(D) ukuran
(E) standar 19. Dua atau beberapa universitas yang besar
bergabung menjadi satu.
16. Sepanjang sejarah peradaban manusia,
selama itu pula kusta ada di dunia. Istilah bergabung dalam konteks ini
Stigmatisasi sebagai manusia terkutuk sepadan dengan kata ….
karena dihinggapi penyakit menular tak (A) kombinasi
tersembuhkan ini hingga kini masih (B) merger
menjadi ganjalan utama dalam memutus (C) akomodasi
rantai penularan. Akibatnya, meskipun (D) konvergensi
secara signifikan terjadi penurunan angka (E) amalgamasi
prevalensi, kasus-kasus baru selalu
bermunculan.
20. Setiap universitas di negeri ini mempunyai 24. Untuk tahun ini, banyak perusahaan yang
afiliasi dengan universitas atau perguruan mengalihkan anggaran iklan ke media
tinggi di luar negeri. cetak, terutama surat kabar yang sanggup
meraup 30% dari total belanja iklan.
Kata yang tepat untuk menggantikan kata
afiliasi adalah …. Makna meraup pada kalimat tersebut
(A) pertalian adalah ….
(B) perlindungan (A) memperoleh
(C) persahabatan
(B) menghasilkan
(D) pengakuan
(C) menguntungkan
(E) perujukan
(D) memproduksi
(E) membelanjakan
21. Dalam satu wilayah yang dihuni berbagai
etnik, terjadi sebuah integrasi budaya.
25. Kejadian itu mencoreng … pemerintah
dalam pelaksanaan Ibadan haji.
Makna istilah integrasi dalam kalimat di
atas adalah ….
(A) percampuran Kata yang tepat untuk mengisi rumpang
(B) penyatuan pada kalimat di atas adalah ….
(C) pembauran (A) prioritas
(D) pembentukan (B) stabilitas
(E) penyesuaian (C) kreativitas
(D) kredibilitas
22. Media masa dalam perkembangannya (E) fasilitas
telah bertransformasi menjadi sebuah
industri bisnis yang menjanjikan karena 26. Dalam sidang tersebut, setiap kelompok
kebutuhan masyarakan akan informasi diminta menyampaikan konsiderasi dari
dan hiburan semakin bertambah. kelompoknya dengan maksud agar dalam
pembuatan putusan dapat dihasilkan
Makna istilah bertransformasi dalam putusan yang adil.
kalimat di atas adalah ….
(A) berubah bentuk
Pilihan kata yang tepat untuk
(B) berpindah haluan
menggantikan kata konsiderasi dalam
(C) berbeda tujuan
kalimat tersebut adalah ….
(D) berpindah sasaran
(A) wawasan
(E) berubah pemberitaan
(B) usulan
(C) pertimbangan
23. Tim Mawson menggunakan anjing husky
(D) pemikiran
untuk menghela keret salju dalam
ekspedisi Antartika pada penghujung (E) tanggapan
1912.

Kata menghela mempunyai makna yang


mirip dengan ….
(A) menyeret
(B) menganjak
(C) memajukan
(D) menggeser
(E) menyorong
27. Dengan kompetisi yang sehat, bukan 30. Pascagempa dan tsunami di Nangroe Aceh
hanya penjual yang diuntungkan, pembeli Darussalam dan Sumatera Utara pada 26
juga sangat diuntungkan karena mendapat Desember 2004, beberapa negara
harga yang murah. membantu sistem pemantau gempa dan
peringatan dini tsunami kepada Indonesia.
Pasangan kata yang memiliki hubungan Namun, kompatibilitasnya dipertanyakan
makna yang sama dengan pasangan kata karena alat dari masing-masing negara
bercetak tebal pada kalimat tersebut berbeda standar dan formal.
terdapat pada kalimat …
(A) Seminar itu diikuti oleh guru SD dan
Pengertian kompatibilitas dalam teks
guru SMP.
tersebut adalah ….
(B) Mereka memesan pangsit kering dan
(A) akurat
basah masing-masing sebanyak lima
(B) cara kerja alat
bungkus.
(C) Dari kopral hingga jenderal (C) kesesuaian
diharuskan menaati peraturan sebagai (D) harmonis
prajurit Negara. (E) reaksi dari dalam
(D) Jalan-jalan di kota sangat padat,
sedangkan jalan di desa sangat 31. Perdebatan kedua belah pihak yang
lengang. memiliki paradigma yang berbeda tidak
(E) Orang tua dan anaknya sama-sama akan dapat mencapai solusi yang saling
meninggal dalam musibah banjir. memuaskan.
Kata paradigma dalam kalimat tersebut
28. Setiap habis gajian, ia selalu mentraktir berarti ….
teman-temannya untuk makan bakso di (A) kepentingan kelompok
warung tegal. (B) kerangka pikir
(C) cara kerja
Kata habis dalam pernyataan berikut ini (D) sudut pandang
yang semakna dengan kata habis dalam (E) landasan teori
kalimat tersebut adalah …
(A) Semua dokumen penting yang ada di 32. Terkait dengan komposisi menteri, jika
ruang administrasi habis terbakar.
dilakukan reshuffle kabinet, faktor
(B) Ia tidak bisa mudik lebaran karena
kapabilitas dan integritas menjadi
tiket kereta api sudah habis.
pertimbangan utama.
(C) Uangnya untuk membayar kuliah
habis untuk belanja baju di Matahari.
(D) Habis riwayatnya terkena panah Kata-kata yang tepat untuk menggantikan
beracun. kata yang bercetak miring dalam kalimat
(E) Buku karangan R.A. Kartini berjudul tersebut adalah ….
Habis Gelap Terbitlah Terang. (A) pergeseran, kemampuan, kecakapan
(B) pergantian, kepandaian, diakui
29. Penyelesaian perjanjian ekstradisi antara (C) pengubahan, kemampuan, kejujuran
Indonesia Singapura luas dan rumit. (D) perubahan, kemampuan, kejujuran
(E) pergeseran, kepandaian, kejujuran
Makna kata ekstradisi dalam kalimat
tersebut adalah ….
(A) penyerahan orang bermasalah ke
negara asal
(B) hubungan legal antarnegara
(C) pengaturan hukum antarnegara
(D) kontrak kerja antarnegara
(E) hukum untuk kriminalitas antarnegara
33. Validitas tes yang digunakan untuk 37. Penderita diabetes melitus harus patuh
menguji calon-calon karyawan perusahaan menjalankan pengobatan.
itu masih diragukan.
Kata patuh dalam kalimat di atas
Kata yang tepat untuk menggantikan kata
validitas dalam kalimat tersebut adalah …. bermakna ….
(A) kebenaran (A) cermat
(B) kesahihan (B) teliti
(C) keandalan (C) tunduk
(D) kelengkapan (D) rutin
(E) ketepercayaan (E) tertib

34. Aparat negara tidak diperbolehkan


melakukan intimidasi kepada masyarakat. 38. (1) Di dunia ditemukan dampak
mengerikan akibat pencemaran sampah di
Kata intimidasi dalam kalimat tersebut laut. (2) Contohnya adalah penyu yang
dapat digantikan oleh kata-kata berikut ini, terjerat plastik, anjing laut yang terlilit tali,
kecuali …. dan burung albartos yang mati karena
(A) ancaman sistem pencernaan yang rusak. (3) Di
(B) paksaan
Indonesia pun laut rusak. (4) Salah satunya
(C) kesalahan
(D) gertakan adalah laut di Manta Point, Nusa Penida,
(E) kekejaman Bali. (5) Laut itu sangat kotor sehingga
yang terlihat bukan terumbu karang
35. Ekspor timah dari Indonesia merupakan namun sampah-sampah yang berenang
penghasil timah terbesar kedua di dunia bersama ikan-ikan.
setelah Cina. Namun, semakin berkurang
setelah pemerintah menutup
pertambangan ilegal di Bangka dan Kata mengerikan pada kalimat (1) tidak
Belitung. Upaya ini menjadikan timah tepat sehingga harus diganti dengan kata
sebagai salah satu komoditas terbaik di adalah ….
London Metal Exchange pada tahun-tahun (A) mengenaskan
terakhir. (B) meresahkan
(C) mengkhawatirkan
Kata upaya dalam teks ini sangat tepat bila
(D) menakutkan
diganti dengan ….
(A) usaha (E) mencemarkan
(B) ikhtiar
(C) kebijakan 39. Namun, penjelasan lain mengatakan
(D) kebijaksanaan bahwa cahaya saat makam membuat
(E) tindakan serangga buta karena banjir cahaya di mata
mereka dan membuat mereka bingung.
36. Regulasi yang memihak konsumen dan
pengembang serta pembangunan
infrastruktur dan utilitas bagi perumahan Lawan kata bingung pada kalimat (5) yang
diharapkan segera diwujudkan. tepat adalah ….
(A) gugup
Padanan kata yang tepat untuk kata (B) absolut
regulasi dalam kalimat tersebut adalah …. (C) pasti
(A) pengaturan
(D) lancar
(B) peraturan
(C) aturan (E) yakin
(D) keteraturan
(E) keberaturan
40. (1) Cheran adalah wilayah yang damai makhluk hidup lainnya. (4) Bahkan,
sebelum dikuasai kartel narkotika. (2) Tak beberapa bahan kimia dalam filter rokok
hanya menjalankan bisnis obat-obatan, bekas dikenal sebagai karsinogen, senyawa
kartel disana juga melakukan penebangan penyebab kanker.
kayu secara liar hingga menghancurkan
hutan kuno di Cheran. (3) Situasi tambah Kata terbelah pada kalimat (2) tidak tepat
runyam karena pemerintah dan aparat sehingga harus diganti dengan kata ….
setempat juga turut terlibat. (4) Sebagian (A) hancur
besar tentara pun ikutan menerima suap (B) musnah
kartel. (5) Dengan keadaan yang terus (C) terproses
kacau, masyarakat sipil di Cheran pun (D) terserap
memilih angkat senjata. (6) Upaya tersebut (E) terurai
berhasil. (7) Para politikus korup, polisi
culas, dan sindikat kartel sekaligus para 43. Mereka menetap di Ku-kang setelah
penebang liar tak lagi diberi kesempatan melarikan diri dari tempat asalnya.
untuk menginjak tanah Cheran. (8) Hingga
kini, Cheran masih menjadi kota otonom Lawan kata menetap pada kalimat (3) yang
yang mampu menjalankan kehidupan tepat adalah ….
sosial politiknya tanpa campur tangan (A) mukim
pemerintah. (9) Mereka membangun (B) tinggal
sistem pemerintahan baru tanpa wali kota, (C) nomad
merancang dewan kota, hinggan (D) menjelajah
membentuk milisi sipil untuk (E) bergerak
menggantikan polisi. (10) Hasilnya, Cheran
menjadi salah satu kota dengan tingkat 44. (1) Ayahnya mengerti Syamaran ingin jadi
kriminalitas terendah di Meksiko. pengarang. (2) Kalau Syamaran kerja
Kata yang memiliki makna denotatif kantoran di kampus, itu bagus, karena bisa
‘melawan’ terdapat pada kalimat …. membaca buku di perpustakaan,
(A) (2) mengarang setelah jam kantor, dan
(B) (3) memakain internet sepuasnya. (3)
(C) (4) Sebenarnya impian menjadi pengarang
(D) (5) telah meredup dan menjauh dari pikiran
(E) (6) Syamaran. (4) Ia telah menyelesaikan novel
kelima yang kini bernasib sama dengan
41. Selain itu, lidah buaya juga bisa novel keempat, taka da yang mau
dikonsumsi dalam bentuk makanan dan menerbitkan. (5) Zaman telah berubah,
minuman. masa keemasan novel pelipur lara telah
berlalu. (6) Kerja kantoran, mungkin itulah
Kata dikonsumsi dalam kalimat di atas takdir yang harus Syamaran jalani, dan
bermakna …. kini ia sedang menjemput takdir itu.
(A) dibuat
(B) digunakan Ungkapan yang digunakan penulis untuk
(C) diolah menggambarkan kepasrahan adalah ….
(D) diproses (A) Syamaran ingin menjadi pengarang
(E) dimakan dalam kalimat (1)
(B) Mengarang setelah jam kantor dalam
42. (1) Penelitian terbaru menemukan bahwa kalimat (2)
puntung rokok yang dibuang (C) Impian menjadi pengarang telah meredup
sembarangan akan merusak lingkungan dalam kalimat (3)
hidup. (2) Puntung rokok terdiri dari (D) Tak ada yang mau menerbitkan dalam
ribuan serat selulosa asetat yang kalimat (4)
membutuhkan waktu bertahun-tahun (E) Ia sedang menjemput takdir itu dalam
untuk terbelah secara biologis. (3) Filter kalimat (6)
rokok bebas juga mengandung ribuan
bahan kimia yang dapat membunuh
tanaman, serangga, tikus, jamur, dan
45. (1) Mengambil selfie setelah bencana yang 47. (1) Kadang-kadang sepanjang hati dia
mengerikan telah menjadi kebiasaan dalam berdiri di tebing karang dikelilingi
kehidupan kita sehari-hari. (2) Praktik pemandangan laut yang menakjubkan. (2)
tersebut sama saja dengan perilaku orang Satu-dua dekor kamar hinggap di
yang bergerombol untuk menonton dekatnya. (3) Dia juga suka menyusuri
kecelakaan di jalanan. (3) Ahli media, pantai memunguti braai, karang, atau
Yasmin Ibrahim dari Queen Mary langkitang. (4) Dia menyeruput siput-siput
University, di Inggris menulis sebuah itu cepat bagai kilat langsung dari
artikel yang menarik terkait topik ini. (4) cangkangnya, layanya orang Jepang
memakan kerang. (5) Buruh-buruh miskin
Dia menyebut fenomena ini sebagai “selfie
yang menghuni gubuk-gubuk rumpang di
bencana” atau “pornografi bencana” dan
sisi dermaga segera meniru kelakuannya,
mendefinisikannya sebagai “perilaku
setidaknya perempuan itu mengajarkan
ganjil yang dimotivasi oleh keinginan
kepada mereka satu cara mengatasi
mencapai kepuasan diri sendiri, dengan kelaparan. (6) Karena itulah, setiap sore
situasi pasca bencana sebagai latar para buruh itu mengundangnya ke gubuk
belakang”. (5) Seorang psikoanalis mereka. (7) Perempuan itu seolah-olah
kondang, Carl Jung, berpendapat bahwa menjadi pelindung mereka.
secara alamiah, manusia senang melihat
orang lain menderita, karena hal tersebut Ungkapan yang digunakan oleh penulis
menghibur diri kita, namun kita tidak dalam bacaan untuk melebih-lebihkan
secara langsung terkena dampaknya. (6) perilaku adalah ….
Dengan melihat penderitaan orang lain, (A) sepanjang hari dalam kalimat (1)
kita diberi kesempatan untuk menghakimi (B) yang menakjubkan dalam kalimat (1)
dan menertawakan orang lain, sementara (C) menyeruput siput-siput dalam kalimat
kita terbebaskan dari merasakan (6)
penderitaan. (7) Carl Jung menciptakan (D) bagai kilat dalam kalimat (4)
sebuah istilah yang dikenal sebagai corpse (E) menghuni gubuk-gubuk dalam kalimat
preoccupation untuk merujuk pada (5)
keinginan seseorang untuk menyaksikan
hal-hal yang aneh dan mengerikan. 48. (1) Telepon ganggang atau komputer
biasanya dikendalikan hanya oleh satu
Kata yang memiliki makna denotatif
pengguna. (2) Pengguna jaringan internet
‘terkenal’ terdapat pada kalimat ….
dalam satu rumah tangga umumnya lebih
(A) (2)
dari satu orang. (3) Masing-masing
(B) (3)
pengguna ingin mengendalikan perangkat
(C) (4) secara bersamaan. (4) Penelitian tentang
(D) (5) kontrol akses atas peralatan rumah tangga
(E) (6) yang terhubung ke internet telah dilakukan
para peneliti dari Institut Horst Gortz,
46. Di dalam alat tersebut, unit-unit yang Universitas Chicago, dan Universitas
terdiri atas molekul-molekul sederhana Washington. (5) Mereka mewawancarai
dibuat. Lawan kata sederhana yang tepat 425 pengguna di Amerika tentang
dalam kalimat (2) adalah …. preferensi mereka dan mendapatkan saran
(A) sulit untuk mengelola akses para pengguna
(B) canggih tersebut. (6) Diperoleh sejumlah faktor
(C) kompleks kontekstual yang memengaruhi hak akses,
(D) terstruktur seperti usia, tempat orang atau perangkat
(E) terpadu itu berada, apakah perangkat itu telah
digunakan sebelumnya, waktu dalam
sehari, dan biaya yang dikeluarkan untuk
menggunakannya.
Kata yang memiliki makna denotatif 50. (1) Energi merupakan kemampuan untuk
‘pilihan’ terdapat pada kalimat …. melakukan kerja dan menjadi hal yang
(A) (2) fundamental dalam kehidupan. (2) Selama
(B) (3)
ada kehidupan pasti terdapat energi. (3)
(C) (4)
(D) (5) Sebagian besar energi adalah energi listrik.
(E) (6) (4) Energi listrik memiliki fungsi yang
sangat krusial dalam kehidupan kita,
49. (1) Pijat bayi merupakan terapi alternatif terutama sebagai penerang pada malam
yang sudah diterapkan sejak lama. (2) hari. (5) Kita semua menggunakan energi
Namun, sekarang pijat bayi digali kembali listrik dalam kehidupan sehari-hari, seperti
oleh ilmu kesehatan dan dijadikan kajian berkendara, memasak, dan memanaskan
ilmiah para pakar. (3) Saat ini pijat bayi atau mendinginkan sesuatu, bahkan
didefinisikan sebagai terapi sentuh dalam menggunakan internet untuk membaca
seni perawatan dan pengobatan dengan berita.
kontak tubuh berkelanjutan yang
menimbulkan rasa nyaman dan aman Kata krusial pada kalimat (4) tidak tepat
sehingga meningkatkan kesehatan fisik sehingga harus diganti dengan kata adalah
dan psikologis bayi. (4) Pijat bayi ….
membantu tumbuh kembang fisik dan (A) penting
emosi bayi dan mempererat hubungan ibu (B) krisis
dan bayi. (C) genting
(D) utama
Kata seni dalam kalimat (3) bermakna …. (E) fundamental
(A) model
(B) gaya
(C) cara
(D) tipe
(E) pola
SINTAKSIS

Sintaksis atau tata kalimat merupakan ilmu yang berhubungan dengan pembentukan kalimat. Dalam
tes seleksi masuk PTN, soal-soal sintaksis yang sering muncul berkaitan dengan (1) unsur kalimat, (2)
pola kalimat, dan (3) jenis kalimat.

A. Unsur Kalimat
Unsur sebuah kalimat terdapat kata, frasa, dan klausa.
(a) Kata
Kata adalah unsur gramatikal terkecil yang memiliki makna.
Contoh:
raja
daun

(b) Frasa
Frasa adalah gabungan dua kata atau lebih.
raja hutan raja hutan rimba
daun pintu daun pintu itu

Frasa memiliki pola D-M (diterangkan-menerangkan) atau M-D (menerangkan-diterangkan).


raja hutan (D-M)
daun pintu (M-D)

(c) Klausa
Klausa adalah satuan gramatikal yang berpotensi menjadi kalimat.
raja hutan rimba tertidur Ia mengendap-endap ketika raja hutan rimba tertidur.
daun pintu itu keropos Daun pintu itu keropos karena rayap.

B. Pola Kalimat
Dalam sebuah kalimat, terdiri dari beberapa fungsi seperti subjek, predikat, objek, pelengkap, dan
keterangan.
(a) Subjek
Hal yang dibahas dalam kalimat (apa, siapa).
Contoh:
Pemerintah menurunkan harga BBM.
S

(b) Predikat
Bagian kalimat yang menjelaskan subjek (kata/frasa verba, kata/frasa adjektiva, kopula).
Contoh:
Pemerintah menurunkan harga BBM.
P
Pemerintah adalah badan tertinggi yang memegang wewenang suatu negara.
P

(c) Objek
Unsur kalimat yang muncul setelah predikat yang berupa kata kerja transitif (membutuhkan
objek; dapat dipasifkan; serta mendapat imbuhan me-, memper-, memper-kan, me-i, memper-
i, dan me-kan).
Contoh:
Pemerintah menurunkan harga BBM.
O

(d) Pelengkap
Unsur kalimat yang muncul setelah predikat yang berupa kata kerja intransitif (tidak
membutuhkan objek; tidak dapat dipasifkan; serta mendapat imbuhan ber-, ber-kan, ter-, ke-
an, dan di-).
Contoh:
Kenaikan harga BBM perlu ditinjau kembali penyebabnya.
Pel.

(e) Keterangan
Unsur kalimat yang menerangkan bagian kalimat lain. Keterangan dapat berpindah posisi
dan biasanya ditandai dengan kata depan atau kata hubung.
Contoh:
Kenaikan harga BBM perlu ditinjau kembali oleh pemerintah.
K

Struktur inti kalimat minimal terdapat S + P


• Bahasa akan terus berkembang karena mengikuti perubahan zaman.
S P
• Setiap bangsa memiliki bahasa yang terus berkembang karena mengikuti perubahan zaman.
S P O

C. Jenis Kalimat
Jenis kalimat, yaitu kalimat aktif dan kalimat pasif.
(a) Kalimat aktif
Kalimat aktif adalah kalimat yang memiliki subjek melakukan perbuatan atau sesuatu.
Terdapat dua jenis kalimat aktif, yaitu kalimat aktif transitif dan kalimat aktif intransitif.
Kalimat aktif transitif adalah kalimat yang memiliki predikat transitif (membutuhkan objek;
dapat dipasifkan; serta mendapat imbuhan me-, memper-, memper-kan, me-i, memper-i, dan
me-kan).
Contoh:
Adik membaca buku ensiklopedia tumbuhan. (transitif)
S P O

Kalimat aktif intransitif adalah kalimat yang memiliki predikat intransitif (tidak
membutuhkan objek; tidak dapat dipasifkan; serta mendapat imbuhan ber-, ber-kan, ter-, ke-
an, dan di-).
Contoh:
Adik berusia sembilan tahun. (intransitif)
S P Pel.

(b) Kalimat pasif


Kalimat pasif adalah kalimat dengan subjek dikenakan perbuatan atau sesuatu.
Contoh:
Buku ensiklopedia tumbuhan dibaca adik.
S P O
1. Pejabat yang dikenal malas itu datang 7. Jumlah frasa dalam kalimat Teman saya
pukul dua siang. sedang membeli sepatu baru adalah ….
(A) 2 buah
Inti frasa yang menjadi komponen pokok (B) 3 buah
dalam pembentukan kalimat di atas adalah (C) 4 buah
…. (D) 5 buah
(A) pejabat dan malas (E) 6 buah
(B) pejabat dan terkenal
(C) pejabat dan datang 8. Presiden Joko Widodo sedang
(D) terkenal dan malas mengusahakan tol lintas Sumatera akan
(E) malas dan datang selesai pada tahun 2019.

2. Perhatikan kalimat-kalimat berikut ini! Kalimat tersebut memiliki frasa ….


Kaki kiri Rina terkilir (A) 2 buah
Anak kecil itu menangis (B) 3 buah
Dinda membeli sebuah mobil (C) 4 buah
(D) 5 buah
Komponen pokok pada frase bertingkat di (E) 6 buah
atas adalah ….
(A) kaki, kecil, mobil 9. Aksi jual beli saham di Bursa Efek Jakarta
(B) kiri, kecil, mobil kembali bergairah.
(C) kaki, anak, sebuah
(D) kaki, anak, mobil Pasangan kata di bawah ini yang setipe
(E) kiri, kecil, sebuah dengan jual beli ialah ….
(A) pantang mundur
3. Frase bertingkat yang sepola dengan (B) terus terang
sedang menulis adalah …. (C) serah terima
(A) suka sekali (D) cuci tangan
(B) akan pergi (E) sepak terjang
(C) kakak saya
(D) cantik sekali 10. Pola gabungan kata baju biru terdapat juga
(E) menulis surat pada gabungan kata ….
(A) doa restu
4. Susunan kata yang tidak mengikuti hukum (B) ke Jakarta
DM adalah …. (C) rumah baru
(A) raja muda (D) panjang tangan
(B) rambut gondrong (E) tangan dingin
(C) merah jambu
(D) suara merintih
(E) beberapa contoh 11. Frasa manakah yang pola maknanya sama
dengan pola makna frasa air minum?
5. Pola gabungan kata ikat pinggang sama (A) frasa pembelajaran campuran dalam
dengan pola gabungan kata …. kalimat (1)
(A) sarung sutera
(B) frasa kelompok-kelompok kecil dalam
(B) minyak goreng
kalimat (2)
(C) celana panjang
(D) besi berani (C) frasa waktu belajar dalam kalimat (4)
(E) tusuk gigi (D) frasa metode pembelajaran dalam
kalimat (6)
6. Pola pembentukan frasa lemah gemulai (E) frasa kesenjangan digital dalam
sama dengan …. kalimat (7)
(A) sangat cantik
(B) teramat banyak
(C) akan datang
(D) sopan santun
(E) ke sekolah
12. Frasa manakah yang berkandungan jenis 16. Frasa manakah yang pola maknanya sama
kata yang sama dengan jenis kata sinar dengan pola makna frasa pengobatan
pada frasa pancaran sinar? klinis?
(A) Frasa berbagai hal dalam kalimat (1) (A) Frasa gejala hiperaktif dalam kalimat
(B) Frasa sebuah topan dalam kalimat (2) (1)
(C) Frasa komposisi mineral dalam (B) Frasa anak perempuan dalam kalimat
kalimat (4) (1)
(D) Frasa tekanan magma dalam kalimat (C) Frasa kurang perhatian dalam kalimat
(5)
(2)
(E) Frasa gunung berapi dalam kalimat (7)
(D) Frasa kegiatan penyesuaian dalam
kalimat (4)
13. Frasa manakah yang berkandungan jenis
(E) Frasa lingkungan kelas dalam kalimat
kata yang sama dengan jenis kata
(4)
masyarakat pada frasa banyak
masyarakat?
17. Konstruksi sejenis “Sepatu kickers itu
(A) Frasa pengamat lingkungan dalam
kalimat (2) mahal” (bukan sepatu kickers mahal)
(B) Frasa kesehatan manusia dalam terdapat pada ….
kalimat (2) (A) armada kapal udara
(C) Frasa mayoritas responden dalam (B) sistem angkutan kolektif
kalimat (3) (C) yang hadir terpesona
(D) Frasa rekayasa genetika dalam kalimat (D) menteri perhubungan darat
(4) (E) tanah milik BUMN
(E) Frasa kedelai impor dalam kalimat (5)
18. Bentuk “sepeda baru yang mahal” sama
14. Frasa manakah yang pola maknanya sama dengan bentuk di bawah ini, kecuali ….
dengan pola makna frasa binatang buas? (A) jaket impor terbaru
(B) tas kargo untuk tamasya
(A) Frasa pola bergaris dalam kalimat (1) (C) sepatu itu mahal harganya
(B) Frasa ahli biologi dalam kalimat (2) (D) jam dinding yang indah
(C) Frasa banyak ide dalam kalimat (3) (E) roda sepeda yang berputar
(D) Frasa upaya serius dalam kalimat (3)
(E) Frasa tangkapan predator dalam
19. Bentuk yang merupakan frasa terdapat
kalimat (4)
pada ….
(A) orang yang berkacamata hitam yang
15. Frasa manakah yang berkandungan jenis
sering duduk di depan rumah
kata yang sama dengan jenis kata belakang (B) menjadi artis terkenal adalah tujuan
pada frasa pintu belakang? hidupnya
(A) Frasa kelenjar tiroid pada kalimat (1) (C) sepeda barunya itu diberikan oleh
(B) Frasa lobus kanan dalam kalimat (2) ayahnya
(C) Frasa mekanisme umpan dalam (D) Tupperware ibu tertinggal di
kalimat (3) sekolahku
(D) Frasa metabolisme tubuh dalam (E) ibu marah kepadaku
kalimat (5)
(E) Frasa pertumbuhan tulang dalam
kalimat (5)
20. Nelayan menangkap ikan di laut. (E) Dengan mengomunikasi suatu
Pola kalimat di atas sama dengan pola lembaga kepada masyarakat, dapat
kalmia berikut, kecuali … ditempuh bermacam-macam cara.
(A) Siswa membaca buku di perpustakaan.
(B) Kamu bergembira pada pesta ulang 24. Kalimat yang polanya sama dengan
tahunnya. kalimat “Akhir-akhir ini flu burung
(C) Kemarin polisi menangkap pencopet menyerang unggas di beberapa wilayah di
di pasar itu. Jawa Tengah dan DIY” adalah …
(D) Kami harus mengembalikan buku-
(A) Belakangan ini minat orang
buku itu pada tempatnya semula.
mengonsumsi ayam goreng menurun
(E) Mereka menonton pesta rakyat di alun-
karena takut tertular flu burung.
alun.
(B) Tiga tahun lalu chikungunya hanya
meresahkan warga Kulon Progo dan
21. Pola kalimat inti yang sama dengan
Purworejo.
“Pemandangan di desa itu sangat indah”
(C) Dua tahun lalu chikungunya
adalah …
(A) Para mahasiswa harus belajar tekun. menyerang penduduk di daerah
(B) Baju seragam kerja ayah sangat bagus. Purworejo.
(C) Indahnya pemandangan di Puncak (D) Bulan lalu unggas yang terinfeksi flu
Pas. burung dimusnahkan dengan cara
(D) Keberaniannya tidak diragukan lagi. dikubur dan dibakar.
(E) Kegigihan para mahasiswa pada masa (E) Bulan ini calon mahasiswa UGM
reformasi membuahkan hasil. mengikuti ujian materi kognitif dan
nonkognitif.
22. Pelayan toko memberikan uang kembali.
Pola kalimat di atas sama dengan pola 25. Kalimat “Koran Masuk Desa merupakah
kalimat … langkah yang strategis guna meningkatkan
(A) Ketua RT ikut berperan dalam kecerdasan masyarakat” memiliki pola
kebersihan lingkungan. yang sama dengan …
(B) Kakak belajar dengan tekun di (A) Pemerintah mengajukan beberapa usul
guna membangun desa-desa
perpustakaan.
tertinggal.
(C) Adik menangis di pelukan Ibu.
(B) Pemerintah meningkatkan
(D) Kereta api berhenti di setiap stasiun. kesejahteraan masyarakat dengan
(E) Bapak Kepala Sekolah menyerahkan proyek-proyek unggulan.
buku Tabanas. (C) Usaha pemerintah berupa pemberian
subsidi kepada rakyat kecil harus
23. Pola kalimat “Sekolah kami selalu didukung.
mendapat predikat sekolah bersih” (D) Usaha itu adalah strategi yang tepat
terdapat juga dalam kalimat … untuk membangun kesadaran rakyat.
(E) Pemerintah berusaha meningkatkan
(A) Salah satu sekolah negeri di kota kami
kesejahteraan rakyat dengan upaya.
membentengi siswa-siswanya dari
pengaruh negatif televisi dengan
pendidikan moral.
(B) Dalam majalah ini, dimuat artikel-
artikel yang bermutu ilmiah.
(C) Dengan tugas sekompleks itu, idealnya
BP dapat lebih dekat dengan siswa.
(D) Dalam dasawarsa ini, dia ingin terus
berprestasi dalam olahraga bela diri.
26. (1) Selama bertahun-tahun, para ilmuwan (B) Kalimat (2)
telah mencoba menjelaskan mengapa laron (C) Kalimat (3)
dan serangga lainnya tertarik pada cahaya, (D) Kalimat (4)
tetapi para ilmuwan tidak sepenuhnya (E) Kalimat (5)
yakin. (2) Salah satu penjelasan adalah
beberapa serangga menggunakan bulan 28. (1) Ayahnya mengerti Syamaran ingin jadi
atau bintang sebagai alat bantu pencari pengarang. (2) Kalau Syamaran kerja
arah. (3) Bagi serangga, lampu jalan kantoran di kampus, itu bagus, karena bisa
mungkin terlihat seperti bulan sehingga membaca buku di perpustakaan,
dapat menyesatkan mereka. (4) Penjelasan mengarang setelah jam kantor, dan
lainnya adalah cahaya mengecoh serangga memakain internet sepuasnya. (3)
untuk melihat ilusi visual dari area yang Sebenarnya impian menjadi pengarang
lebih gelap di sekitar tepi lampu, yang telah meredup dan menjauh dari pikiran
disebut Mach Bands, dan serangga terbang Syamaran. (4) Ia telah menyelesaikan novel
menuju tempat gelap untuk sembunyi. (5) kelima yang kini bernasib sama dengan
Namun, penjelasan lain mengatakan novel keempat, taka da yang mau
bahwa cahaya saat malam membuat menerbitkan. (5) Zaman telah berubah,
serangga buta karena banjir cahaya di mata masa keemasan novel pelipur lara telah
mereka dan membuat mereka bingung. berlalu. (6) Kerja kantoran, mungkin itulah
takdir yang harus Syamaran jalani, dan
Kalimat manakah pada bacaan di atas yang kini ia sedang menjemput takdir itu.
memiliki pola dasar sama dengan pola
dasar pada kalimat Serangga telah Bentuk aktif dari kalimat pasif Sampai
terperangkap di permukaan yang tidak selarut ini pekerjaan yang menumpuk itu
alami sehingga mereka menjadi mangsa belum juga diselesaikan olehnya memiliki
empuk bagi burung dan kadal? pola yang sama dengan pola kalimat ….
(A) Kalimat (1) (A) (1) (D) (4)
(B) Kalimat (2) (B) (2) (E) (5)
(C) Kalimat (3) (C) (3)
(D) Kalimat (4)
(E) Kalimat (5) 29. (1) Beragam plastik baru yang kita
gunakan saat ini dibuat dengan mencacah
27. (1) Ma Huan, penerjemah Cheng Ho, minyak bumi dalam sebuah alat yang
mencatat dalam Yingya Shenglan (1416) disebut cracker di pabrik petrokimia. (2) Di
bahwa Ku-Kang dikenal sebagai San-bo- dalam alat tersebut, unit-unit yang terdiri
zhai atau Palembang. (2) Waktu wilayah atas molekul-molekul sederhana dibuat. (3)
ini menjadi bawahan Jawa, sejumlah besar Unit-unit ini kemudian dikombinasikan
penduduknya berasal dari Guangdong, menjadi konfigurasi yang berbeda-beda.
Zhangzhou, dan Quanzhou. (3) Mereka (4) Untuk menerapkan proses yang sama
menetap di Ku-kang setelah melarikan diri terhadap sampah plastik, perlu
dari tempat asalnya. (4) Chen Chen Zuyi dikembangkan proses baru. (5) Para
dan teman-temannya termasuk yang peneliti di Universitas Chalmers sedang
melarikan diri dari Guangdong pada tarikh mengusahakan aspek teknis agar proses
Hongwu atau sekitar 1368-1398. (5) Mereka seperti itu dapat dirancang dan
membawa seluruh keluarganya dan diintegrasikan di pabrik petrokimia yang
menetap di Ku-kang. ada, tetapi dengan biaya jauh lebih rendah.

Kalimat manakah Menurut sejarawan Kalimat manakah pada bacaan di atas yang
Universitas Indonesia, Didik Pradjoko dan memiliki pola dasar sama dengan pola dsar
arkeolog Bambang Budi Utomo dalam pada kalimat Agar tercapai hasil optimal
Atlas Pelabuhan-Pelabuhan Bersejarah di dari proses penanganan sampah,
Indonesia, sekira abad ke-15, setelah sebaiknya dibuat panduan lengkap yang
Sriwijaya mengalami kemunduran, terjadi efektif?
masa kekosongan pemerintah di (A) (1) (D) (4)
Palembang? (B) (2) (E) (5)
(A) Kalimat (1) (C) (3)
30. (1) Kadang-kadang sepanjang hati dia 32. Perhatikan kalimat berikut!
berdiri di tebing karang dikelilingi Peristiwa itu terjadi sejak aturan baru yang
pemandangan laut yang menakjubkan. (2) ketat itu diumumkan.
Satu-dua dekor kamar hinggap di Dalam aturan itu, setiap siswa yang
dekatnya. (3) Dia juga suka menyusuri terlambat dikenakan wajib lapor.
pantai memunguti braai, karang, atau
langkitang. (4) Dia menyeruput siput-siput Penggabungan kedua kalimat tersebut
itu cepat bagai kilat langsung dari yang benar adalah …
cangkangnya, layanya orang Jepang (A) Peristiwa itu terjadi sejak aturan baru
memakan kerang. (5) Buruh-buruh miskin yang ketat itu diumumkan, dimana
yang menghuni gubuk-gubuk rumpang di setiap siswa yang terlambat dikenakan
sisi dermaga segera meniru kelakuannya, wajib lapor.
setidaknya perempuan itu mengajarkan (B) Setiap siswa yang terlambat dikenakan
kepada mereka satu cara mengatasi wajib lapor sejak aturan baru yang
kelaparan. (6) Karena itulah, setiap sore ketat itu diumumkan.
para buruh itu mengundangnya ke gubuk (C) Ketika aturan baru yang ketat itu
mereka. (7) Perempuan itu seolah-olah diumumkan, setiap siswa yang
menjadi pelindung mereka. terlambat dikenakan wajib lapor.
(D) Peristiwa bahwa setiap siswa yang
Bentuk aktif dari kalimat pasif Seekor terlambat dikenakan wajib lapor
camar dilepaskan perempuan itu ke laut dilakukan sejak aturan baru yang ketat
lepas memiliki pola yang sama dengan itu diumumkan.
pola kalimat …. (E) Aturan baru yang ketat itu
(A) (1) diumumkan, karena itu setiap siswa
(B) (2) yang terlambat dikenakan wajib lapor.
(C) (3)
(D) (4) 33. Dengan bergotong royong, rasa persatuan
(E) (5) dan kesatuan menjadi semakin erat.
Gotong royong pun dapat menghemat
31. (1) Kata Kakek, belum saatnya aku pengeluaran kegiatan.
berkenalan dengan kesediahn, dipeluk
perpisahan, apalagi dibunuh kebencian. (2) Kalimat mana yang merupakan hasil
Itu juga yang ia katakana ketika aku pemaduan dua kalimat tersebut?
menangis di hari kematian Nenek. (3) Di (A) Tidak hanya rasa persatuan dan
Rupit, laki-laki 80 tahun itu merawat aku
kesatuan menjadi semakin erat, tetapi
seperti menyirami Bumi dengan dendam
gotong royong dapat juga menghemat
pada Negara yang dibawa mati. (4) Ketika
pengeluaran kegiatan.
beranjak remaja, aku mulai mengerti. (5)
(B) Selain mempererat rasa persatuan dan
Baginya, label veteran, janji tunjangan
pejuang, hadiah umrah, dan bingkisan kesatuan, gotong royong dapat
sarung murahan adalah satir rutin 17 menghemat pengeluaran kegiatan.
Agustusan. (6) Dia malah bersyukur tidak (C) Dengan bergotong royong, rasa
diakui sebagai pejuang kalai persatuan dan kesatuan menjadi
keberadaannya hanya untuk semakin erat sehingga dapat
seremonialitas itu. menghemat pengeluaran kegiatan.
(D) Rasa persatuan dan kesatuan menjadi
Bentuk aktif dari kalimat pasif Aku ditegur semakin erat, di samping dapat
oleh nasib layaknya gelombang yang menghemat pengeluaran kegiatan,
menghantam sampan memiliki pola yang dengan bergotong royong.
sama dengan pola kalimat …. (E) Dengan bergotong royong, rasa
(A) (1) persatuan dan kesatuan menjadi
(B) (2) semakin erat dan dapat menghemat
(C) (3) pengeluaran kegiatan.
(D) (4)
(E) (5)
34. Hingga tahun 2019 Badan Pengembangan 35. Wayang merupakan media yang dapat
Bahasa dan Perbukuan telah memetakan membentuk karakter anak bangsa. Hal ini
bahasa-bahasa daerah di Indonesia. Hasil terjadi karena wayang sebagai sebuah
pemetaan itu menunjukkan bahwa gambaran karakter manusia dapat
Indonesia memiliki 718 bahasa daerah. diartikan pula sebagai gambaran
kehidupan manusia.
Kalimat mana yang merupakan hasil
pemaduan kedua kalimat di atas? Kalimat mana yang merupakan hasil
(A) Hingga tahun 2019 badan pemaduan kalimat di atas?
pengembangan bahasa dan perbukuan (A) Wayang merupakan media
telah memetakan bahasa-bahasa pembentukan karakter anak bangsa
daerah di Indonesia sehingga sebagai gambaran kehidupan manusia.
menghasilkan 718 bahasa daerah. (B) Wayang merupakan media yang dapat
(B) Hingga tahun 2019 badan membentuk karakter anak bangsa
pengembangan bahasa dan perbukuan karena menggambarkan karakter
telah memetakan bahasa-bahasa manusia.
daerah di Indonesia dan hasilnya (C) Wayang merupakan media yang dapat
menunjukkan bahwa Indonesia membentuk karakter anak bangsa
memiliki 718 bahasa daerah. sebagai gambaran kehidupan manusia.
(C) Hingga tahun 2019 badan (D) Wayang merupakan media pembentuk
pengembangan bahasa dan perbukuan karakter anak bangsa yang berarti
telah memetakan bahasa-bahasa gambaran kehidupan manusia.
daerah di Indonesia dan menunjukkan (E) Wayang merupakan media yang
bahwa Indonesia memiliki 718 bahasa membentuk karakter anak bangsa
daerah. karena dapat berarti gambaran
(D) Hingga tahun 2019 badan kehidupan manusia.
pengembangan bahasa dan perbukuan
telah memetakan bahasa-bahasa 36. Perhatikan kalimat berikut!
daerah di Indonesia dengan hasil yang Orang berkemeja biru itu pandai.
menunjukkan bahwa Indonesia Ia menduduki jabatan penting.
memiliki 718 bahasa daerah.
(E) Hingga tahun 2019 badan Penggabungan yang tepat bagi kedua
pengembangan bahasa dan perbukuan
kalimat di atas adalah …
telah memetakan bahasa-bahasa
(A) Orang berkemeja biru itu pandai dan
daerah di Indonesia dan ternyata
Indonesia memiliki 718 bahasa daerah. menduduki jabatan penting.
(B) Ia menduduki jabatan penting di
kantornya sebab ia pandai.
(C) Orang berkemeja biru yang
menduduki jabatan penting itu pandai.
(D) Orang berkemeja biru itu pandai
meskipun menduduki jabatan penting.
(E) Orang itu menduduki jabatan penting
agar supaya pandai.
37. Perhatikan kalimat berikut! sangat kondusif bagi perkembangan
Pertumbuhan industri tergantung pada ekonomi. […] hubungan antara ekonomi
tenaga listrik dan produk kimia. dan pertahanan keamanan dapat
Produk kimia cenderung menjadi sumber diibaratkan dua sisi sekeping mata uang.
pencemaran lingkungan.
Kita harus menjaga kelestarian Kata-kata yang tepat untuk melengkapi
lingkungan. bacaan di atas adalah ….
(A) bahwa, di sini, di lain pihak, dengan
Rangkaian yang tepat dari ketiga kalimat begitu
tersebut adalah … (B) jika, di satu pihak, sementara di pihak
(A) Pertumbuhan industri tergantung yang lain, dalam hal ini
pada tenaga listrik dan produk kimia (C) bahwa, di satu sisi, di sisi lain, sehingga
yang cenderung menjadi sumber (D) bahwa, di satu sisi, sementara di sisi
pencemaran lingkungan dan kita harus yang lain, dengan demikian
menjaga kelestariannya. (E) kalau, dalam hal ini, di samping itu,
(B) Kita harus menjaga kelestarian sehingga
lingkungan karena pertumbuhan
industri cenderung pada tenaga listrik 39. Umumnya usaha kecil lahir dari usaha
dan produk kimia yang cenderung keluarga. Usaha kecil memang merupakan
menjadi sumber pencemaran bagian dari upaya peningkatan
lingkungan. pendapatan keluarga.
(C) Produk kimia cenderung menjadi
sumber pencemaran lingkungan Kata yang tepat untuk menggabungkan
sehingga kita harus menjaga dua kalimat di atas adalah …
kelestarian lingkungan karena (A) meskipun
pertumbuhan industri bergantung (B) jika
pada tenaga listrik dan produk kimia. (C) sedangkan
(D) Pertumbuhan industri tidak hanya (D) karena
bergantung pada tenaga listrik, tetapi (E) tetapi
juga bergantung pada produk kimia
yang cenderung menjadi sumber 40. Situasi tambah runyam karena pemerintah
pencemaran lingkungan sehingga kita dan aparat setempat juga turun terlibat.
harus menjaga kelestariannya. Sebagian besar tentara pun ikutan
menerima suap kartel.
(E) Kita harus menjaga kelestarian
lingkungan yang diakibatkan oleh Gagasan pada kedua kalimat akan menjadi
pertumbuhan industri yang bukan saja utuh bila keduanya dihubungan dengan
bergantung pada tenaga listrik, ….
melainkan juga produk kimia (A) padahal
cenderung menjadi sumber. (B) artinya
(C) jelasnya
(D) bahkan
38. Tidak dapat disangkal […] ada hubungan
(E) singkatnya
erat antara kondisi ekonomi dengan
kekuatan pertahanan keamanan suatu
bangsa. […] kondisi ekonomi yang baik
dapat memberikan dukungan pada
terjaminnya pertahanan dan keamanan
negara. […] pertahanan keamanan yang
memadai dapat memberikan suasana yang
41. Dengan melihat penderitaan orang lain, berfungsi sebagai penghubung
kita diberi kesempatan untuk menghakimi antarkalimat. Kata-kata perangkat itu
dan menertawakan orang lain, sementara adalah ….
kita terbebaskan dari merasakan (A) namun, hal itu, akan tetapi
penderitaan Carl Jung menciptakan sebuah (B) meskipun demikian, oleh sebab itu,
istilah yang dikenal sebagai corpse dengan demikian
preoccupation untuk merujuk pada (C) sedangkan, akibatnya, jadi
keinginan seseorang untuk menyaksikan (D) namun, akibatnya, jadi
hal-hal yang aneh dan mengerikan. (E) adapun, namun, bahkan

Gagasan pada kalimat (6) dan (7) akan 44. Penggunaan kata penghubung
menjadi utuh bila keduanya dihubungan intrakalimat yang tepat terdapat dalam
dengan …. kalimat …
(A) lalu (A) Walaupun sudah dilakukan pendataan
(B) kemudian secara cermat, tetapi masih juga ada
(C) maka keluarga miskin yang belum terdaftar
(D) padahal untuk mendapatkan subsidi BBM.
(E) oleh karena itu (B) Jika kurang koordinasi antara pihak
eksekutif dan legislatif, maka semua
42. Mereka mewawancarai 425 pengguna di peraturan yang dibuat selalu
Amerika tentang preferensi mereka dan menimbulkan masalah dalam
mendapatkan saran untuk mengelola akses pelaksanaannya.
para pengguna tersebut. diperoleh (C) Kondisi perekonomian negara akan
sejumlah faktor kontekstual yang semakin terpuruk jika pemerintah
memengaruhi hak akses, seperti usia, tidak segera menyatukan visi dan
tempat orang atau perangkat itu berada, misinya bersama anggota dewan.
apakah perangkat itu telah digunakan (D) Dengan diberlakukannya keputusan
sebelumnya, waktu dalam sehari, dan sepihak oleh pemerintah, maka dewan
biaya yang dikeluarkan untuk juga mengambil keputusan sepihak
menggunakannya. tanpa menghiraukan suara rakyatnya.
(E) Pemerintah tidak bermaksud untuk
Gagasan pada kedua kalimat akan menjadi memberatkan kehidupan masyarakat,
utuh bila keduanya dihubungan dengan melainkan bermaksud untuk
…. menyelamatkan perekonomian negara.
(A) hasilnya
(B) artinya 45. Asam folat selama ini dikenal luas sebagai
(C) jelasnya suplemen wajib pada masa prakehamilan
(D) akibatnya dan kehamilan […] timbul anggapan
(E) singkatnya bahwa zat ini hanya dibutuhkan oleh para
calon ibu. […], baik pria maupun wanita
43. Perhatikan paragraf berikut! sama-sama membutuhkan asupan harian
Kanker perut kini masih menjadi penyebab asam folat. […], kebutuhan harian asam
kematian tertinggi kedua. […], dalam folat pria dewasa sama besarnya dengan
beberapa dekade terakhir ini jumlah kebutuhan wanita dewasa.
kasusnya cenderung menurun […]
mungkin terjadi karena tingginya Kata yang tepat untuk mengisi tiga bagian
konsumsi buah-buahan dan sayuran segar, kosong dalam bacaan di atas berturut-turut
serta berkurangnya kandungan garam adalah ….
dalam makanan akibat proses (A) lalu, oleh karena itu, sedangkan
pendinginan. […] di negeri ini kasus (B) dan, walaupun, adapun
kandungan akibat kanker perut masih (C) sehingga, padahal, bahkan
menjadi masalah utama yang harus diatasi. (D) meskipun, akan tetapi, jadi
(E) karena, sebenarnya, dengan demikian
Agar paragraf itu menjadi susunan yang
padu, tempat-tempat kosong itu harus diisi
dengan kata-kata perangkai yang
46. Perhatikan kalimat berikut! 49. Bahan bakar fosil ini berasal dari hewan
Ani gemar menanam sayur-sayuran. atau tumbuhan sisa organisme yang matu
Ari gemar memasak. jutaan tahun yang lalu dan mengalami
penguraian dalam bentuk hidrokarbon.
Kedua kalimat di atas dapat digabungkan
menjadi kalimat majemuk Apa makna kalimat di atas?
mempertentangkan apabila dihubungkan (A) Hewan atau tumbuhan yang mati
dengan kata …. jutaan tahun yang lalu mengalami
(A) karena itu penguraian hidrokarbon menjadi
(B) dan bahan bakar fosil.
(C) namun (B) Sisa organisme dari hewan atau
(D) sedangkan tumbuhan jutaan tahun lalu menjadi
(E) kemudian fosil sesudah mengalami penguraian.
(C) Senyawa hidrokarbon yang
47. Akhir-akhir ini tidak sedikit buruh mengalami penguraian merupakan
berbagai perusahaan yang diberhentikan. awal terbentuknya bahan bakar fosil.
Tidak aneh jika kerusuhan mudah sekalli (D) Bahan bakar fosil terbentuk dari hasil
muncul. penguraian hewan atau tumbuhan
yang mati jutaan tahun lalu dalam
Konjungsi yang tepat untuk bentuk hidrokarbon.
menggabungkan kalimat di atas adalah …. (E) Fosil berasal dari hidrokarbon yang
(1) dengan demikian dibentuk dari organisme hewan atau
(2) oleh karena itu tumbuhan yang mati jutaan tahun
(3) jadi yang lalu.
(4) sehingga
50. Bengkalis, sebuah kabupaten di Riau yang
48. Filter rokok bekas juga mengandung sebagian wilayahnya terletak di Riau
ribuan bahan kimia yang dapat Kepulauan, ternyata memiliki potensi alam
membunuh tanaman, serangga, tikus, yang sangat besar.
jamur, dan makhluk hidup lainnya.
Kalimat di atas merupakan perluasan dari
Apa makna kalimat di atas? kalimat …
(A) Filter rokok bekas mengandung bahan (A) Bengkalis sebuah kabupaten di Riau.
kimia yang dapat membunuh (B) Sebagian wilayah Bengkalis terletak di
tumbuhan, hewan, dan makhluk Riau daratan.
hidup. (C) Sebagian lainnya terletak di Riau
(B) Filter rokok bekas dapat membunuh Kepulauan.
tanaman, serangga, tikus, jamur, dan (D) Bengkalis memiliki potensi alam.
makhluk hidup lainnya. (E) Potensi alam yang sangat besar.
(C) Dalam filter rokok bekas tergantung
ribuan zat kimia yang merusak
lingkungan makhluk hidup.
(D) Filter rokok bekas berunsur ribuan zat
kimia beracun yang dapat membunuh
makhluk hidup.
(E) Dalam filter rokok bekas terdapat
banyak zat kimia yang berbahaya bagi
makhluk hidup.
51. Deretan gunung dalam kelompok 54. Otonomi sekolah berperan dalam
Pegunungan Jaya Wijaya di Wamena menampung consensus umum tentang
dengan hutan menghijau disekitarnya pemberdayaan sekolah yang meyakini
memang memunculkan pemandangan bahwa untuk meningkatkan kualitas
amat cantik. pendidikan keputusan sedapat mungkin
dibuat oleh mereka yang ada digaris depan
Kalimat di atas merupakan perluasan dari yang bertanggung jawab secara langsung
kalimat inti … terhadap pelaksanaan kebijakan serta guru
(A) Deretan gunung memunculkan dan kepala sekolah yang terkena akibat
pemandangan amat cantik. kebijakan itu.
(B) Deretan gunung menghijau amat
cantik. Kalimat tersebut dikembangkan dari
(C) Hutan menghijau memunculkan kalimat inti …
pemandangan. (A) Otonomi sekolah berperan.
(D) Deretan gunung dalam kelompok (B) Otonomi sekolah menampung
Jayawijaya amat cantik. consensus.
(E) Pegunungan Jayawijaya menghijau (C) Otonomi sekolah meningkatkan
memunculkan penadangan kualitas.
(D) Otonomi sekolah dibuat.
52. Para mahasiswa melakukan proses turun (E) Otonomi sekolah bertanggung jawab.
ke jalan untuk menentang kebijakan
pemerintah menaikkan harga BBM. 55. Silang pendapat yang terjadi antara
Departemen Kesehatan dan Badan
Kalimat ini terbentuk dari perluasan Pengawasan Obat (POM) dalam
kalimat inti … penanganan khusus formalin secara tidak
(A) Pemerintah menaikkan harga BBM. langsung menggeser isu pokok yang
(B) Mahasiswa turun ke jalan. berpihak pada kepentingan konsumen,
(C) Mahasiswa menentang kebijakan yaitu mendapatkan produk makanan yang
pemerintah. aman bagi kesehatan.
(D) Mahasiswa melakukan protes.
(E) Mahasiswa menentang kenaikan BBM. Kalimat inti dari kalimat luas di atas adalah
….
53. Sekarang ini, hal penting yang sering (A) Silang pendapat terjadi.
dilupakan oleh para karyawan perusahaan (B) Silang pendapat menggeser isu pokok.
yang sedang dilanda kebangkrutan adalah (C) Kasus formalin berpihak pada
pencarian solusi yang tepat yang dapat kepentingan konsumen.
menguntungkan kedua belah pihak. (D) Silang pendapat berpihak pada
kepentingan konsumen.
Kalimat inti dari kalimat luas tersebut (E) Kepentingan konsumen, yaitu
adalah … mendapatkan produk makanan
(A) Yang sering dilupakan karyawan
adalah pencarian solusi.
(B) Hal penting adalah pencarian solusi.
(C) Karyawan perusahaan dilanda
kebangkrutan.
(D) Solusi yang tepat dapat
menguntungkan.
(E) Hal penting sering dilupakan.
KALIMAT EFEKTIF

Kalimat efektif adalah kalimat yang memenuhi (1) kelengkapan struktur, (2) kehematan dan
kecermatan, serta (3) kesejajaran. Soal-soal kalimat efektif sering muncul dalam tes seleksi masuk PTN.

A. Kelengkapan Struktur
Sebuah kalimat harus memiliki struktur yang lengkap, yaitu minimal terdapat subjek dan
predikat.
Contoh:
Para siswa yang mempersiapkan diri dalam UTBK. (tidak lengkap)
S
Para siswa mempersiapkan diri dalam UTBK. (lengkap)
S P O K

B. Kehematan dan Kecermatan


Kehematan merupakan salah satu syarat agar informasi yang ingin disampaikan dalam kalimat
tidak boros. Cara untuk menjaga kehematan kalimat, yaitu dengan cara menghindari kata
bersinonim, menghindari penggunakan gaya bahasa pleonasme (tambahan kata untuk
menguatkan sesuatu), dan menghindari pengulangan kata.
Contoh:
Aku tidak marah, hanya sedikit kecewa saja. (boros)
Aku tidak marah, hanya sedikit kecewa./Aku tidak marah, sedikit kecewa saja. (hemat)

Kamarnya ada di lantai dua sehingga kami perlu naik ke atas satu lantai. (boros)
Kamarnya ada di lantai dua sehingga kami perlu naik satu lantai. (hemat)
Kamarnya ada di lantai dua sehingga kami perlu ke atas satu lantai. (hemat)

Para siswa-siswi sudah berkumpul di aula untuk mengikuti sosialiasi TPS UTBK. (boros)
Para siswa sudah berkumpul di aula untuk mengikuti sosialiasi TPS UTBK. (hemat)
Siswa-siswi sudah berkumpul di aula untuk mengikuti sosialiasi TPS UTBK. (hemat)

Kecermatan mengikuti prinsip tepat dalam menyampaikan informasi, dapat dipahami, dan tidak
boleh multitafsir/ambigu.
Contoh:
Kucing makan tikus berlari. (ambigu)
Kucing makan, tikus berlari. (tidak ambigu)
Rumah seniman yang nyentrik itu akan dijual dengan harga murah. (ambigu)
Rumah seniman-yang-nyentrik itu akan dijual dengan harga murah. (tidak ambigu)

C. Kesejajaran
Kalimat yang mengikuti kesejajaran berprinsip pada keparalelan atau kesamaan antarunsur
kalimat, seperti penggunaan imbuhan yang sama dalam pembentukan kata atau menggunakan
imbuhan yang memiliki makna yang sama.
Contoh:
• Guru-guru yang dinyatakan lolos dalam seleksi ASN wajib mengikuti pendidikan dan
pelatihan.
• Tugas tersebut dilakukan dalam rangka untuk meningkatkan kompetensi, mendorong
kemandirian, dan melatih komunikasi.
1. Presiden membicarakan tentang serangan 4. Kalimat berikut yang strukturnya benar
Amerika terhadap Afghanistan dalam adalah …
sidang kabinet kemarin. (A) Meningkatkan kualitas pendidikan
melalui penyusunan standar
Kalimat tersebut akan menjadi kalimat kompetensi nasional berdasarkan
yang baku jika diperbaiki menjadi …
bidang-bidang keahlian.
(A) Presiden membicarakan serangan
(B) Membentuk jaringan komunikasi
Amerika terhadap Afghanistan dalam
sidang kabinet kemarin. antarguru sebagai wahana untuk
(B) Dalam sidang kabinet, Presiden meningkatkan proses pembelajaran.
membicarakan tentang serang (C) Peningkatan mutu kualitas guru
Amerika terhadap Afghanistan. dengan melaksanakan pendidikan dan
(C) Presiden membicarakan tentang pelatihan merupakan langkah penting
serangan Amerika terhadap untuk perbaikan mutu pendidikan.
Afghanistan. (D) Melalui peran serta masyarakat
(D) Presiden berbicara serangan Amerika menjadikan ciri konsep pendidikan
terhadap Afghanistan di depan sidang
pada era otonomi.
kabinet kemarin.
(E) Dengan mengembalikan pendidikan
(E) Presiden membicarakan tentang
serangan Amerika terhadap kepada masyarakat mengharapkan
Afghanistan kemarin di depan sidang akan memberi peluang kepada
kabinet. lembaga tersebut semakin
ditingkatkan perannya.
2. Sikap berbahasa yang positif dan kebiasaan
berbahasa Indonesia dengan baik dan 5. Pernyataan yang membentuk kalimat yang
benar perlu … dan … di kalangan baku adalah …
masyarakat. (A) Pada kesempatan itu menetapkan
berlakunya tarif baru di bidang
Pasangan kata yang sesuai untuk mengisi
komunikasi.
tempat kosong di atas adalah…
(B) Munculnya anggapan bahwa gejala
(A) ditingkatkan-dikembangkan
(B) meningkat-berkembang tersebut adalah gejala yang tidak
(C) meningkat-dikembangkan mudah diamati.
(D) dikembangkan-bertingkat (C) Perkembangan pedagang kaki lima
(E) dikembangkan-meningkat pada saat ini sulit mencari pekerjaan.
(D) Karena penyakit jenis ini memang
3. Sebagaimana telah ditetapkan dalam tidak mengenal status sosial ekonomi
GBHN, bahwa pembangunan pendidikan penderita.
dititikberatkan pada peningkatan setiap (E) Dalam menetapkan tarif dasar listrik,
jenjang dan jenis pendidikan.
pemerintah menggunakan berbagai
pertimbangan.
Kalimat di atas dapat diperbaiki dengan
menghilangkan kata ….
(A) sebagaimana
(B) telah
(C) dalam
(D) bahwa
(E) pada
6. Dalam rapat yang menghadiri para 9. Kalimat berikut yang termasuk kalimat
gubernur kemarin memutuskan bahwa baku adalah …
UMR baru dapat dilaksanakan karena (A) Menurut Yeo bahwa birokrasi di
beberapa pertimbangan. Indonesia terlalu rumit sehingga
menimbulkan banyak masalah bagi
Rangkaian kata di atas akan menjadi
investor yang mau bergabung.
kalimat baku jika dilakukan hal berikut …
(B) Pihak biro harga BBM berpengaruh
(A) Kata dalam dihilangkan dan kata
memutuskan diubah menjadi besar terhadap minat wisatawan untuk
diputuskan. melakukan kunjungan.
(B) Kata menghadiri diubah menjadi (C) Berdasarkan hasil survei menunjukkan
menghadirkan. bahwa sampai saat ini wisata pantai
(C) Kata memutuskan diganti disepakati. masih tetap diminati, terutama oleh
(D) Kata dalam dihilangkan, kata kaum muda.
menghadiri diubah menjadi dihadiri, (D) Menurut pengamatan lingkungan,
dan kata memutuskan diubah menjadi krisis air bersih dan masalah udara
diputuskan.
segar mulai merisaukan penduduk di
(E) Kata dalam dihilangkan dan kata
kota-kota besar.
menghadiri diganti dihadiri.
(E) Dengan adanya alam yang menelan
7. Dalam berwiraswasta memerlukan korban ratusan ribu nyawa manusia
kreativitas untuk menyiasati apa yang ada menggugah nurani dan sikap
dan melahirkan peluang bisnis. kedermawanan semua orang.

Kalimat tersebut menjadi benar apabila …. 10. Kalimat untuk mengembangkan Kota Batu
(A) kata memerlukan diubah menjadi sebagai kota pariwisata terkemuka di
diperlukan Indonesia mengandung makna
(B) kata dalam diganti dengan kata mendorong pertumbuhan dan perubahan
dengan
Kota Batu sebagai pusat kunjungan para
(C) kata menyiasati diubah menjadi
turis tidak baku.
mensiasati
(D) kata kreativitas ditulis kreatifitas
(E) kata untuk diubah menjadi karena Kalimat tersebut akan menjadi baku
apabila ….
8. Di bawah ini adalah kalimat-kalimat yang (A) kata untuk dihilangkan.
rancu, kecuali … (B) kata yang ditambahkan sebelum kata
(A) Untuk pembangunan gedung itu terkemuka.
menghabiskan biaya seratus juta (C) kata mengandung diganti dengan kata
rupiah. memiliki.
(B) Pelaksanaan pembangunan jembatan
(D) kata Kota Batu yang kedua diganti
dilaksanakan selama satu tahun.
dengan kata tersebut.
(C) Struktur Bahasa Indonesia berbeda
(E) kata bagi ditambahkan setelah kata
dengan struktur Bahasa asing.
(D) Kepada Bapak dan Ibu sekalian harap kunjungan.
mengambil snack dan minuman di
ruang istirahat.
(E) Dalam rapat itu membicarakan biaya
SPP.
11. Kalimat berikut yang termasuk kalimat (D) Karakter fisik manusia Indonesia
baku adalah … ditunjukkan warna kulit, lipatan
(A) Pada bagian pertama tulisan ini akan kelopak mata, dan rambut yang secara
menelusuri arti dan dimensi umum digolongkan ke dalam ras
mekanisme kekuasaan. Mongoloid dan Australomelanesid.
(B) Kalau dilihat pada zaman sekarang, (E) Karakter fisik manusia Indonesia
kolonialisme seperti zaman dahulu, menunjukkan warna kulit, lipatan
yakni menduduki wilayah secara fisik, kelopak mata, dan rambut secara
barangkali sudah tidak ada. umum digolongkan ke dalam ras
(C) Jika disajikan secara lebih rinci dan Mongoloid dan Australomelanesid.
lebih operasional, Bab III laporan
penelitian ini akan memberikan 13. Di antara kalimat berikut ini yang
informasi yang lengkap pada menggunakan ragam kalimat baku adalah
pembacanya. …
(D) Sebelum moderator memberikan (A) Garis keturunan dari pihak ibu masih
sulit diakomodasi dalam RUU
kesempatan kepada peserta seminar
Kewarganegaraan.
untuk bertanya, terlebih dahulu (B) Menginjak tahun 1970-an kemajuan
meminta pemakalah menyajikan ilmu dan teknologi di segala bidang
makalahnya. maju pesat dengan ditemukannya
(E) Subsidi BBM yang diberikan oleh microchip dimana merupakan
pemerintah itu untuk membantu komponen dasar komputer.
rakyat miskin agar dapat meringankan (C) Dari dialog itu tampak banyak sekali
kelemahan daripada Undang-Undang
beban perekonomian mereka.
Nomor 62 itu.
(D) Ketidakadilan dalam RUU itu terlihat
12. Dari karakter fisik manusia Indonesia terutama dalam ketentuan yang mana
menunjukkan warna kulit, lipatan kelopak mengatur perkawinan antar-bangsa.
mata, rambut yang secara umum (E) Banyak pengusaha nakal yang
digolongkan ke dalam ras Mongoloid dan melarikan diri keluar negeri dengan
Australomelanesid. membawa aset dari negara secara
melanggar hukum.
Kalimat tersebut menjadi baku jika diubah
14. Jumlah dana yang dialokasikan untuk
menjadi …
pemberdayaan masyarakat di tingkat
(A) Dari karakter fisik manusia Indonesia kelurahan cukup besar sehingga tidak
menunjukkan warna kulit, lipatan menutup kemungkinan peningkatan
kelopak mata, dan rambut yang secara kualitas lingkungan yang jauh lebih baik
umum digolongkan ke dalam ras daripada yang dicapai tahun lalu.
Mongoloid dan Australomelanesid.
(B) Dari karakter fisik manusia Indonesia Kalimat tersebut tidak efektif sehingga
perlu perbaikan dengan cara ….
ditunjukkan warna kulit, lipatan
(A) mengganti kata dialokasikan dengan
kelopak mata, dan rambut yang secara dianggarkan
umum digolongkan ke dalam ras (B) mengganti kata untuk dengan guna
Mongoloid dan Australomelanesid. (C) mengganti kata daripada dengan
(C) Karakter fisik manusia Indonesia dibandingkan
menunjukkan warna kulit, lipatan (D) mengganti kata cukup dengan jauh
kelopak mata, dan rambut yang secara (E) mengganti kata menutup dengan
umum digolongkan ke dalam ras tertutup
Mongoloid dan Australomelanesid.
15. Kalimat berikut ini tidak baku, kecuali… (C) Bank pemerintah Pakistan telah
(A) Khawatir cadangan air tanah tersedot memberikan izin bagi enam bank Islam
dan sulit mendapatkan air bersih, dan sebelas bank konvensional untuk
warga setempat menolak rencana mengembangkan sistem perbankan
pembuatan sumur dalam di sekitar Islam.
tempat tinggal mereka. (D) Saat ini, virus mempunyai varian
(B) Bidang lingkungan hidup, dana yang sasaran bukan hanya pada media
dialokasikan sebesar Rp300 juta yang penyimpanan, melainkan pada hampir
diarahkan pada kegiatan konservasi, semua hal yang berhubungan dengan
yaitu sumur resapan. komputer dan sistem di dalamnya
(C) Untuk mengetahui penyebab yang memungkinkan untuk diserang.
keracunan, sisa makanan sebagai (E) Dengan perkembangan internet dan
sampel telah dikirim ke laboratorium jaringan yang cukup pesat, virus
Dinas Kesehatan Jawa Tengah. dilengkapi kemampuan menyerang
(D) Bagi pengusaha yang belum menggaji melalui jaringan komputer sehingga
karyawannya sesuai dengan penyerangannya sangat cepat.
ketentuan, diwajibkan membuat surat
penangguhan ke gurbenur. 18. Pengendalian bahaya banjir yang
(E) Untuk larangan praktik pelayanan dilakukan pemerintah Indonesia tidak
kesehatan pasien bagi mantra akan berhasil kalau PERDA yang
kesehatan dan perawat sudah final. menaungi tidak dibuat. Dengan demikian
usaha itu hanya akan membuang uang sia-
16. Kalimat berikut yang seluruhnya ditulis sia. Pemerintah perlu segera membuat
sesuai dengan aturan EYD adalah … PERDA khususnya yang menyangkut
(A) Kehematan yaitu salah satu ciri kalimat pembagian tugas antarinstansi.
efektif. Paragraf tersebut akan menjadi paragraf
(B) Meskipun demikian, hal itu tidak baku jika ejaannya diperbaiki sebagai
berarti bahwa mereka bebas dari tugas. berikut, kecuali ….
(C) Jika informasinya jelas kalimat itu (A) huruf awal kata pemerintah ditulis
disebut kalimat efektif, karena dapat huruf kapital.
dipahami isinya. (B) kata PERDA ditulis dengan huruf
(D) Betapa pun panjangnya suatu kalimat kecil, kecuali huruf pertama.
yang penting adalah informasinya. (C) sebelum kata usaha itu diberi tanda
(E) Dalam kalimat efektif tidak ada ide koma (,).
kalimat yang dapat ditafsirkan secara (D) sebelum kata khususnya diberi tanda
ganda maknanya. koma (,).
(E) kata antarinstansi ditulis terpisah.
17. Kalimat berikut yang ditulis sesuai dengan
pedoman EYD, kecuali …
(A) Pakistan dan Malaysia akan bekerja
sama melakukan riset, pengembangan,
pelatihan, dan pendidikan keuangan
Islam.
(B) Tidak dapat dipungkiri bahwa
pendidikan multi kultural telah
meleburkan hegemoni pasar yang
dikuasai laki-laki ke dalam pangkuan
kaum perempuan.
19. Kalimat yang menggunakan ragam baku (C) Ketika diwawancarai media, Sarkozy
adalah … mengatakan bahwa peningkatan
(A) Dari hasil penelitian membuktikan kehadiran Prancis di Afghanistan
bahwa epilepsi bukan penyakit bukan kuantitatif, melainkan kualitatif.
menular. (D) Beberapa jam sesudah menerima
(B) Mengenai rancangan undang-undang Sarkozy, Karzai menerima kunjungan
itu belum disetujui anggota DPR. Rudd.
(C) Hari ini Indonesia akan (E) Berdasarkan laporan Dinas Pertanian
memberangkatkan tiga belas Jawa Tengah, setiap dua bulan sekali
mahasiswa untuk mengikuti dibuka produk pertanian Jawa Tengah.
olimpiade matematika di Prancis.
(D) Menurut penelitian Depbudpar 2005 22. Untuk kurangi pemanasan global maka
mengungkapkan orang Indonesia masyarakat diminta untuk meminimalisir
yang berlibur ke luar negeri rata-rata pemakaian kendaraan bermotor.
mengeluarkan dana 860 dolar AS
dalam setiap kunjungan. Kalimat di atas akan menjadi kalimat baku
(E) Karya sastra melukiskan hakikat yang apabila diperbaiki dengan cara berikut,
nyata dari objek sebagai titik tolak, kecuali ….
kemudian dengan perlahan bergerak (A) mengganti kata untuk pada awal
ke arah imajinasi dan perasaan. kalimat dengan dalam.
(B) mengubah kata kurangi menjadi
20. Dengan adanya bank syariah diharapkan mengurangi.
dapat mendukung perkembangan (C) menambahkan tanda koma (,) sesudah
ekonomi nasional, memfasilitasi segmen kata global.
pasar yang belum terjangkau atau tidak (D) menghilangkan kata maka.
terminat dengan bank konvensional, dan (E) mengubah kata meminimalisir
dapat memfasilitasi distribusi utilitas menjadi meminimalkan.
barang modal untuk kegiatan produksi
melalui skema sewa-menyewa. 23. Kalimat yang penulisannya mengikuti
EYD adalah …
Ketidakbakuan kalimat tersebut terletak (A) Sepuluh tahun yang lalu Keluarga
pada …. Satrio tinggal di jalan RE Martadinata,
(A) kurangnya tanda koma (,) sebelum No. 5.
kata diharapkan. (B) Sebagian besar dari kita beranggapan
(B) pemakaian kata dengan. bahwa kebudayaan Timur lebih halud
(C) gabungan kata bank syariah yang jika dibandingkan dengan kebudayaan
ditulis dengan huruf kecil. barat.
(D) pemakaian pengembangan yang (C) Ia mengakui ke dalaman wawasan
seharusnya perkembangan. tokoh masyarakat kampung Naga itu.
(E) pemakaian tanda koma (,) sebelum (D) Pengusaha besar itu menghadiahi atlit-
kata dan. atlit berprestasi internasional dengan
bea siswa untuk belajar di luar negeri.
21. Diantara kalimat-kalimat berikut, yang (E) Beberapa tokoh anti rasisme dari
bukan kalimat baku adalah … beberapa negara sepakat untuk
(A) Dengan pendidikan diharapkan dapat menanda tangani suatu kesepahaman.
membentuk manusia Indonesia yang
demokratis, berkeadilan, dan tidak
diskriminatif.
(B) Bahan ajar yang berperspektif gender
sebaiknya dimulai sejak dini, mulai
tingkat pendidikan dasar.
24. Penurunan cukai impor beras menjadi Rp. (E) Untuk keperluan pelestarian
450,- per kilogram, per 1 Januari 2008 lingkunga, diperlukan tanggung jawab
dimaksudkan agar Bulog dapat kolektif dan antisipasi dari para
mengendalikan harga beras. pengusaha yang bergerak di bidang
pengusahaan hutan.
Kalimat tersebut akan benar ejaannya
apabila …. 27. Di antara kalimat berikut, yang merupakan
(A) Rp. 450,- ditulis Rp450,- kalimat baku adalah …
(B) per kilogram ditulis perkilogram (A) Untuk membangun kampung wisata,
(C) per 1 Januari 2008 ditulis per-1 Januari maka masyarakat harus menjadi aktor
2008 kunci atau perkasa.
(D) Impor ditulis import (B) Untuk memaknai kehidupan,
(E) Sebelum kata agar ditambahkan tanda peradaban manusia selalui ditandai
koma (,) dengan perubahan.
(C) Klien mengajak Talia dan Francesca
25. Kalimat yang baku adalah … berlibur ke Panama, dimana
(A) Untuk mengatasi banjir Bengawan konglomerat itu memiliki villa.
Solo, penanaman pohon di sekitar (D) Bersahabat dengan teknologi di era
Waduk Gajah Mungkur dan sepanjang informasi, memang menjadi sebuah
bengawan tidak dapat ditawar lagi. tuntutan.
(B) Sedangkan penanganan jangka pendek (E) Musim penghujan harga sejumlah
dengan memperbaiki tanggul komoditi sayuran di beberapa daerah
sepanjang bengawan yang rusak. melambung.
(C) Juga dengan membuat bendungan
kecil di puluhan titik sepanjang 28. Kalimat yang merupakan kalimat baku
bengawan. adalah …
(D) Di sepanjang bengawan menemukan (A) Ternyata jamu tidak hanya terkenal di
lebih dari 20 tempat yang potensial Indonesia saja, tetapi di Negara-
untuk dibangun waduk. Negara lain pun mengkonsumsi jamu.
(E) Jika di tempat tersebut dibangun (B) Untuk Negara Asia di antarannya ada
waduk kecil-kecil, akan bermanfaat Cina dan Thailand, sedangkan untuk
mengurangi banjir. benua Eropa, jamu banyak ditemukan
di Eropa Barat dan Jerman.
26. Kalimat yang tidak baku adalah … (C) Sedangkan omzet produksi industri
(A) Sesuai dengan kesepakatan, jamu Indonesia di dunia berjumlah 300
pertunjukan itu akan diadakan di Jawa juta dollar AS.
dan Sumatera. (D) Penurunan nilai ekspor terjadi karena
(B) Masa yang akan datang kenaikan yang adanya isu bahwa pada obat-obatan
tidak terkendali dari harga kebutuhan herbal terdapat kandungan bahan
sehari-hari tidak boleh terjadi lagi. kimia berbahaya.
(C) Ramalan BMKG semakin kita (E) Dan masalah kwalitas jamu yang
perhatikan, terutama setelah muncul mengandung campuran kimia kini
berbagai dampak tak terduga dari dipermasalahkan oleh Negara importir
perubahan iklim. karena ada yang meninggal setelah
(D) Di negara-negara bermusim empat, mengkonsumsi jamu tersebut.
ramalan cuaca dan geofisika sudah
lama menjadi bagian dari kehidupan
sehari-hari.
29. Kalimat berikut tidak baku, kecuali … 31. Menulis berarti menterapkan semua
(A) Dengan teknik tersebut para astronom pengetahuan yang dimilikinya. Dalam
dapat memperkirakan ukuran planet menulis, banyak proses yang harus
meskipun belum diketahui massanya, diperhatikan; diantaranya adalah
namun diperkirakan antara 5,7 hingga pemakaian ejaan sesuai dengan kaidah
11 massa bumi. bahasa yang dianut, pemakaian struktur
(B) Landasan pacu bandara Ahmad Yani bahasa yang benar, dan pemakaian diksi
Semarang tergenang sepanjang 500 yang mengungkapkan fikiran.
meter dan harus menggunakan 23
pompa air untuk menyurutkan Dalam paragraf tersebut terdapat
genangan. kesalahan tata tulis. Kesalahan yang
(C) Bandara menunda semua dimaksud adalah sebagai berikut, kecuali
penerbangan sejak Minggu pagi ….
hingga pukul 21.00 karena kondisi (A) kesalahan penulisan menterapkan
landasan pacu masih buruk. (B) kesalahan penulisan diantaranya
(D) Komisi pemilihan umum berencana (C) kesalahan tanda koma (,) sebelum dan
mengatur penetapan calon terpilih (D) kesalahan tanda titik koma (;) setelah
yang berpihak pada calon perempuan, diperhatikan
jika partai politik meraih tiga kursi (E) kesalahan penulisan fikiran alih-alih
DPR/DPRD, salah satunya mesti pikiran
diberikan kepada calon perempuan
yang perolehan suaranya terbanyak. 32. Memanfaatkan keberhasilan di berbagai
(E) Berdasarkan data di Dinas Bina Marga bidang kehidupan serta kekuatannya di
Pemkot Surabaya, maka titik-titik bidang fisik dan militer, maka budaya
genangan itu menyebar hampir di Barat cukup mendominasi dunia dan umat
seluruh wilayah Surabaya mulai sisi manusia.
paling barat, timur, hingga selatan.
Kalimat mana yang merupakan hasil
30. Yang merupakan kalimat baku adalah … perbaikan kalimat di atas?
(A) Buku ini terdapat istilah-istilah (A) Memanfaatkan keberhasilan di
kekerabatan pada orang Jawa dan berbagai bidang kehidupan serta
Batak dalam susunan masyarakat yang kekuatannya di bidang fisik dan
ditulis berdasarkan ilmu antropologi. militer, budaya Barat cukup
(B) Dari beberapa pokok persoalan yang mendominasi dunia dan umat
diberikan untuk membandingkan dua manusia.
atau lebih dialek, antara lain dalam (B) Untuk memanfaatkan keberhasilan di
bidang fonetik atau semantik. berbagai bidang kehidupan serta
(C) Adapun yang akan saya uraikan di sini kekuatannya di bidang fisik dan
ialah kebersihan dan kesehatan, saya militer, budaya Barat cukup
terdorong untuk mengemukakannya mendominasi dunia dan umat
karena sering dilalaikan orang dan manusia.
sesungguhnya kebersihan dan (C) Karena memanfaatkan keberhasilan di
kesehatan itu perlu, sebab dengan berbagai bidang kehidupan serta
bersih tentu akan menjadi sehat. kekuatannya di bidang fisik dan
(D) Karena nilai yang didapatkan lebih militer, maka budaya Barat cukup
besar dari penolakan maka hipotesis mendominasi dunia dan umat
nihil ditolak. manusia.
(E) Jika data yang diolah tidak memenuhi (D) Jika memanfaatkan keberhasilan di
syarat, baik dari segi kuantitas berbagai bidang kehidupan serta
maupun kualitasnya, hasil penelitian kekuatannya di bidang fisik dan
di lapangan itu tidak dapat militer, maka budaya Barat cukup
dipertanggungjawabkan secara ilmiah. mendominasi dunia dan umat
manusia.
(E) Dengan memanfaatkan keberhasilan di
berbagai bidang kehidupan serta
kekuatannya di bidang fisik dan
militer, budaya Barat cukup
mendominasi dunia dan umat
manusia.
33. Karena Instagram sangat mengedepankan 35. Oleh karena itu, dengan keragaman bahasa
visual, ditambah dengan fitur-fitur edit, daerah di suatu wilayah menjadikan posisi
variasi dalam mempresentasikan para Bahasa Indonesia sebagai bahasa
penggunanya bisa memilih dirinya di pergaulan semakin penting.
media sosial.
Kalimat mana yang merupakan hasil
Dari kalimat di atas “Karena Instagram perbaikan kalimat-kalimat di atas?
sangat mengedepankan visual, ditambah (A) Oleh karena itu, keragaman bahasa
dengan fitur-fitur edit, ….” Kutipan
daerah di suatu wilayah menjadikan
tersebut menjadi kalimat logis jika
posisi Bahasa Indonesia sebagai bahasa
dilanjutkan dengan ….
pergaulan semakin penting.
(A) para penggunanya bisa
merepresentasikan dirinya dengan (B) Oleh karena itu, menjadikan posisi
variasi di media sosial Bahasa Indonesia sebagai bahasa
(B) para penggunanya bisa memilih pergaulan bagi keragaman bahasa
variasi dalam mempresentasikan daerah di suatu wilayah semakin
dirinya di media sosial penting.
(C) variasi dalam merepresentasikan (C) Oleh karena itu, menjadikan posisi
dirinya di media sosial bisa dipilih oleh Bahasa Indonesia sebagai bahasa
para penggunanya pergaulan sehingga semakin penting
(D) dipilih variasi dalam posisi bahasa daerah di suatu wilayah.
mempresentasikan para penggunanya (D) Oleh karena itu, dengan keragaman
di media sosial bahasa daerah, maka Bahasa Indonesia
(E) di media sosial para penggunanya bisa semakin penting sebagai bahasa
merepresentasikan variasi yang dipilih pergaulan di suatu wilayah.
oleh dirinya (E) Oleh karena itu, keragaman bahasa
daerah di suatu wilayah menjadikan
34. Hal ini disebabkan oleh berbagai hal, pentingnya posisi Bahasa Indonesia
seperti penurunan kerapatan bebatuan di
sebagai bahasa pergaulan.
atas ruang magma atau mencairnya es di
puncak sebuah gunung berapi.
36. Untuk membuat kawasan hijau terjangkau
Dalam kalimat di atas, frasa seperti bagi semua kalangan, terlebih masa
penurunan kerapatan bebatuan di atas pandemi, perlu diusahakan pembuatan
ruang magma atau mencairnya es di kawasan hijau oleh pihak pemerintah.
puncak sebuah gunung berapi akan
menjadi kesejajaran bentuk jika diperbaiki Perhatikan kutipan berikut “Untuk
menjadi … membuat kawasan hijau terjangkau bagi
(A) Seperti turunannya kerapatan semua kalangan, terlebih masa pandemi,
bebatuan di atas ruang magma atau ….” Kutipan tersebut menjadi kalimat logis
pencairannya es di puncak sebuah jika dilanjutkan dengan ….
gunung berapi. (A) pihak pemerintah diperlukan untuk
(B) Seperti turunnya kerapatan bebatuan pengusahaan kawasan hijau
di atas ruang magma atau mencairnya (B) perlunya pengusahaan kawasan hijau
es di puncak sebuah gunung berapi. oleh pihak pemerintah
(C) Seperti menurunnya kerapatan (C) kawasan hijau diperlukan untuk
bebatuan di atas ruang magma atau diusahakan oleh pemerintah
pencairan es di puncak sebuah gunung
(D) pihak pemerintah perlu
berapi.
mengusahakan pembuatan kawasan
(D) Seperti kerapatan bebatuan yang
hijau
menurun di atas ruang magma atau
mencairnya es di puncak sebuah (E) pembuatan kawasan hijau perlu
gunung berapi. diusahakan oleh pihak pemerintah
(E) Seperti menurunnya kerapatan
bebatuan di atas ruang magma atau
mencairnya es di puncak sebuah
gunung berapi.
37. Di sisi lain, peneliti independen, pengamat (D) gangguan jangka pendek dari kimia
lingkungan, dan konsumen laut berdampak pada organisme yang
mengkhawatirkan risiko baru dari pangan ditemukan oleh ilmuwan
PRG, seperti menimbulkan alergi (E) ilmuwan menemukan bahwa
makanan, kenaikan resistansi antibiotik, organisme laut terdampak oleh
dan dampak kesehatan manusia yang tidak gangguan jangka pendek dari kimia
diinginkan lainnya. laut

Berdasarkan kalimat di atas “…, seperti 39. “Kelenjar tiroid juga meningkatkan
menimbulkan alergi makanan, kenaikan metabolisme tubuh dan membantu
resistansi antibiotik, dan dampak pertumbuhan tulang dan saraf serta panas
kesehatan manusia yang tidak diinginkan juga dihasilkannya bagi tubuh,” kata dia.
lainnya" akan menjadi kesejajaran bentuk
jika diperbaiki menjadi … Berdasarkan kalimat di atas, rasa panas
(A) Seperti menimbulkan alergi makanan, juga dihasilkannya bagi tubuh akan
menaikkan resisteansi antibiotik, dan memiliki kesejajaran bentuk dengan frasa
menimbulkan dampak kesehatan yang lain jika diperbaiki menjadi ….
manusia yang tidak diinginkan (A) dihasilkan juga panas bagi tubuh
lainnya. (B) bagi tubuh panas juga dihasilkan
(B) Seperti timbulnya alergi makanan, (C) menghasilkan panas bagi tubuh
kenaikan resisteansi antibiotik, dan (D) juga penghasil panas bagi tubuh
menimbulkan dampak kesehatan (E) Hasilkan juga panas bagi tubuh
manusia yang tidak diinginkan
lainnya. 40. Untuk kecintaan kepada wayang dan
(C) Seperti timbulkan alergi makanan, karakternya kepada anak sebaiknya
naikkan resistensi antibiotik, dan ditanamkan sejak dini.
terdampak kesehatan manusia yang
tidak diinginkan lainnya. Kalimat mana yang merupakan hasil
(D) Seperti menimbulkan alergi makanan, perbaikan kalimat di atas?
naiknya resisteansi antibiotik, dan (A) Untuk kecintaan kepada wayang dan
dampak kesehatan manusia yang tidak karakternya sejak dini sebaiknya
diinginkan lainnya. ditanamkan sejak dini.
(E) Seperti menimbulkan alergi makanan, (B) Bentuk kecintaan kepada wayang dan
kenaikan resisteansi antibiotik, dan karakternya sejak dini sebaiknya
berdampak kesehatan manusia yang ditanamkan kepada anak.
tidak diinginkan lainnya. (C) Kecintaan kepada wayang dan
karakternya sejak dini sebaiknya
38. Dengan mempelajari keseimbangan itu, ditanamkan kepada anak.
dampak gangguan jangka pendek dari (D) Untuk mencintai wayang dan
kimia laut terhadap organisme ditemukan karakternya sebaiknya ditanamkan
oleh ilmuwan. sejak diri kepada anak-anak.
(E) Kecintaan anak-anak kepada wayang
Perhatikan kutipan berikut “Dengan dan karakternya sebaiknya
mempelajari keseimbangan itu, ….” ditanamkan sejak dini.
Kutipan tersebut menjadi kalimat logis jika
dilanjutkan dengan ….
(A) kimia laut organisme yang ditemukan
ilmuwan terdampak oleh gangguan
jangka pendek
(B) dampak gangguan jangka pendek
terhadap organisme kimia laut
ditemukan oleh ilmuwan
(C) organisme kimia laut yang ditemukan
oleh ilmuwan terkena dampak
gangguan jangka pendek
PARAGRAF

Paragraf merupakan satu kesatuan kalimat yang membahas satu ide pokok. Paragraf akan membentuk
wacana atau bacaan yang utuh. Dalam tes seleksi masuk PTN, soal-soal yang berhubungan dengan
paragraf dan wacana seperti (1) ide pokok, (2) simpulan, (3) kepaduan paragraf, (4) hubungan
antarparagraf, dan (5) posisi penulis dalam wacana.

A. Ide Pokok
Ide pokok disebut juga gagasan utama/pikiran utama/gagasan pokok/pikiran pokok/inti
paragraf. Ide pokok adalah masalah utama yang dibahas dalam paragraf.
Contoh:
Walaupun khasiat susu bagi tubuh sudah tidak diragukan lagi, ternyata tidak setiap orang
bersedia mengonsumsi susu dengan baik. Hal ini disebabkan oleh dua kemungkinan, yaitu sifat
yang terkandung dalam susu yang tidak disukai orang atau sifat biologis orang yang
bersangkutan. Penyebab pertama dapat diatasi dengan penambahan sari jeruk, markisa, apel, atau
lainnya sehingga dapat menghilangkan rasa asli susu yang memualkan. Sementara itu, penyebab
kedua dapat diatasi dengan mengganti air susu dengan olahan susu yang telah mengalami
perlakuan khusus, yaitu fermentasi, seperti keju atau yoghurt .

B. Simpulan
Simpulan merupakan pendapat akhir dari sebuah wacana.
Contoh:
Walaupun khasiat susu bagi tubuh sudah tidak diragukan lagi, ternyata tidak setiap orang
bersedia mengonsumsi susu dengan baik. Hal ini disebabkan oleh dua kemungkinan, yaitu sifat
yang terkandung dalam susu yang tidak disukai orang atau sifat biologis orang yang
bersangkutan. Penyebab pertama dapat diatasi dengan penambahan sari jeruk, markisa, apel, atau
lainnya sehingga dapat menghilangkan rasa asli susu yang memualkan. Sementara itu, penyebab
kedua dapat diatasi dengan mengganti air susu dengan olahan susu yang telah mengalami
perlakuan khusus, yaitu fermentasi, seperti keju atau yoghurt .

Simpulan:
Tidak setiap orang bersedia mengonsumsi susu karena kandungan dalam susu yang tidak disukai
atau sifat biologis pengonsumsi susu.

C. Kepaduan Paragraf
Kepaduan paragraf adalah kekompakan hubungan antara kalimat yang satu dengan yang kalimat
lainnya pada paragraf.
Contoh paragraf tidak padu:
Seperti yang telah diketahui banyak orang saat ini bahwa daun kelor memiliki banyak manfaat.
Salah satu manfaatnya adalah sebagai bahan sayur, karena daun ini memiliki kandungan vitamin
dan mineral yang sangat tinggi. Manfaat yang lain bisa digunakan sebagai bahan obat-obatan
terhadap penyakit yang ada di dalam tubuh. Akarnya juga memiliki nilai ekonomis yang tinggi.
Selain itu, daun kelor juga bermanfat untuk memercantik kulit.

Contoh paragraf padu:


Seperti yang telah diketahui banyak orang saat ini bahwa daun kelor memiliki banyak manfaat.
Salah satu manfaatnya adalah sebagai bahan sayur, karena daun ini memiliki kandungan vitamin
dan mineral yang sangat tinggi. Manfaat yang lain bisa digunakan sebagai bahan obat-obatan
terhadap penyakit yang ada di dalam tubuh. Selain itu, daun kelor juga bermanfat untuk
memercantik kulit.

D. Hubungan Antarparagraf
Hubungan antarparagraf adalah keterkaitan paragraf satu dengan paragraf lainnya secara logis.
Jenis-jenis hubungan antarparagraf antara lain hubungan (a) perincian, (b) contoh, (c) pemaparan
atau penjelasan, (d) akibat, (e) penambahan, (f) simpulan, dan (g) pertentangan.
(a) Perincian
Paragraf sesudahnya merupakan perincian dari hal yang disampaikan pada paragraf
sebelumnya.
Contoh:
Komisi Pemberantasan Korupsi (KPK) pada tahun 2018 mencatat bahwa jumlah penindakan
kasus korupsi mengalami penurunan. Penurunan tersebut terjadi mulai dari jumlah
penyelidikan, penyidikan, penuntutan, inkrah (berkekuatan hukum tetap), dan eksekusi.

Jumlah penyelidikan mengalami penurunan sebanyak 38,2 persen, dari 123 kasus di 2017
menjadi 76 kasus di 2018. Angka penyidikan turun 29,8 persen, dari 121 kasus menjadi 85
kasus. Jumlah penuntutan mengalami penurunan hingga 51,5 persen. Selain itu, tahap inkrah
juga turun 44,1 persen, dari jumlah 84 kasus menjadi 47 kasus.

(b) Contoh
Paragraf sesudahnya merupakan contoh dari hal yang disampaikan pada paragraf
sebelumnya.
Contoh:
Ricky selalu nampak enerjik dan terkadang iseng. Apalagi di saat pria yang memiliki satu
anak laki-laki ini mengeluarkan jurus gombalan ala Indonesia untuk menggoda tamu
perempuan. Bak gayung bersambut, perempuan-perempuan ini terlihat sangat menikmati
jurus gombalan Pakde Ricky.

Contohnya saja ketika pria berusia 48 tahun ini nekat menggoda seorang anak perempuan
yang tengah makan malam bersama ayah dan ibunya. “Hey pretty girl, what’s your name?”
ucap Ricky mengawali rayuan mautnya. “Paetyn,” jawab perempuan itu nampak malu-malu.

(c) Pemaparan atau Penjelasan


Paragraf sesudahnya merupakan pemaparan atau penjelasan lebih lanjut dari hal yang
disampaikan pada paragraf sebelumnya.
Contoh:
Merayakan Hari Batik Nasional pada 2 Oktober 2022, ibu negara Iriana Jokowi akan
berkebaya bersama ribuan wanita. Istri Presiden Jokowi itu dan para wanita berkebaya akan
memecahkan rekor MURI sekaligus mendukung gerakan Kebaya Goes to UNESCO.

Kebaya Goes to UNESCO merupakan gerakan untuk mendukung pendaftaran kebaya


sebagai warisan budaya tak benda versi Organisasi Pendidikan, Keilmuan, dan Kebudayaan
yang berada di bawah naungan Perserikatan Bangsa-Bangsa (PBB) itu. Kebaya hendak
menyusul batik yang sudah mendapat pengakuan UNESCO sejak 2 Oktober 2009.
(d) Akibat
Paragraf sesudahnya merupakan akibat dari hal yang disampaikan pada paragraf
sebelumnya.
Contoh:
Pemerintah melalui Kementerian Perhubungan menetapkan tarif baru ojek online (ojol)
untuk semua zona. Ketetapan tersebut berdasarkan Keputusan Menteri Perhubungan
(Kepmenhub) Nomor KP 564 Tahun 2022.

Aturan ini muncul untuk menyesuaikan dengan kenaikan harga BBM dan inflasi. Namun
para pengemudi ojol tetap melakukan protes karena tipisnya keuntungan dan potongan
biaya oleh aplikator yang melebihi batas dari Kepmenhub.

(e) Penambahan
Paragraf sesudahnya merupakan penambahan dari hal yang disampaikan pada paragraf
sebelumnya.
Contoh:
Proses transfer kekuasaan dari Ratu Elizabeth II yang wafat kepada Raja Charles III berjalan
damai. Namun, masih ada beberapa faktor yang diperkirakan mengganjal pamor sang raja.
Kendala ini mulai dari insiden meninggalnya Putri Diana atau Lady Diana akibat kecelakaan
mobil hingga keberadaan Camilla Parker Bowles sebagai istri baru Raja Charles III pada 9
April 2005 silam.

Tidak hanya itu, Raja Charles III juga memiliki dua pekerjaan rumah yang perlu untuk
dirampungkan. Salah satunya adalah kemampuan Raja Charles III untuk meyakinkan
negara-negara persemakmuran bahwa mereka akan lebih sejahtera di bawah Britania Raya.

(f) Simpulan
Paragraf sesudahnya merupakan simpulan dari hal yang disampaikan pada paragraf
sebelumnya.
Contoh:
Pasal 20 Permendikbudristek Nomor 48 Tahun 2022 menyatakan bahwa seleksi nasional
berdasarkan prestasi dan seleksi nasional berdasarkan tes diselenggarakan oleh Kementerian
bekerja sama dengan PTN.

Dengan demikian, segala urusan terkait seleksi nasional penerimaan mahasiswa baru telah
beralih menjadi wewenang Kementerian, tidak lagi diselenggarakan oleh LTMPT.

(g) Pertentangan
Paragraf sesudahnya merupakan pertentangan dari hal yang disampaikan pada paragraf
sebelumnya.
Contoh:
Ahmad Yani menjelaskan tidak benar ada Dewan Jenderal. Yani mengakui telah menyuruh
Parman dan Brigjen Ahmad Sukendro menjalin relasi dengan Kedubes Inggris dan AS, tapi
bukan untuk tujuan kudeta. Saat itu, Yani mengakui Dewan Jenderal yang dimaksudkan tak
lain adalah Dewan Kepangkatan dan Jabatan Tinggi (Wanjakti). Lembaga itu bertugas
menilai dan mempromosikan perwira militer ke jabatan yang lebih tinggi.
Meskipun telah disanggah Yani, kabar seputar Dewan Jenderal terus berembus. Sukarno tak
100 persen percaya atas keterangan Yani. Apalagi Yani mulai menunjukkan tanda-tanda
ketidakpatuhan dan ketidaksetujuan atas rencana pembentukan Angkatan Kelima. Sukarno
juga melihat tindakan tegas AD dalam menghambat perkembangan PKI yang membuatnya
berpikir untuk mencopot Yani dari jabatannya sebagai Panglima AD.

E. Posisi Penulis dalam Wacana


Posisi penulis dalam wacana dapat tercerminkan dari tujuan penulis dan keberpihakan penulis.
Tujuan penulis merupakan maksud yang ingin dicapai oleh penulis saat menulis sebuah wacana.
Tujuan penulis dalam sebuah wacana untuk menjelaskan, menginformasikan, meyakinkan, dan
menyampaikan kepada pembaca mengenai sesuatu, baik berupa fakta, pendapat, ataupun
peristiwa/cerita. Tujuan penulis dapat terlihat dari isi wacana.
Contoh:
Sebanyak 127 orang tewas dalam kerusuhan di Stadion Kanjuruhan, Malang, usai laga Arema FC
dan Persebaya. Sedangkan 180 lainnya kini dalam perawatan di rumah sakit.

Kapolda Jawa Timur (Jatim) Irjen Nico Afinta mengungkap penyebab para korban meninggal
dunia. Menurutnya, tragedi maut itu terjadi karena penumpukan massa.

Berdasarkan kutipan wacana di atas, menunjukkan bahwa penulis bertujuan untuk


menginformasikan peristiwa dalam kerusuhan di Stadion Kanjuruhan.

Keberpihakan penulis merupakan kecenderungan penulis dalam memberikan dukungan kepada


pihak atau hal yang sedang dibahas dalam teks.
Contoh:
Sebanyak 23 koruptor mendapatkan hadiah pembebasan bersyarat pada Selasa, 6 September 2022.
Kerugian negara yang para narapidana korupsi ini timbulkan mulai Rp 1 miliar hingga Rp 202
miliar. Sebagian besar merupakan mantan pejabat negara yang telah menjalani masa tahanan 4-9
tahun penjara.

Sebelumnya, mereka dikurung di Lapas Kelas I Sukamiskin, Bandung, Jawa Barat, dan Kelas II-A
Tangerang, Banten. Kementerian Hukum dan Hak Asasi Manusia menyebut pembebasan itu
sesuai dengan Undang-Undang Nomor 22 Tahun 2022 tentang Pemasyarakatan serta
Permenkumham Nomor 7 Tahun 2022 tentang Syarat dan Tata Cara Pemberian Remisi, Asimilasi,
Cuti Mengunjungi Keluarga, Pembebasan Bersyarat, Cuti Menjelang Bebas, dan Cuti Bersyarat.

Berdasarkan kutipan wacana di atas, menunjukkan bahwa penulis berpihak pada negara.
(1) Pendidikan karakter dijadikan 3. Tujuan penulis menghasilkan tulisan
landasan untuk mewujudkan visi tersebut adalah ….
pembangunan nasional. (2) Upaya (A) menjelaskan cara melaksanakan
pembentukan karakter sesuai dengan budaya pendidikan karakter
bangsa tidak hanya dilakukan pada kegiatan (B) menjelaskan karakteristik pendidikan
intra-kurikuler di sekolah, tetapi juga melalui
karakter
pembiasaan dalam kehidupan di masyarakat.
(C) menjelaskan pengertian pendidikan
(3) Melalui pembiasaan, siswa tidak hanya
mempelajari pengetahuan tentang hal-hal yang karakter
benar dan salah, tetapi juga mampu merasakan (D) menjelaskan pentingnya pendidikan
nilai yang baik dan tidak baik. (4) Oleh karena karakter di sekolah
itu, sekolah dan masyarakat memiliki peranan (E) menjelaskan pendidikan karakter yang
yang benar dalam pendidikan karakter. kreatif dan inovatif
(5) Ketika sekolah mulai menerapkan
nilai-nilai karakter tertentu pada siswa, setiap 4. (1) Di dunia ditemukan dampak
nilai yang harus selalu disampaikan guru mengerikan akibat pencemaran sampah di
melalui pembelajaran langsung atau disatukan
laut. (2) Contohnya adalah penyu yang
ke dalam mata pelajaran. (6) Guru juga harus
terjerat plastik, anjing laut yang terlilit tali,
kreatif dan inovatif dalam menyampaikan
materi pembelajaran. (7) Hal demikian akan dan burung albartos yang mati karena
membuat siswa lebih senang dalam belajar. (8) sistem pencernaan yang rusak. (3) Di
Siswa membuat suasana belajar yang sesuai. (9) Indonesia pun laut rusak. (4) Salah satunya
Para siswa yang bosan dalam belajar akan adalah laut di Manta Point, Nusa Penida,
mengubah paradigma mereka ketika belajar Bali. (5) Laut itu sangat kotor sehingga
dengan kreativitas yang diberikan oleh huruf. yang terlihat bukan terumbu karang
namun sampah-sampah yang berenang
1. Kata demikian pada kalimat (7) mengacu …. bersama ikan-ikan.
(A) nilai yang disampaikan guru pada
siswa
Kalimat manakah yang paling tepat
(B) nilai-nilai karakter
sebagai perluasan kalimat (3) agar padu
(C) penyampaian materi pembelajaran
(D) kreativitas dan inovasi guru dengan kalimat yang lain?
(E) suasana belajar yang inovatif (A) Di Indonesia pun laut yang rusak
makin meluas.
2. Anda perlu menambahkan kalimat berikut (B) Di Indonesia pun laut yang rusak
dalam paragraf kedua. akibat bau busuk sampah.
Contohnya, guru biologi yang menyampaikan (C) Di Indonesia pun laut yang rusak
materi asal-usul makhluk hidup dapat sehingga airnya keruh.
menceritakan asal-usul manusia dari Adam
(D) Di Indonesia pun laut yang rusak
dan keberadaan Tuhan.
tercemar bertambah rusak.
Kalimat tersebut paling tepat ditempatkan (E) Di Indonesia pun laut yang rusak
setelah …. karena tercemar sampah.
(A) kalimat (6)
(B) kalimat (7)
(C) kalimat (8)
(D) kalimat (9)
(E) kalimat (10)
5. (1) Diabetes melitus merupakan penyakit (8) Pada saat ini sikap budaya gotong-
yang banyak dijumpai di negara kita. (2) royong yang semula menjadi sikap hidup
Jumlah penyandangnya makin meningkat. bangsa telah mengalami banyak gempuran. (9)
(3) Peningkatan jumlah itu disebabkan oleh Gempuran itu terutama bersumber pada
perubahan gaya hidup dan perkembangan budaya barat yang agresif, dinamis, dan bebas.
teknologi kedokteran sehingga penyakit (10) Memanfaatkan keberhasilan di berbagai
itu mudah didiagnosis. (4) Penyandang bidang kehidupan serta kekuatannya di bidang
diabetes harus berusaha mengendalikan fisik dan militer, maka budaya Barat cukup
gula darah dan mengamalkan gaya hidup mendominasi dunia dan umat manusia. (11)
sehat. Arus globalisasi dalam bidang budaya ini
sangat cepat merasuk ke dalam masyarakat,
Mengapa pada paragraf pertama perlu terutama kalangan muda. (12) Dampak
ditambahkan kalimat? globalisasi ini telah memengaruhi hampir
(A) Jumlah penderita diabetes melitus di semua aspek kehidupan yang ada di
negara kita pada kalimat (1) perlu masyarakat. (13) Salah satu aspek kehidupan
disebutkan. yang terpengaruh adalah aspek budaya
(B) Persentase peningkatan jumlah gotong-royong.
penyandang diabetes melitus perlu (Diadaptasi dari https://majalah1000guru.net)

disebutkan setelah kalimat (2).


(C) Kemudian diagnosis penyakit diabetes 6. Apa judul yang paling tepat untuk teks
melitus pada kalimat (3) perlu tersebut?
diuraikan secara jelas. (A) Intervensi Budaya Barat
(D) Diperlukan kalimat penjelas yang (B) Hakikat Budaya Gotong Royong
mengaitkan gaya hidup dan (C) Tergerusnya Budaya Asli Indonesia
pengendalian gula darah setelah (D) Memudarnya Budaya Gotong Royong
kalimat (3). (E) Lunturnya Budaya Asli Indonesia Saat
(E) Gaya hidup sehat pada kalimat (4) Ini
perlu diberi contoh secara konkret agar
kalimat menjadi jelas. 7. Kalimat yang menyebabkan
ketidakpaduan ide paragraf pertama
(1) Gotong royong yang dilakukan adalah ….
masyarakat Indonesia di masa lalu telah (A) kalimat (2)
memberikan banyak manfaat. (2) Melakukan (B) kalimat (3)
setiap pekerjaan secara bergotong-royong (C) kalimat (4)
dapat meringankan dan mempercepat (D) kalimat (7)
penyelesaian pekerjaan. (3) Dengan bergotong (E) kalimat (8)
royong, rasa persatuan dan kesatuan menjadi
semakin erat. (4) Gotong royong pun dapat
menghemat pengeluaran kegiatan. (5)
Sayangnya, pada zaman modern ini penerapan
nilai-nilai gotong royong memudar. (6) Orang-
orang sudah memikirkan kebutuhan mereka
sendiri tanpa memperhatikan lingkungan
sekitar. (7) Setiap manusia merupakan
makhluk sosial yang selalu membutuhkan
bantuan orang lain.
(1) Berdasarkan laporan UNESCO yang 9. Kalimat (4) pada bacaan di atas mewadahi
berjudul “The Social and Economic Impact of maksud penulis untuk ….
Illiteracy” yang dirilis pada tahun 2010, tingkat (A) merangkum pendapat penulis
literasi rendah mengakibatkan kehilangan atau mengenai tingkat literasi yang rendah
penurunan produktivitas, tingginya beban (B) memberikan contoh mengenai
biaya kesehatan, kehilangan proses pendidikan penyebab rendahnya tingkat literasi
baik pada tingkat individu maupun pada (C) memberikan opini yang berseberangan
tingkat sosial dan terbatasnya hak advokasi mengenai tingkat literasi yang rendah
akibat rendahnya partisipasi sosial dan politik. (D) mengemukakan dampak dari
(2) Literasi rendah juga menurut UNESCO, rendahnya literasi seseorang terhadap
menimbulkan dampak antara. Misalnya, kesehatan
tingginya kecelakaan kerja dan tingginya (E) menawarkan solusi mengenai
prevalensi sakit akibat pekerjaan. (3) Dampak rendahnya literasi masyarakat
antara literasi rendah juga muncul dalam
persoalan kesehatan masyarakat, karena (1) Generasi milenial atau orang yang
masyarakat dengan literasi rendah juga lahir pada tahun 1980 sampai 2000 adalah salah
umumnya memiliki kesadaran rendah akan satu yang terdampak oleh pandemi COVID-19.
kebersihan makanan dan gizi buruk dan (2) Generasi milenial bahkan bisa mendapat
memiliki perilaku seksual berisiko tinggi. (4) tantangan yang lebih berat mengingat status
Akibatnya, prevalensi penyakit seksual, mereka sebagai generasi sandwich yang baru.
kehamilan, aborsi, kelahiran, kematian menjadi (3) Padahal di tengah kondisi pandemi
tinggi. (5) Literasi rendah juga berdampak pada yang serba tidak menentu ini, banyak
tingginya angka putus sekolah dan perusahaan yang harus memotong gaji bahkan
pengangguran yang berdampak pada merumahkan karyawannya, serta tak sedikit
rendahnya kepercayaan diri. (6) Orang dengan Usaha Mikro Kecil, dan Menengah (UMKM)
tingkat literasi rendah sulit menjadi mandiri yang merugi karena minimnya transaksi jual
atau berdaya, dan tergantung secara ekonomi beli. (4) Generasi sandwich harus
pada keluarga, kerabat, dan negara. (7) mengeluarkan duit lebih tidak hanya untuk
Kriminalitas, penyalahgunaan obat dan mengurus anak, tapi juga merawat orang
alkohol, serta kemiskinan dan kesenjangan, tuanya yang bisa aja sakit dan membutuhkan
juga merupakan dampak dari rendahnya pengobatan hal ini karena merawat orang tua
tingkat literasi. memang budaya Indonesia yang kental. (5)
Tentu pada situasi ini, hampir semua kalangan
8. Apakah gagasan pada bacaan di atas memiliki beban finansial, terutama kalangan
dipisahkan menjadi dua paragraf? yang berada dalam usia produktif antara 15
sampai dengan 64 tahun. (6) Data dari
Bagaimanakah pengelompokan kalimat- Direktorat Jenderal Kementerian
kalimatnya agar gagasan pada masing- Ketenagakerjaan menyebutkan bahwa hingga
masing paragraf padu dan utuh? bulan Mei 2020 tercatat sudah 2,9 juta pekerja
(A) (1)-(2) dan (3)-(4)-(5)-(6)-(7) yang dirumahkan. (7) Generasi sandwich
(B) (1)-(2)-(3) dan (4)-(5)-(6)-(7) adalah orang-orang usia dewasa atau paruh
(C) (1)-(2)-(3)-(4) dan (5)-(6)-(7) baya yang terhimpit (sandwich) dalam
(D) (1)-(2)-(3)-(4)-(5) dan (6)-(7) memenuhi kebutuhan finansial dan kesehatan
(E) (1)-(2)-(3)-(4)-(5)-(6) dan (7) orang tuanya yang lansia dan juga
keturunannya.

10. Gagasan pada paragraf 2 akan menjadi


runtut apabila urutan kalimat-kalimatnya
….
(A) (7)-(4)-(3)-(6)-(5)
(B) (7)-(3)-(6)-(4)-(5)
(C) (4)-(6)-(3)-(5)-(7)
(D) (5)-(7)-(4)-(3)-(6)
(E) (3)-(6)-(7)-(4)-(5)
(1) Sejak dahulu, lidah buaya dikenal
sebagai tanaman herbal yang banyak (1) Sekolah favorit menjadi incaran para
digunakan untuk mengatasi berbagai keluhan orang tua. (2) Setiap memasuki awal tahun
pada kulit dan rambut. (2) Namun, saat ini
pelajaran, orang tua berlomba memasukkan
lidah buaya sudah banyak dikembangkan di
Asia, Eropa, dan Amerika Selatan. (3) Lidah anaknya ke sekolah favorit. (3) Mereka
buaya diketahui memiliki banyak khasiat bagi beranggapan bahwa sekolah favorit mampu
kulit dan rambut. (4) Itulah sebabnya, lidah mengantarkan sang anak meraih prestasi
buaya dipakai sebagai bahan berbagai produk akademik sehingga berpeluang melanjutkan ke
kecantikan dan kesehatan. (5) Gel atau ekstrak sekolah favorit jenjang berikutnya. (4)
lidah buaya yang ditambahkan ke dalam Globalisasi sekolah favorit dan “non favorit”
sabun, sampo, losion, atau krim wajah. (6)
pun berkembang. (5) Ujungnya, sekolah favorit
Selain itu, lidah buaya juga bisa dikonsumsi
dalam bentuk makanan dan minuman. menjadi rebutan. (6) Sebaliknya, sekolah yang
(7) Lidah buaya merupakan tanaman dianggap tidak favorit kesulitan memperoleh
yang mengandung beragam nutrisi untuk peserta didik.
kulit. (8) Berbagai manfaat lidah buaya untuk (7) Agar rutinitas tahunan tersebut tidak
kesehatan dan kecantikan kulit wajah antara berulang, sejak 2017 Kemendikbud menguji
lain adalah membantu mempertahankan coba sistem Penerimaan Peserta Didik Baru
kelembaban kulit, mempercepat penyembuhan
(PPDB) berbasis zonasi. (8) Pada 2018, melalui
luka di kulit, memperkuat jaringan kulit, dan
membantu menghaluskan permukaan kulit. (9) Permendikbud Nomor 14/2018, sistem PPDB
Akan tetapi, perlu diingat bahwa tidak semua telah diinisiasi dan diuji coba di sejumlah
orang cocok dengan lidah buaya. (10) Jika daerah, seperti Bali dan Yogyakarta. (9) Sistem
muncul ruam kemerahan, gatal, atau perih PPDB ini diberlakukan secara menyeluruh
pada kulit setelah diolesi lidah buaya, pada 2019 melalui Permendikbud Nomor
kemungkinan hal itu disebabkan oleh iritasi
51/2018 sebagai penyempurnaan aturan
atau alergi.
sebelumnya. (10) Kebijakan ini jelas bertujuan
11. Apa inti teks tersebut? memeratakan kualitas sekolah sehingga
(A) Lidah buaya dapat diolah sebagai masyarakat tidak lagi membuat dikotomi
bahan kecantikan antara sekolah unggulan dan bukan sekolah
(B) Lidah buaya memiliki khasiat bagi unggulan. (11) Ke depan, kebijakan ini harus
kulit dan rambut segera diikuti dengan penataan sekolah dan
(C) Lidah buaya dipakai untuk produk
penataan guru, baik dalam jumlah maupun
kecantikan
(D) Lidah buaya berguna bagi perawatan kualitasnya.
(Diadaptasi dari https://www.jawapos.com)
rambut
(E) Lidah buaya bermanfaat bagi
kesehatan 13. Kata ini pada kalimat (10) merujuk ….
(A) penyempurnaan aturan
12. Mengapa pada paragraf pertama perlu (B) PPDB berbasis zonasi
ditambahan kalimat? (C) Permendikbud Nomor 14/2018
(A) Keluhan pada kulit dan rambut dalam
(D) Permendikbud Nomor 51/2018
kalimat (1) perlu diberi contoh.
(B) Informasi tentang negara asal tanaman (E) pengujicobaan sistem PPDB
lidah buaya pada kalimat (2) perlu
disebutkan.
(C) Perlu diberi contoh khasiat lidah buaya
bagi rambut dan kulit pada kalimat (3).
(D) Produk kecantikan dan kesehatan
dalam kalimat (4) perlu diberi contoh.
(E) Perlu ditambahkan penjelasan tentang
bentuk gel lidah buaya setelah kalimat
(5).
14. Anda perlu menambahkan kalimat berikut (D) Selain itu, puntung rokok merupakan
dalam paragraf kedua. limbah yang sangat berbahaya bagi
Bila sebelumnya kriteria penentuan adalah nilai kehidupan di bumi.
ujian nasional di jenjang pendidikan (E) Padahal, puntung rokok juga termasuk
sebelumnya, mulai tahun ini patokan yang sampah yang mengandung zat yang
digunakan adalah jarak antara rumah peserta sangat berbahaya.
didik dan sekolah.
(1) Bahasa Indonesia merupakan alat
Kalimat tersebut paling tepat ditempatkan penghubung kemajemukan suku bangsa di
setelah …. Nusantara. (2) Kedudukan bahasa Indonesia
(A) kalimat (7) sebagai lingua franca atau bahasa pergaulan
(B) kalimat (8) berlangsung sejak abad ke-7 pada masa
(C) kalimat (9) Kerajaan Sriwijaya dalam rupa bahasa melayu.
(D) kalimat (10) (3) Sebagai lingua franca, posisi bahasa
(E) kalimat (11) Indonesia semakin kuat. (4) Kongres II Pemuda
Indonesia pada 28 Oktober 1928 mendaulat
15. Tujuan penulis menghasilkan teks tersebut bahasa Melayu sebagai bahasa nasional
adalah …. Indonesia.
(A) memaparkan tujuan PPDB berbasis (5) Di samping bahasa Indonesia,
zonasi terdapat pula banyak bahasa daera di
(B) membuktikan perlunya PPDB berbasis Indonesia. (6) Hingga tahun 2019 Badan
zonasi Pengembangan Bahasa dan Perbukuan telah
(C) menguraikan kebijakan PPDB berbasis memetakan bahasa-bahasa daerah di
zonasi Indonesia. (7) Hasil pemetaan itu menunjukkan
(D) meyakinkan keberhasilan PPDB bahwa Indonesia memiliki 718 bahasa daerah.
berbasis zonasi (8) Selain itu, terdapat 79 bahasa daerah di
(E) menggambarkan dikotomi berbasis wilayah Maluku, 72 bahasa di Nusa Tenggara
sistem zonasi Timur, 62 bahasa di Sulawesi, 58 bahasa di
Kalimantan, 26 bahasa di Sumatra, 11 bahasa di
16. (1) Puntung rokok termasuk benda yang Nusa Tenggara Barat, dan 10 bahasa di Jawa
banyak mengotori planet bumi. (2) dan Bali. (9) Sejak itu, upaya pelestarian bahasa
Puntung rokok sangat berbahaya. (3) Indonesia mulai mengental di dalam dada para
Setidaknya dua pertiga dari total 5,6 triliun pemuda. (10) Oleh karena itu, dengan
batang rokok yang diisap setiap tahun keragaman bahasa daerah di suatu wilayah
dibuang sembarang. (4) Sejak tahun 1980- menjadikan posisi bahasa Indonesia sebagai
an, puntung rokok menyumbang 30% bahasa pergaulan semakin penting.
hingga 40% dari semua sampah yang (Diadaptasi dari harian Kompas, 28 Oktober 2019)
ditemukan di jalanan, taman, pantai, dan
saluran air. 17. Apa judul tulisan yang paling tepat untuk
teks tersebut?
Kalimat manakah yang paling tepat (A) Asal Mula Bahasa Indonesia
sebagai perluasan kalimat (2) agar padu (B) Pemerataan Bahasa di Indonesia
dengan kalimat yang lain? (C) Keragaman Bahasa di Indonesia
(A) Karena itu, puntung rokok sangat (D) Pelestarian Bahasa Indonesia dan
berbahaya bagi planet bumi. Bahasa Daerah
(B) Namun, puntung rokok juga sangat (E) Perkembangan Bahasa Indonesia
berbahaya bagi planet bumi. sebagai Lingua Franca
(C) Bahkan, puntung rokok sangat
berbahaya bagi lingkungan hidup.
18. Kalimat yang menyebabkan 20. Tujuan penulisan kalimat (3) pada bacaan
ketidakpaduan ide paragraf kedua adalah di atas adalah untuk ….
…. (A) memberikan contoh rendahnya tingkat
(A) kalimat (6) literasi di Indonesia
(B) kalimat (7) (B) menguatkan pendapat perihal akibat
(C) kalimat (8) rendahnya literasi
(D) kalimat (9) (C) memerinci perihal penyebab
(E) kalimat (10) rendahnya tingkat literasi di Indonesia
(D) menekankan betapa rendahnya tingkat
(1) Media massa seperti Kompas, The literasi di Indonesia
Jakarta Post, dan kantor berita Antara (E) memberikan pendapat mengenai
memaparkan data yang diklaim berasal dari pentingnya literasi
UNESCO (data tersebut tidak ditemukan di
basis data UNESCO dan permintaan data ke (1) Dua dekade demokratisasi di
kantor UNESCO di Jakarta tidak dijawab) Indonesia ditandai oleh kehadiran para aktivis
mengenai minat baca orang Indonesia yang masyarakat sipil dalam politik formal. (2)
rendah. (2) Disebutkan angka 0,001, yang Mereka berasal dari Lembaga Swadaya
diartikan hanya 1 dan 1.000 orang Indonesia Masyarakat (LSM) atau merupakan mantan
memiliki minat baca tinggi. demonstran mahasiswa dan bagian kelompok
(2) Tahun lalu Central Connection State intelektual progresif. (3) Setelah gerakan
University juga mendudukkan tingkat literasi Reformasi 1998 berhasil menumbangkan rezim
masyarakat Indonesia di posisi 60 dari 61 otoriter Soeharto, para aktivis masyarakat sipil
negara yang disurvei, hanya setingkat di atas mulai mengubah strategi mereka. (4) Mereka
Botswana. (4) Meski rangking ini bukan berupaya untuk mereformasi kondisi sosial,
perkara minat baca tetapi antara lain masalah ekonomi, dan politik Indonesia dari dalam,
akses komputer, sirkulasi surat kabar, dan menggantikan strategi lama “perjuangan dari
tingkat pemahaman literasi, survei ini juga luar”. (5) Banyak mantan aktivis yang masuk
kerap dipakai pejabat dan tokoh untuk partai-partai politik besar untuk menjadi
menyatakan keprihatinan atas rendahnya anggota parlemen atau kepala daerah. (6)
minat baca. (5) Survei Sosial dan Ekonomi Sebagian ada yang diangkat sebagai pejabat
Nasional (Susenas) yang dilakukan Badan negara atau komisaris Badan Usaha Milik
Pusat Statistik (BPS) sampai 2015 juga Negara (BUMN). (7) Kebanyakan dari mereka
menunjukkan kecenderungan meningkatnya menempati posisi sebagai staf khusus sebuah
penonton televisi hingga mencapai 91,5% posisi yang fleksibel dalam hal status
sementara pembaca surat kabar hanya 13,1% pekerjaan, fungsi, karier, dan insentif finansial
pada 2015, atau rerendah sepanjang tercatat di partai-partai politik. Dewan Perwakilan
oleh BPS sejak 1984 Rakyat (DPR) atau kantor-kantor pemerintah.
(8) Namun, kehadiran para mantan aktivis itu
19. Dimanakah sebaiknya posisi Pejabat dan tampak gagal dalam mewujudkan perubahan
tokoh yang kerap menyatakan keprihatinan fundamental untuk memperbaiki kualitas
sama bahwa warga Indonesia rendah minat demokrasi. (9) Ini berlaku terutama dalam isu-
bacanya? isu hak asasi manusia, layanan publik, dan
(A) Setelah kalimat (1) korupsi, menurut beberapa studi yang
(B) Setelah kalimat (2) diterbitkan dalam jurnal sosial politik Prisma.
(C) Setelah kalimat (3)
(D) Setelah kalimat (4) 21. Apabila gagasan pada bacaan di atas
(E) Setelah kalimat (5) dipisahkan menjadi dua paragraf.

Bagaimanakah pengelompokan kalimat-


kalimatnya agar gagasan pada masing-
masing paragraf padu dan utuh?
(A) (1)-(2) dan (3)-(4)-(5)-(6)-(7)-(8)-(9)
(B) (1)-(2)-(3) dan (4)-(5)-(6)-(7)-(8)-(9)
(C) (1)-(2)-(3)-(4) dan (5)-(6)-(7)-(8)-(9)
(D) (1)-(2)-(3)-(4)-(5) dan (6)-(7)-(8)-(9)
(E) (1)-(2)-(3)-(4)-(5)-(6) dan (7)-(8)-(9)
22. Kalimat (8) pada bacaan di atas mewadahi (1) Membincang sastra anak sama saja
maksud penulis untuk …. dengan memutar kembali jarum jam dan
(A) menyindir kehadiran mantan aktivis mengingat masa anak dulu saat menjelang
sebagai politisi tidur, ketika kakek, nenek, dan atau orang tua
(B) memberikan informasi mengenai kita mendongengkan cerita tentang kancil,
fungsi mantan aktivis di politik formal buaya, harimau, juga legenda asal-usul sebuah
(C) mengkritik mantan aktivis yang terjun daerah. (2) Terlepas dari motif komunikasi
di dunia politik formal antar anggota keluarga, pada momentum
(D) mengemukakan akibat dari masuknya tersebut, sastra telah menjadi gerbang nilai-
aktivis ke dalam politik formal nilai sosial dan humanis. (3) Dunia anak adalah
(E) menawarkan solusi terhadap dunia kaya imajinasi. (4) Sebagai anak-anak.
keberadaan mantan aktivis di politik Kita dulu pasti pernah mengimajinasikan diri
formal kita sebagai tokoh dalam dongeng, superhero,
tokoh kartun, bahkan mungkin presiden. (5)
(1) Indonesia belum menandatangani Sekarang, sebagai orang tua, kata tugas kita
Konvensi Pengungsi PBB. (2) Namun, pada semua adalah memberikan arah imajinasi
2016 terbit peraturan presiden (perpes) anak-anak ke wilayah yang positif. (6) Dengan
mengenai penanganan pengungsi dari luar demikian, hubungan orang tua dan anak juga
negeri. (3) Peraturan tersebut menunjukkan menjadi arena berimajinasi.
posisi hukum Indonesia yang terbaru
mengenai pengungsi dan pencari suaka. (4) 24. Dimanakah sebaiknya posisi kalimat Tanpa
Indonesia menganggap dirinya sebagai negara kita sadari, sejatinya keluarga kita telah
transit bagi para pengungsi dan pencari suaka, mengaka kita untuk belajar sastra pada bacaan
alih-alih negara tujuan atau tempat tinggal di atas?
baru. (A) Antara kalimat 1 dan 2
(5) Namun, perpres tersebut mencakup (B) Antara kalimat 2 dan 3
pula ketentuan untuk pencegatan dan (C) Antara kalimat 3 dan 4
penyelamatan para pengungsi dalam perairan (D) Antara kalimat 4 dan 5
Indonesia. (6) Dalam proses perancangan (E) Antara kalimat 5 dan 6
perpres ada praduga bahwa individu yang
mencari suaka tidak akan dihadap pada 25. Tujuan penulisan … sastra telah menjadi
refoulement (pemulangan secara tidak gerbang nilai-nilai sosial dan humanis
sukarela ke negara asal mereka menghadapi dalam kalimat (2) bacaan di atas adalah
persekusi). (7) Tidak ada ketentuan untuk untuk ….
penempatan jangka panjang. (8) Menurut (A) menyimpulkan tahapan-tahapan
ketentuan dalam peraturan presiden tersebut, dalam perkembangan sastra anak
setiap pengungsi yang diproses oleh UNHCR (B) menginformasikan tahapan
di Indonesia akan ditempatkan di negara ketiga selanjutnya dalam perkembangan
atau dipulangkan ke negara asal. sastra anak
(C) menyatakan pentingnya peran sastra
23. Gagasan pada paragraf 2 akan menjadi anak dalam komunikasi di dalam
runtut apabila urutan kalimat-kalimatnya keluarga
adalah …. (D) mengungkapkan pendapat penulis
(A) (5)-(8)-(7)-(6) tentang peran penting sastra anak
(B) (8)-(5)-(7)-(6) (E) memberikan suatu contoh tentang apa
(C) (6)-(7)-(8)-(5) yang diimajinasikan oleh anak
(D) (5)-(7)-(8)-(6)
(E) (8)-(7)-(5)-(6)
(1) Proses pendidikan bisa dilakukan 28. (1) Masa remaja adalah masa pertumbuhan
melalui media komunikasi seperti telepon, yang terjadi dengan sangat cepat. (2)
komputer, dan e-mail. (2) Interaksi antara guru Remaja membutuhkan nutrisi yang lebih
dan siswa tidak hanya melalui tatap muka, banyak untuk dapat mendukung
tetapi juga bisa dilakukan melalui media pertumbuhan tulang, perubahan hormon,
online. (3) Teknologi memberikan banyak dan perkembangan jaringan otak. (3) Salah
manfaat sekarang ini. (4) Guru dapat satu nutrisi utama yang dibutuhkan remaja
memberikan layanan tanpa harus berhadapan adalah kalsium. (4) Salah satu sumber
langsung dengan siswanya. (5) Demikian pula kalsium bisa dikonsumsi remaja adalah
siswa dapat memperoleh informasi dalam susu. (5) Remaja perlu mengoptimalkan
lingkup yang luas dari berbagai sumber kesehatan tulangnya agar memiliki risiko
melalui cyberspace di internet. (6) Hal yang lebih rendah mengalami osteoporosis saat
paling mutakhir adalah berkembangnya apa dewasa. (6) Risiko ini ditemukan lebih
yang disebut cyber teaching. (7) Cyber teaching tinggi pada remaja wanita. (7) Remaja
adalah proses pengajaran menggunakan harus mendapatkan kalsium yang cukup
internet. (8) Istilah lain yang semakin popular untuk digunakan dalam pertumbuhan
saat ini ialah e-learning. (9) E-learning tulang.
merupakan suatu model pembelajaran
menggunakan media teknologi komunikasi Gagasan pada paragraf 2 akan menjadi
dan informasi runtut apabila urutan kalimat-kalimatnya
adalah ….
26. Apabila gagasan pada bacaan di atas (A) (3)-(7)-(4)-(5)-(6)
dipisahkan menjadi dua paragraf. (B) (3)-(7)-(4)-(6)-(5)
Bagaimanakah pengelompokan kalimat- (C) (3)-(7)-(5)-(4)-(6)
kalimatnya agar gagasan pada masing- (D) (3)-(7)-(5)-(6)-(4)
masing paragraf padu dan utuh? (E) (3)-(7)-(6)-(5)-(4)
(A) (1)-(2)-(3) dan (4)-(5)-(6)-(7)-(8)-(9)
(B) (1)-(2)-(3)-(4) dan (5)-(6)-(7)-(8)-(9) (1) Pijat bayi merupakan terapi alternatif
(C) (1)-(2)-(3)-(4)-(5) dan (6)-(7)-(8)-(9) yang sudah diterapkan sejak lama. (2) Namun,
(D) (1)-(2)-(3)-(4)-(5)-(6) dan (7)-(8)-(9) sekarang pijat bayi digali kembali oleh ilmu
(E) (1)-(2)-(3)-(4)-(5)-(6)-(7) dan (8)-(9) kesehatan dan dijadikan kajian ilmiah para
pakar. (3) Saat ini pijat bayi didefinisikan
27. Kalimat wujud pada bacaan di atas sebagai terapi sentuh dalam seni perawatan
mewadahi maksud penulis untuk …. dan pengobatan dengan kontak tubuh
(A) memberikan definisi singkat istilah berkelanjutan yang menimbulkan rasa nyaman
cyber teaching dan aman sehingga meningkatkan kesehatan
(B) menyimpulkan pentingnya cyber fisik dan psikologis bayi. (4) Pijat bayi
teaching bagi siswa dan guru ICT membantu tumbuh kembang fisik dan emosi
(C) menekankan bahwa cyber teaching bayi dan mempererat hubungan ibu dan bayi.
sudah diterapkan di seluruh sekolah (5) Pijat bayi mengutamakan sentuhan
(D) menawarkan solusi tentang ICT dan jari pemijat yang dapat memberikan stimulasi
masalahnya dalam proses pada tubuh bayi. (6) Pijat bayi bermanfaat
pembelajaran untuk meningkatkan sistem imunitas dan
(E) memberikan contoh tentang proses merangsang fungsi pencernaan, yaitu
pembelajaran berdasarkan ICT meningkatkan penyerapan makanan sehingga
dapat membantu peningkatan berat badan
bayi. (7) Bayi yang dipijat secara rutin memiliki
kualitas tidur yang baik karena bayi merasa
nyaman dan sirkulasi pembuluh darahnya
lancar setelah dipijat. (8) Sirkulasi pembuluh (1) Bersekolah merupakan proses
darah yang lancar, dapat memberikan efek pembelajaran menjadi manusia seutuhnya
yang baik bagi pernapasan bayi dan yang beradab dan berbudaya. (2) Manusia
meningkatkan kecukupan oksigen bayi. (9) berguna dan bermakna bagi manusia lainnya
Saat pijat bayi, tekanan yang diberikan dapat sehingga sekolah merupakan salah satu sarana
memperkuat otot dan tulang bayi untuk efektif untuk membangun karakter generasi
melakukan gerakan kasar. (10) Gerakan pijat muda yang sadar lingkungan. (3) Sekolah sadar
bayi di bagian perut dapat mengurangi risiko bahwa kebutuhan memelihara lingkungan
sembelit dan kolik. merupakan upaya baik demi masa depan.
(4) Untuk mewujudkan sekolah sadar
29. Apa inti teks tersebut? lingkungan, setidaknya diperlukan perhatian
(A) Pijat bayi merupakan terapi alternatif pada dua aspek penting, yaitu inisiatif dan
untuk meningkatkan kesehatan bayi. pelembagaan. (5) Inisiatif dari semua warga
(B) Pijat bayi merupakan terapi alternatif sekolah ditumbuhkan demi terciptanya
untuk memperlancar sirkulasi darah budaya sekolah yang sadar lingkungan. (6)
bayi. Inisiatif yang disertai komitmen dapat menjadi
(C) Pijat bayi merupakan terapi alternatif kunci keberhasilan menumbuhkan budaya
yang bermanfaat bagi pertumbuhan sadar lingkungan. (7) Hal itu dapat datang dari
bayi. mana saja. (8) Pelembagaan dapat dilakukan
(D) Pijat bayi merupakan terapi alternatif dengan menempatkan sekolah sebagai ajang
yang dapat mempererat relasi ibu dan
kerja sama antar personel di sekolah untuk
anak.
mewujudkan budaya sekolah yang sejalan
(E) Pijat bayi merupakan terapi alternatif
dengan tujuan pembangunan berkelanjutan,
yang dapat mempercepat
terutama yang berkaitan dengan tumbuhnya
perkembangan bayi.
kesadaran lingkungan.

30. Mengapa pada paragraf pertama perlu


31. Kata itu pada kalimat (7) mengacu ….
ditambahkan kalimat?
(A) inisiatif yang disertai komitmen
(A) Antara kalimat (1) dan (2) perlu
(B) kunci keberhasilan
disisipkan kalimat penjelas tentang
(C) tumbuhnya budaya sadar lingkungan
praktik pijat bayi pada zaman dahulu.
(D) inisiatif dan pelembagaan
(B) Perlu diberikan penjelasan setelah
(E) terciptanya budaya sekolah
kalimat (2) tentang hasil pengkajian
ilmiah yang berkaitan dengan pijat
32. Anda perlu menambahkan kalimat berikut
bayi.
dalam paragraf kedua.
(C) Diperlukan tambahan penjelasan
Biasanya inisiatif membangun kesadaran
tentang tujuan terapi sentuh setelah
lingkungan menjadi lebih diperhatikan jika data
kalimat (3).
dari pihak sekolah daripada dari individu.
(D) Perlu ditambahkan kalimat contoh
tentang terapi sentuh setelah kalimat
Kalimat tersebut paling tepat ditempatkan
(3).
(E) Setelah kalimat (4) perlu ditambahkan setelah ….
dampak fisik dan emosi pada bayi (A) kalimat (4)
setelah menjalani terapi sentuh. (B) kalimat (5)
(C) kalimat (6)
(D) kalimat (7)
(E) kalimat (8)
33. Tujuan penulis menghasilkan teks tersebut (1) Wayang merupakan media yang
adalah …. dapat membentuk karakter anak bangsa. (2)
(A) memaparkan inisiatif sekolah untuk Hal ini terjadi karena wayang sebagai sebuah
menanamkan kesadaran lingkungan di gambaran karakter manusia dapat diartikan
pula sebagai gambaran kehidupan manusia. (3)
sekolah
Dalam wayang, kita bisa melihat nilai-nilai
(B) membuktikan kesadaran lingkungan
kebaikan dan keburukan melalui tokoh-tokoh
sebagai kunci keberhasilan wayang dan cerita wayang dimainkan. (4)
pembelajaran Tokoh-tokoh wayang tersebut memiliki
(C) menguraikan hal-hal yang berkaitan berbagai kepribadian. (5) Ada yang tamak dan
dengan kesadaran lingkungan mudah emosi, tetapi ada pula yang murah hati
(D) menjelaskan aspek-aspek dan bijaksana. (6) Contoh tokoh yang Arif
pembentukan kesadaran lingkungan bijaksana adalah Sri Rama dan Arjuna. (7)
di lingkungan sekolah Kedua tokoh tersebut selalu mengedepankan
(E) mengajak siswa untuk memperhatikan kebenaran dan keadilan serta bertutur kata
harus diimbangi dengan tingkah laku yang
kebersihan lingkungan sekolah
baik. (8) Untuk kecintaan kepada wayang dan
karakternya kepada anak sebaiknya
34. (1) Energi merupakan kemampuan untuk ditanamkan sejak dini.
melakukan kerja dan menjadi hal yang (9) Zaman selalu mengalami perubahan,
fundamental dalam kehidupan. (2) Selama namun wayang tetap eksis. (10) Wayang tidak
ada kehidupan pasti terdapat energi. (3) hanya dijadikan sebagai media hiburan, tetapi
Sebagian besar energi adalah energi listrik. juga membentuk karakter, terutama pada
(4) Energi listrik memiliki fungsi yang generasi muda. (11) Tokoh-tokoh wayang,
sangat krusial dalam kehidupan kita, seperti Pandawa dan Punakawan, dapat
terutama sebagai penerang pada malam diperkenalkan kepada mereka. (12) Selain itu,
mereka juga diperkenalkan bagaimana
hari. (5) Kita semua menggunakan energi
bertingkah laku yang baik. (13) Melalui
listrik dalam kehidupan sehari-hari, seperti
karakter itu, generasi muda diperkenalkan
berkendara, memasak, dan memanaskan pada kejujuran dan kebaikan. (14) Dengan
atau mendinginkan sesuatu, bahkan demikian, generasi penerus akan lebih banyak
menggunakan internet untuk membaca mengenal dan mendapatkan nilai-nilai
berita. karakter bagi kehidupannya.

Kalimat manakah yang paling tepat 35. Apa judul yang paling tepat untuk teks
sebagai perluasan kalimat (3) agar padu tersebut?
dengan kalimat yang lain? (A) Wayang dan Generasi Muda
(B) Karakter Positif Tokoh Wayang
(A) Padahal, sebagian besar energi adalah
(C) Wayang Pembentuk Karakter
energi listrik.
(D) Eksistensi Wayang di Masa Kini
(B) Hal itu karena sebagian besar energi (E) Wayang dan Perubahan Zaman
adalah energi listrik.
(C) Sebagian besar energi yang digunakan 36. Kalimat yang menyebabkan
adalah energi listrik. ketidakpaduan ide paragraf kedua adalah
(D) Sebagian besar kebutuhan energi di ….
dunia adalah energi listrik. (A) kalimat (9)
(E) Sementara itu, sebagian besar energi (B) kalimat (10)
adalah energi listrik. (C) kalimat (11)
(D) kalimat (12)
(E) kalimat (14)
37. (1) Vitamin A murni berupa kristal kuning 39. Urutkanlah enam kalimat berikut ini
pucat, tahan terhadap panas dan sinar, dan sehingga menjadi paragraf yang baik!
mudah rusak karena oksidasi. (2) Vitamin 1. Dalam hal ini, pemandu harus
A merupakan salah satu vitamin yang dilengkapi dengan kemampuan
tidak larut dalam air. (3) Karenanya, memberi penjelasan secara terperinci
vitamin A berkhasiat untuk menjaga kepada wisatawan.
keuntuhan kulit serta meningkatkan 2. Dalam menjual pariwisata, terutama
kesehatan mata dan pertahanan tubuh ketika masuk sentra-sentra kerajinan,
terhadap infeksi kuman, virus, dan parasit. peran pemandu sangat besar.
(4) Namun, vitamin ini larut dalam pelarut 3. Mereka perlu penjelasan yang
lemak. (5) Vitamin ini penting untuk terperinci dan lugas tentang apa yang
keutuhan lapisan epitel yang menyeliputi dilihatnya.
tubuh, yaitu kulit dan jaringan mukosa 4. Dengan demikian, mereka akan
yang menutupi mata dan rongga saluran merasa mendapatkan suatu
pencernaan, saluran pernapasan, serta pengalaman yang luar biasa, seakan-
saluran kemih. akan ikut mengalami sendiri.
5. Wisatawan tidak hanya dibiarkan
Urutan kalimat yang tepat untuk paragraf melihat demonstrasi proses produksi
di atas adalah …. kerajinan tanpa penjelasan.
(A) 1-3-2-5-4 6. Misalnya, ketika menyaksikan
(B) 2-4-5-1-3 demostrasi proses pemintalan sutra,
(C) 2-5-4-1-3 mereka harus diberi penjelasan tentang
(D) 5-1-4-2-3 proses ini mulai dari pengadaan bahan
(E) 2-4-1-5-3 hingga maksud permintalan.

38. (1) Demikian halnya kalau kita sedang Urutan yang baik adalah ….
bekerja. (2) Karbohidrat dapat diibaratkan (A) 3-4-5-1-2-6
sebagai kayu bakar dalam proses (B) 2-1-5-3-6-4
pembakaran. (3) Karbohidrat merupakan (C) 1-2-4-5-6-3
makanan yang dapat menjadi sumber (D) 4-5-3-2-1-6
tenaga. (4) Semakin keras kita bekerja, (E) 2-1-6-3-5-4
semakin banyak pula sumber energi yang
diperlukan. (5) Akan tetapi, jika hanya 40. (1) Setiap hari jumlah berlipat dua, dua
mengonsumsi karbohidrat tanpa protein, helai pada hari kedua, empat pada hari
tubuh kita akan menjadi lemah. (6) ketiga, delapan pada hari keempat, dan
Semakin besar api yang kita inginkan, seterusnya. (2) Di sebuah kolam, menurut
semakin banyak pula kayu bakar yang kita teka-teki itu, tumbuh sehelai daun teratai.
perlukan. (3) Jawabannya, “pada hari kedua puluh
Sembilan.” (4) Orang Perancis
Urutan yang tepat agar kalimat-kalimat di menggunakan teka-teki untuk
atas menjadi paragraf yang baik adalah …. mengajarkan pertumbuhan penduduk
(A) (3)-(2)-(1)-(4)-(5)-(6) yang berlipat ganda kepada anak sekolah.
(B) (3)-(2)-(6)-(1)-(4)-(5) (5) “Kalau kolam teratai itu penuh pada
(C) (2)-(6)-(1)-(4)-(5)-(3) hari ketiga puluh, kapankah kolam itu
(D) (2)-(3)-(6)-(1)-(4)-(5) tertutup separuhnya oleh daun teratai?”
(E) (3)-(5)-(2)-(6)-(1)-(4)
Urutan kalimat yang tetap untuk paragraf
di atas adalah ….
(A) (2)-(1)-(3)-(5)-(4)
(B) (4)-(2)-(1)-(3)-(5)
(C) (4)-(5)-(1)-(2)-(3)
(D) (4)-(2)-(1)-(5)-(3)
(E) (5)-(1)-(2)-(3)-(4)
PENALARAN

Penalaran dalam soal-soal literasi bahasa Indonesia menguji kemampuan logika dalam berbahasa
seperti (1) kesesuaian pernyataan dan (2) simpulan logis.
A. Kesesuaian Pernyataan
Kesesuaian pernyataan merupakan jenis soal yang akan menguji kemampuan menemukan
kesamaan informasi dalam wacana. Kemampuan menemukan informasi ini dapat dipertajam
dengan menguasai teknik membaca skimming (membaca sekilas) dan scanning (membaca
memindai).

Kesesuaian pernyataan berkaitan dengan penemuan butir-butir informasi pada


(a) teks,
(b) tabel,
(c) grafik, maupun
(d) diagram.

Jenis informasi yang dicari seperti


(a) pernyataan benar/salah sesuai dengan wacana,
(b) pernyataan yang menguatkan/melemahkan,
(c) pernyataan relevan/tidak relevan,
(d) kesesuaian isi tabel/grafik/diagram,
(e) perbandingan isi tabel/grafik/diagram,
(f) jumlah isi tabel/grafik/diagram, atau
(g) rata-rata isi tabel/grafik/diagram.

B. Simpulan Logis
Simpulan logis dalam literasi bahasa Indonesia berkaitan dengan kemampuan penalaran dalam
wacana. Cara menaruk simpulan dapat dilakukan dengan (a) generalisasi, (b) analogi, dan (c)
kausalitas.
(a) Generalisasi
Generalisasi adalah cara menarik simpulan dengan menggunakan informasi-informasi
khusus sehingga bisa mewakili seluruh informasi dalam teksnya. Cara menarik simpulan
dengan generalisasi dapat dilakukan dengan mencari ide pokok dan mencari kata kunci.

(b) Analogi
Analogi adalah cara menarik simpulan dengan membandingkan informasi-informasi khusus
yang memiliki banyak persamaan. Cara-cara menarik simpulan dengan analogi seperti
tautologi (pernyataan majemuk yang selalu bernilai benar), kontradiksi (pernyataan majemuk
yang selalu bernilai salah), modus ponens, modus tollens, dan silogisme.
• Tautologi
p ~p pV~p
B S B
S B B

Dia berdiri atau tidak berdiri. (benar)


p V ~p
Dia tidak berdiri atau berdiri. (benar)
~p V p
• Kontradiksi
p ~p pΛ~p
B S S
S B S

Dia berdiri dan tidak berdiri. (salah)


p Λ ~p
Dia tidak berdiri dan berdiri. (salah)
~p Λ p

• Modus ponens
premis 1 : p => q Jika dia berdiri, maka dia lelah.
premis 2 :p Dia berdiri
kesimpulan : q Dia lelah

premis 1 : p => q Jika dia berdiri, maka dia lelah.


premis 2 : ~p Dia tidak berdiri
kesimpulan : tidak dapat ditarik tidak dapat ditarik kesimpulan
kesimpulan

• Modus tollens
premis 1 : p => q Jika dia berdiri, maka dia lelah.
premis 2 : ~q Dia tidak lelah
kesimpulan : ~p Dia tidak berdiri

premis 1 : ~q => ~p Jika dia tidak berdiri, maka dia tidak lelah.
premis 2 : ~q Dia tidak lelah
kesimpulan : ~p Dia tidak berdiri

premis 1 : p => q Jika dia berdiri, maka dia lelah.


premis 2 :q Dia lelah
kesimpulan : tidak dapat ditarik tidak dapat ditarik kesimpulan
kesimpulan

• Silogisme
premis 1 : p => q Jika dia berdiri, maka dia lelah.
premis 2 : q => r Jika dia lelah, maka dia akan haus.
kesimpulan : p => r Jika dia berdiri, maka dia akan haus.

(c) Kausalitas
Kausalitas adalah cara menarik simpulan dengan menggunakan fakta yang berupa ‘sebab’
sehingga akan muncul simpulan berupa ‘akibat’. Cara menarik simpulan dengan metode
kausalitas dapat dilakukan dengan mencari sebab dalam wacana.
Teks berikut digunakan untuk menjawab soal 2. Manakah informasi berikut yang dapat
nomor 1 sampai dengan 4. memperkuat pernyataan “obat herbal yang
sering dijadikan obat alternatif karena
Obat herbal sering dijadikan obat harga obat kimia yang tidak terjangkau”?
alternatif karena harga obat kimia yang tidak (A) Sebagian besar obat herbal memiliki
terjangkau. Meskipun banyak masyarakat
harga yang jauh lebih tinggi
yang mengonsumsinya, sebagian tidak
dibandingkan obat kimia.
mengetahui zat apa yang terkandung di dalam
(B) Sebagian besar pasien yang tergolong
obat herbal. Padahal, zat tertentu bisa
masyarakat tidak mampu secara
berbahaya bagi kesehatan. Ketidaktahuan
masyarakat ini terjadi ketika produsen tidak ekonomi beralih dari obat kimia ke
menyertakan informasi yang lengkap pada obat herbal.
kemasan obat herbal. Produsen berlaku (C) Sebagian obat herbal memberikan
demikian apabila pemerintah tidak melakukan reaksi fisik yang lebih cepat dirasakan
pengawasan secara ketat. Selain itu, tidak oleh pasien dibandingkan obat kimia.
adanya tindakan hukum dari pemerintah (D) Sebagian pasien lebih memilih obat
menyebabkan jumlah produsen yang menjual herbal sebagai alternatif karena dapat
obat herbal berbahaya meningkat. Obat herbal terhindar dari efek samping obat
yang berbahaya biasanya izin edarnya belum kimia.
ada, efeknya langsung terasa pada sekali (E) Sebagian obat kimia lebih sulit
minum, dan produsennya tidak jelas. diperoleh dibandingkan obat herbal
yang beredar bebas.
1. Berdasarkan bacaan tersebut, manakah
simpulan berikut yang benar?
3. Berdasarkan bacaan tersebut, apabila suatu
(A) Pemerintah melakukan pengawasan
obat efeknya langsung terasa pada sekali
secara ketat apabila sebagian
masyarakat tidak mengetahui zat yang minum dan produsennya tidak jelas.
terkandung di dalam obat herbal. Manakah simpulan berikut yang paling
(B) Pemerintah tidak melakukan mungkin benar?
pengawasan secara ketat apabila (A) Obat tersebut bukan merupakan obat
sebagian masyarakat mengetahui zat kimia.
yang terkandung di dalam obat herbal. (B) Obat tersebut berisiko membahayakan
(C) Sebagian masyarakat mengetahui zat kesehatan.
yang terkandung di dalam obat herbal (C) Obat tersebut memiliki harga yang
apabila pemerintah tidak melakukan sangat murah.
pengawasan secara ketat. (D) Obat tersebut banyak dikonsumsi oleh
(D) Sebagian masyarakat tidak masyarakat.
mengetahui zat yang terkandung di (E) Obat tersebut mendapat tindakan
dalam obat herbal apabila pemerintah
hukum dari pemerintah.
melakukan pengawasan secara ketat.
(E) Sebagian masyarakat tidak
mengetahui zat yang terkandung di
dalam obat herbal apabila pemerintah
tidak melakukan pengawasan secara
ketat.
4. Manakah informasi berikut yang dapat (D) Penghijauan kembali di hutan gundul
memperlemah pernyataan “Tidak adanya menyebabkan penebangan hutan liar
tindakan hukum dari pemerintah meningkat.
menyebabkan jumlah produsen yang (E) Akar pohon sekitar hutan yang
menjual obat herbal berbahaya berkurang disebabkan oleh gagalnya
meningkat”? upaya penghijauan.
(A) Jumlah kasus obat herbal yang dibawa
ke pengadilan terus meningkat, 6. Pemerintah saat ini sedang
sedangkan jumlah obat herbal yang mengembangkan Palapa Ring untuk
beredar menurun. menghubungkan internet di berbagai
(B) Produsen yang tidak mendapatkan wilayah ibu kota kabupaten/kota di
tindakan hukum terus memproduksi Indonesia. Palapa Ring menggunakan
obat herbal berbahaya jaringan serat optik yang mampu
(C) Jumlah obat herbal yang ditemukan di menggantikan satelit yang lebih mahal.
pasaran terus meningkat, sedangkan Jika biaya satelit dapat ditekan, biaya
jumlah kasus yang dilaporkan koneksi internet menjadi lebih rendah.
menurun
(D) Sebagian produsen yang pernah Berdasarkan informasi tersebut, manakah
mendapatkan hukuman masih tetap pernyataan berikut yang pasti benar?
memproduksi obat herbal berbahaya (A) Mahalnya biaya koneksi internet saat
dan mengedarkannya di pasaran ini disebabkan belum adanya Palapa
(E) Peningkatan obat herbal berbahaya Ring
didorong oleh keuntungan yang dapat (B) Jaringan serat optik baru digunakan
diperoleh produsen secara tepat pertama kali pada Palapa Ring
(C) Palapa Ring mampu menyediakan
5. Penebangan hutan secara liar akan koneksi yang lebih cepat dibandingkan
mengurangi akar pohon di sekitar hutan. satelit yang ada saat ini
Berkurangnya akar pohon membuat (D) Palapa Ring membuat biaya koneksi
struktur tanah di hutan menjadi lemah. internet menjadi lebih terjangkau bagi
Ketika struktur tanah lemah, tanah akan masyarakat
rentan terhadap longsor. Meskipun banyak (E) Saat ini kabupaten/kota di Indonesia
upaya penghijauan kembali di hutan belum terhubung satu sama lain
gundul, masyarakat di sekitar hutan melalui internet
gundul tetap tidak mau tinggal disana
karena tanah mereka rentan terhadap
bencana tanah longsor.

Berdasarkan informasi tersebut, manakah


pernyataan berikut yang benar?
(A) Tanah yang rentan terhadap longsor
menyebabkan akar pohon sekitar
hutan berkurang.
(B) Akar pohon di hutan yang berkurang
disebabkan oleh struktur tanah hutan
yang lemah.
(C) Tanah di sekitar hutan menjadi rentan
terhadap longsor karena banyak
penebangan hutan secara liar.
7. Normalnya, urin mengandung kadar 9. Saat ini menurunnya kemampuan remaja
protein yang rendah. Kandungan protein untuk menjalin relasi sudah makin
dalam urin berlebihan menunjukkan mengkhawatirkan. Beberapa ahli
berpendapat bahwa salah satu
adanya gejala penyakit dalam tubuh.
penyebabnya adalah penggunaan gawai
yang berlebihan.
Berdasarkan pernyataan tersebut,
manakah yang paling mungkin menjadi Manakah pernyataan berikut yang akan
akibat dari kandungan protein berlebih memperlemah pendapat para ahli
dalam urin? tersebut?
(A) Orang yang memiliki kelebihan (A) Penggunaan gawai yang berlebihan
memengaruhi kesehatan mental
kandungan protein dalam urin harus
generasi milenial
banyak minum (B) Saat banyak remaja yang mengalami
(B) Kandungan protein yang berlebih ketergantungan pada gawai
ditemukan pada hasil tes urin orang (C) Remaja lebih banyak menghabiskan
sakit waktu untuk menggunakan gawai
(C) Orang yang sangat sehat tidak daripada untuk menjalin relasi
memiliki kandungan protein dalam (D) Banyak aplikasi pada gawai yang
bermanfaat untuk memperluas
urinnya
pergaulan remaja
(D) Jenis penyakit dalam tubuh ditentukan (E) Penggunaan gawai membuat
dari kandungan protein dalam urin kepekaan sosial remaja menurun
(E) Orang sehat tidak perlu memeriksakan
kandungan protein dalam urinnya 10. Makan mi instan menjadi kegemaran
banyak orang walaupun berdampak buruk
bagi kesehatan. Ahli gizi menyebutkan
8. Setiap daerah di Indonesia dapat memiliki
beberapa cara untuk mengonsumsi mi
lebih dari satu jenis pakaian adat. Ada instan secara sehat. Pertama,
pakaian adat yang dikenakan untuk mencampurnya dengan sayuran karena
melakukan aktivitas sehari-hari da nada serat yang terkandung dalam sayuran
juga yang hanya digunakan untuk acara dapat mencegah penyerapan karbohidrat
khusus. Pakaian adat tertentu memiliki dan gula. Kedua, meracik bumbu sendiri
simbol keagungan sehingga tidak semua karena kadar natrium atau garam yang ada
di bumbu mi instan sangat tinggi, yaitu
orang boleh mengenakannya sehari-hari.
850-950 mg, padahal tubuh hanya
Simpulan berdasarkan informasi dalam membutuhkan natrium sebanyak 2 mg.
teks tersebut adalah semua pakaian yang
memiliki simbol keagungan tidak boleh Berdasarkan informasi tersebut, manakah
dikenakan sehari-hari. pernyataan berikut yang pasti salah?
(A) Kadar garam yang tinggi membuat
banyak orang gemar makan mi instan.
Manakah pernyataan berikut yang
(B) Nutrisi yang terkandung dalam mi
menggambarkan kualitas simpulan
instan tidak memiliki manfaat bagi
tersebut? tubuh.
(A) Simpulan pasti benar (C) Mengonsumsi mi instan dengan
(B) Simpulan mungkin benar bumbu mi instan dan sayuran dapat
(C) Simpulan pasti salah membuat seseorang mendapatkan
(D) Simpulan mungkin salah asupan natrium yang sesuai dengan
kebutuhan.
(E) Simpulan tidak dapat dinilai karena
(D) Serat dalam mi instan mencegah
informasi tidak cukup penyerapan karbohidrat dan gula.
(E) Produsen menambahkan sayuran
dalam mi instan agar lebih
menyehatkan.
11. Ketua OSIS SMA ABC mengungkapkan (B) Biaya yang dibayarkan mahasiswa
bahwa tahun ini semakin sedikit jumlah perguruan tinggi swasta besar jika
siswa SMA ABC yang mau terlibat dalam peningkatan kualitas perguruan tinggi
kegiatan bakti sosial. Hal ini disebabkan swasta pada sarana dan prasarana
oleh rendahnya kemampuan siswa untuk tidak dilakukan.
berinteraksi dengan orang lain. (C) Peningkatan kualitas perguruan tinggi
swasta pada sarana dan prasarana
Manakah pernyataan berikut yang akan dilakukan jika biaya yang dibayarkan
memperkuat argumen ketua OSIS SMA mahasiswa perguruan tinggi swasta
ABC tersebut? tidak besar.
(A) Kegiatan bakti sosial diwajibkan untuk (D) Peningkatan kualitas perguruan tinggi
sekolah pada tahun sebelumnya. swasta pada sarana dan prasarana
(B) Siswa di SMA ABC lebih memilih tidak dilakukan jika biaya yang
kegiatan mandiri. dibayarkan mahasiswa perguruan
(C) Banyak siswa ABC yang tidak tahu ada tinggi swasta besar.
bakti sosial di sekolah. (E) Peningkatan kualitas perguruan tinggi
(D) Siswa SMA ABC lebih suka datang ke swasta pada sarana dan prasarana
acara pentas seni. dilakukan jika biaya yang dibayarkan
(E) Kegiatan bakti sosial dilakukan dalam mahasiswa perguruan tinggi swasta
bentuk sumbangan uang. besar.

Teks berikut digunakan untuk menjawab soal 13. Manakah informasi berikut yang dapat
nomor 12 sampai dengan 15. memperkuat pernyataan “Kualitas yang
semakin meningkat menjadi faktor
Di Indonesia jumlah perguruan tinggi pendorong meningkatnya jumlah peminat
swasta mendominasi sebanyak 90% dari ke perguruan tinggi swasta?”
jumlah seluruh perguruan tinggi. Banyaknya (A) Jumlah pendaftar ke perguruan tinggi
peminat perguruan tinggi swasta adalah satu swasta bermutu baik lebih banyak
faktor pendorong munculnya banyak dibandingkan perguruan tinggi swasta
perguruan tinggi swasta baru. Kualitas yang lainnya.
semakin meningkat juga menjadi faktor (B) Jumlah mahasiswa di perguruan tinggi
pendorong meningkatnya jumlah peminat ke swasta yang bermutu baik lebih sedikit
perguruan tinggi swasta. Kualitas perguruan dibandingkan perguruan tinggi swasta
tinggi swasta biasanya dinilai dari kualitas lainnya.
pembelajaran, dosen, serta sarana dan (C) Semakin tinggi kualitas perguruan
prasarana. Saran dan prasarana biasanya tinggi swasta, makin ketat persyaratan
menjadi fokus perguruan tinggi swasta. seleksi masuknya.
Peningkatan kualitas perguruan tinggi swasta (D) Semakin tinggi kualitas perguruan
pada sarana dan prasarana dilakukan jika dana tinggi swasta, makin mahal biaya
yang dikelola perguruan tinggi swasta banyak. masuknya.
Dana yang banyak tersebut disebabkan oleh (E) Jumlah mendaftar ke perguruan tinggi
besarnya biaya yang dibayarkan oleh swasta hampir setara dengan ke
mahasiswa perguruan tinggi swasta. perguruan tinggi negeri.

12. Berdasarkan bacaan tersebut, manakah


simpulan berikut yang benar?
(A) Biaya yang dibayarkan mahasiswa
perguruan tinggi swasta tidak besar
jika peningkatan kualitas perguruan
tinggi swasta pada sarana dan prasana
dilakukan.
14. Berdasarkan bacaan tersebut, apabila Berdasarkan informasi tersebut, manakah
perguruan tinggi swasta memiliki dosen pernyataan berikut yang benar?
dan pembelajaran yang berkualitas, (A) Relaksasi yang meningkat akan
manakah simpulan berikut yang paling menekan ujung-ujung saraf.
mungkin benar? (B) Pijat refleksi disebabkan oleh kegiatan
(A) Perguruan tinggi swasta lebih diminati yang sangat padat.
daripada perguruan tinggi negeri. (C) Tidur lebih nyenyak akan menekan
(B) Perguruan tinggi swasta memiliki ujung-ujung saraf.
mahasiswa yang berkualitas. (D) Pijat refleksi akan membuat tidur lebih
(C) Perguruan tinggi swasta memiliki nyenyak.
sarana teknologi mutakhir. (E) Aktivitas fisik akan menekan ujung-
(D) Perguruan tinggi swasta tersebut ujung saraf.
merupakan perguruan tinggi baru.
(E) Perguruan tinggi swasta memiliki 17. Gas air mata dapat menyebabkan gejala
banyak peminat. sakit mata, masalah pernapasan, iritasi
kulit, pendarahan, bahkan kebutaan.
15. Manakah informasi berikut yang dapat Amnesti International memasukkan gas air
memperlemah pernyataan “Banyaknya mata sebagai bagian dari barang
peminat perguruan tinggi swasta adalah perdagangan internasional yang
salah satu faktor pendorong munculnya membahayakan. Meskipun demikian, gas
banyak perguruan tinggi swasta baru”? air mata hampir selalu digunakan oleh
(A) Jumlah perguruan tinggi swasta baru pihak berwenang untuk meredam
mengalami peningkatan meskipun demonstrasi.
jumlah pendaftar ke perguruan tinggi
swasta tetap sama. Berdasarkan informasi tersebut, manakah
(B) Biaya pendidikan di perguruan tinggi pernyataan berikut yang pasti benar?
swasta baru tidak setinggi di (A) Pihak berwenang yang menggunakan
perguruan tinggi swasta yang telah gas air mata melanggar Amenesti
berdiri semenjak lama. Internasional.
(C) Pendaftar ke perguruan tinggi semakin (B) Orang yang mengalami iritasi kulit
meningkat seiring dengan akibat gas air mata cenderung
penambahan perguruan tinggi swasta. ditemukan dalam demonstrasi yang
(D) Perizinan mendirikan perguruan rusuh.
tinggi swasta semakin dipermudah (C) Gangguan pernapasan karena gas air
dari tahun ke tahun. mata sangat berbahaya.
(E) Perguruan tinggi swasta memiliki (D) Gas air mata menjadi senjata paling
kualitas yang sama dengan perguruan efektif untuk meredakan demonstrasi.
tinggi negeri. (E) Banyak orang mengalami sakit mata
setelah mengikuti demonstrasi.
16. Pijat refleksi yang dilakukan di bagian
tangan dan kaki akan menekan ujung-
ujung saraf. Tekanan di ujung saraf akan
meningkatkan relaksasi. Tubuh yang rileks
akan membuat tidur lebih nyenyak.
Meskipun kegiatan sangat padat, tubuh
tidak akan kelelahan karena tidur yang
nyenyak.
18. Wilayah hutan yang dijadikan 20. Saat ini pendapatan wilayah Y dari sektor
pertambangan lebih sering mengalami pajak menurun. Menurut kepala daerah
erosi. Makin tinggi laju erosi, makin sering wilayah Y, hal ini disebabkan oleh
banjir terjadi. rendahnya kesadaran masyarakat dalam
membayar pajak.
Berdasarkan pernyataan tersebut, Manakah pernyataan berikut yang
manakah yang paling mungkin menjadi memperlemah pendapat kepala daerah
akibat dari hutan yang dijadikan tersebut?
pertambangan? (A) Tidak ada program sosialisasi
(A) Kegiatan pertambangan tidak dapat membayar pajak di wilayah Y.
dilakukan pada saat terjadi banjir. (B) Banyak pelanggaran pajak di wilayah
(B) Kelebatan hutan di sekitar wilayah Y.
pertambangan berkurang. (C) Hampir semua anggota masyarakat
(C) Kawasan hutan yang berfungsi sebagai wilayah Y membayar pajak sesuai
penandah hujan berkurang. ketentuan.
(D) Kawasan hutan sekitar pertambangan (D) Pelanggaran pajak di wilayah Y
sulit untuk ditumbuhi pohon. ditindak dengan tegas.
(E) Banyak daerah di sekitar hutan yang (E) Pendapatan terbesar wilayah Y berasal
dijadikan pertambangan mengalami dari sektor nonpajak.
banjir.
21. Anggrek bulan adalah salah satu bunga
19. Semua penduduk Desa X melakukan Nasional Indonesia yang pertama kali
kegiatan berkebun. Untuk meningkatkan ditemukan oleh Dr. C.L. Blume, seorang
hasil kebun, penduduk membuat pupuk ahli botani. Tanaman ini hidup dengan
kandang. Namun, tidak semua penduduk cara menempel pada batang atau cabang
memiliki ternak sehingga di antara mereka pohon lain dan tumbuh subur di daerah
ada yang menggunakan kompos. Hal ini dengan ketinggian hingga 600 meter di atas
menunjukkan bahwa penduduk Desa X permukaan laut. Anggrek bulan termasuk
dapat mengusahakan cara agar kebunnya dalam tanaman yang menyukai sedikit
menghasilkan panen yang melimpah. cahaya matahari sebagai penunjang
hidupnya. Selain itu, bunga nya dapat
Simpulan berdasarkan informasi dalam mencapai ukuran paling besar, yaitu
teks tersebut adalah semua penduduk berdiameter 10 cm.
Desa X yang tidak memiliki ternak
melakukan kegiatan berkebun. Berdasarkan informasi tersebut, manakah
Manakah pernyataan berikut yang pernyataan berikut yang pasti salah?
menggambarkan kualitas simpulan (A) Banyak anggrek bulan yang hidup
tersebut? subur di hutan-hutan dengan
(A) Simpulan pasti benar ketinggian 300 m di atas permukaan
(B) Simpulan mungkin benar laut.
(C) Simpulan pasti salah (B) Anggrek bulan hidup yang di bawah
(D) Simpulan mungkin salah cahaya dapat tumbuh dengan ukuran
(E) Simpulan tidak dapat dinilai karena bunga berdiameter lebih dari 10 cm.
informasi tidak cukup (C) Anggrek bulan tidak dapat tumbuh
subur di daerah yang memiliki banyak
cahaya.
(D) Terdapat jenis pohon anggrek yang
diameter bunganya berdiameter lebih
dari 10 cm di luar Indonesia.
(E) Anggrek bulan tidak dapat tumbuh di
dataran rendah
22. Tahun ini, jumlah pencurian di RT 08 (B) Semua guru dan dosen menggunakan
mengalami penurunan dibandingkan bukan buku dan bukan internet.
tahun sebelumnya. Ketua RT 08 (C) Sebagian guru dan dosen
menyampaikan bahwa penyebab utama menggunakan buku dan internet.
hal tersebut adalah adanya kegiatan ronda (D) Sebagian guru dan dosen
yang wajib diikuti oleh semua kepala menggunakan buku tentang internet.
keluarga. (E) Semua guru dan dosen menggunakan
buku dan internet.
Manakah pernyataan berikut yang akan
memperkuat argumen ketua RT 08 25. Semua kerabat dekat menghadiri pesta
tersebut? keluarga.
(A) Jumlah warga yang mengikuti Asep menghadiri pesta keluarga,
kegiatan ronda selam setahun ini tidak sedangkan Dadang tidak hadir.
mengalami perubahan. (A) Asep bukan kerabat dekat keluarga.
(B) Pencurian di RT 08 tetap terjadi saat (B) Dadang bukan kerabat dekat keluarga.
dini hari. (C) Dadang dan Asep kerabat dekat
(C) Selain kegiatan ronda, RT 08 juga keluarga.
menggalakkan kegiatan lapor diri 1x24 (D) Dadang dan Asep bukan kerabat dekat
jam bagi tamu. keluarga.
(D) Jumlah pencurian yang terjadi di RT 08 (E) Dadang dan Asep dua orang kerabat
tidak sebanyak yang terjadi di RT-RT dekat.
lainnya.
(E) Pada tahun sebelumnya, kegiatan 26. Semua calon mahasiswa menempuh tes
ronda belum dilakukan di lingkungan psikologi.
RT 08. Sebagian calon mahasiswa memiliki skor
TOEFL di atas 450.
23. Semua mesin memiliki katup. (A) Sebagian calon mahasiswa yang
Sebagian mesin berukuran kecil. memiliki skor TOEFL di atas 450
(A) Semua mesin berukuran kecil dan menempuh tes psikologi.
memiliki katup. (B) Semua calon mahasiswa yang
(B) Semua mesin berukuran tidak kecil memiliki skor TOEFL di atas 450 tidak
dan tidak memiliki katup. menempuh tes psikologi.
(C) Semua mesin berukuran kecil dan (C) Semua calon mahasiswa yang
tidak memiliki katup. memiliki skor TOEFL di atas 450 tidak
(D) Sebagian mesin berukuran tidak kecil menempuh tes psikologi.
dan memiliki katup. (D) Sebagian calon mahasiswa yang
(E) Sebagian mesin berukuran kecil dan memiliki skor TOEFL di atas 450 tidak
tidak memiliki katup. menempuh tes psikologi.
(E) Semua calon mahasiswa yang
24. Semua guru dan dosen menggunakan menempuh tes psikologi tidak
buku sebagai sumber bahan untuk memiliki skor TOEFL di atas 450.
mengajar.

Sebagian guru dan dosen menggunakan


internet sebagai sumber bahan untuk
mengajar.
(A) Semua guru dan dosen menggunakan
buku tentang internet.
27. Tidak ada bunga mawar kecuali berwarna 30. Semua obat mengandung vitamin.
putih dan merah. Sebagian obat rasanya pahit.
Samuel menerima bunga bukan putih, (A) Sebagian obat rasanya pahit dan tidak
bukan merah. mengandung vitamin.
(A) Samuel menerima bunga mawar putih. (B) Semua obat rasanya pahit dan
(B) Samuel menerima bunga mawar mengandung vitamin.
merah. (C) Sebagian obat rasanya pahit, tetapi
(C) Samuel menerima bunga bukan mengandung vitamin.
mawar. (D) Sebagian obat rasanya tidak pahit dan
(D) Samuel menerima bunga bukan tidak mengandung vitamin.
mawar putih. (E) Semua obat rasanya tidak pahit, tetapi
(E) Samuel menerima bunga mawar mengandung vitamin.
bukan merah dan bukan putih.
31. Semua asrama dilengkapi kantin dan ada
28. Semua pelaut adalah perenang. jam kunjungan.
Sebagian perenang bukan penyelam. Ani berada di tempat yang tidak
(A) Semua penyelam adalah pelaut. dilengkapi kantin dan tidak ada jam
(B) Semua penyelam bukan pelaut. kunjungan.
(C) Sebagian penyelam bukan perenang. (A) Ani berada di asrama yang tidak
(D) Semua pelaut adalah penyelam. dilengkapi kantin.
(E) Sebagian pelaut bukan penyelam. (B) Ani berada di asrama yang tidak ada
jam kunjungan.
29. Semua siswa kelas F dapat berbahasa (C) Ani berada di bukan asrama
Inggris. (D) Ani berada di asrama yang tidak
Semua siswa kelas F yang menduduki dilengkapi kantin.
rangking 10 besar juga dapat berbahasa (E) Ani berada di asrama yang tidak
Belanda. dilengkapi kantin dan tidak ada jam
(A) Yang dapat berbahasa Belanda, kunjungan.
menduduki rangking 10 besar dan
tidak dapat berbahasa Inggris. 32. Semua siswa adalah anggota pramuka.
(B) Yang dapat berbahasa Belanda, tidak Semua penghuni asrama adalah siswa.
menduduki rangking 10 besar dan (A) Semua siswa adalah penghuni asrama.
dapat berbahasa Inggris. (B) Semua anggota pramuka adalah
(C) Yang tidak dapat berbahasa Belanda, penghuni asrama.
tidak menduduki rangking 10 besar (C) Semua penghuni asrama adalah
dan dapat berbahasa Inggris. anggota pramuka.
(D) Yang tidak dapat berbahasa Belanda, (D) Sebagian penghuni asrama adalah
menduduki rangking 10 besar dan anggota pramuka.
dapat berbahasa Inggris. (E) Sebagian penghuni asrama adalah
(E) Yang tidak dapat berbahasa Belanda, bukan anggota pramuka.
tidak menduduki rangking 10 besar
dan tidak dapat berbahasa Inggris.
33. Seluruh perwira baru wajib mengikuti 37. Semua pekerja memakai masker.
latihan tempur. Sebagian pekerja tidak memakai sarung
Sebagian perwira baru mengunjungi tangan.
asrama. (A) Ada pekerja yang tidak memakai
(A) Seluruh perwira baru yang mengikuti masker, tetapi memakai sarung
latihan tempur tidak mengunjungi tangan.
asrama. (B) Ada pekerja yang tidak memakai
(B) Seluruh perwira baru yang mengikuti masker dan tidak memakai sarung
latihan tempur mengunjungi asrama. tangan.
(C) Sebagian perwira baru yang (C) Ada pekerja yang memakai sarung
mengunjungi asrama tidak mengikuti tangan, meskipun tidak memakar
latihan tempur. masker.
(D) Seluruh perwira baru yang (D) Ada pekerja yang tidak memakar
mengunjungi asrama mengikuti masker, tetapi memakai sarung
latihan tempur. tangan.
(E) Sebagian perwira baru yang tidak (E) Ada pekerja yang tidak memakai
mengunjungi asrama tidak mengikuti sarung tangan, tetapi memakai
latihan tempur. masker.

34. Semua tentara adalah militer. 38. Semua pantai berpasir dan panas.
Sebagian militer bukan pelatih. Andi berada di tempat yang tidak berpasir
(A) Semua tentara bukan pelatih. dan tidak panas.
(B) Sebagian tentara bukan pelatih. (A) Andi berada di pantai berpasir.
(C) Sebagian tentara bukan militer. (B) Andi berada di pantai panas.
(D) Semua tentara adalah pelatih. (C) Andi berada di bukan pantai.
(E) Semua militer adalah pelatih. (D) Andi berada di pantai bukan berpasir.
(E) Andi berada di pantai bukan berpasir
35. Tentara pemberani dapat bertempur. dan bukan panas.
Sebagian tentara dari kota B tidak dapat
bertempur. 39. Semua pohon berdaun dan berbunga.
(A) Semua tentara di kota B tidak Tanaman A berbunga, tetapi tidak
pemberani. berdaun.
(B) Tidak ada tentara di kota B yang (A) Tanaman A adalah pohon yang
pemberani. berbunga.
(C) Sebagian tentara di kota B tidak (B) Tanaman A adalah pohon yang tidak
pemberani. berdaun.
(D) Semua tentara di kota B pemberani. (C) Tanaman A adalah pohon yang
(E) Semua tentara yang tidak pemberani berbunga dan tidak berdaun.
berasal dari kota B. (D) Tanaman A adalah bukan pohon
berdaun.
36. Semua pesawat adalah kendaraan. (E) Tanaman A adalah bukan pohon
Sebagian kendaraan tidak komersial. meskipun berbunga.

(A) Sebagian pesawat tidak komersial.


(B) Semua yang komersial bukan pesawat.
(C) Sebagian yang komersial adalah
kendaraan.
(D) Semua kendaraan adalah komersial.
(E) Semua kendaraan tidak komersial.
40. Semua polisi pandai menembak dan (B) Semua mahasiswa tidak diizinkan
tangkas. memotret di dalam museum.
Tuan X tidak tangkas, tetapi pandai (C) Sebagian mahasiswa diizinkan
memotret ketika mengunjungi
menembak.
museum.
(A) Tuan X seorang polisi yang pandai (D) Hanya mahasiswa yang berlibur ke
menembak. museum yang tidak dilarang
(B) Tuan X seorang polisi yang tidak memotret.
tangkas. (E) Tidak ada pengunjung museum yang
(C) Tuan X seorang polisi yang pandai berstatus mahasiswa.
menembak dan tidak tangkas.
44. Semua siswa kelas 3 sedang berada di kelas
(D) Tuan X bukan seorang polisi,
atau di laboratorium sekolah.
meskipun pandai menembak. Ruang laboratorium sekolah sedang
(E) Tuan X bukan seorang polisi yang kosong.
tangkas. (A) Semua siswa kelas 3 tidak berada di
kelas.
41. Semua warga negara yang berumur di atas (B) Tidak ada siswa kelas 3 yang sedang
17 tahun memiliki hak pilih. berada di laboratorium atau di kelas.
(C) Semua siswa kelas 3 sedang berada di
Semua mahasiswa S-1 berumur di atas 17
laboratorium dan di kelas.
tahun. (D) Tidak ada siswa kelas 3 yang berada di
(A) Semua mahasiswa S-1 memiliki hak kelas.
pilih. (E) Semua siswa kelas 3 sedang berada di
(B) Semua warga negara memiliki hak kelas.
pilih.
45. Semua tebing berwarna hitam.
(C) Tidak semua mahasiswa S-1 memiliki
Sebagian jurang adalah tebing.
hak pilih. (A) Semua jurang adalah tebing.
(D) Tidak ada mahasiswa S-1 yang (B) Semua jurang berwarna hitam.
memiliki hak pilih. (C) Semua yang hitam adalah tebing.
(E) Tidak ada warga negara yang memiliki (D) Sebagian jurang berwarna hitam.
hak pilih menjadi mahasiswa S-1. (E) Semua yang hitam adalah jurang.

46. Semua tanaman adalah kayu.


42. Semua binatang karnivora bertaring.
Beberapa jenis kayu adalah keras.
Sebagian binatang yang bertaring (A) Semua kayu adalah tanaman.
menyusui. (B) Semua tanaman adalah keras.
(A) Sebagian binatang yang bertaring (C) Sebagian tanaman bukan kayu.
bukan karnivora. (D) Semua kayu yang keras adalah
(B) Semua binatang karnivora menyusui. tanaman.
(E) Sebagian kayu yang keras bukan
(C) Semua binatang yang tidak menyusui
tanaman.
tidak bertaring.
(D) Semua binatang menyusui pasti 47. Semua yang wangi adalah bunga.
bertaring. Sebagian yang wangi berwarna hijau.
(E) Sebagian binatang karnivora tidak (A) Semua bunga adalah wangi.
menyusui. (B) Semua bunga berwarna hijau.
(C) Sebagian bunga berwarna hijau.
43. Tidak seorang pun pengunjung museum (D) Semua yang wangi berwarna hijau.
diizinkan memotret.
(E) Sebagian yang wangi adalah bunga.
Sebagian mahasiswa mengisi liburan
dengan mengunjungi museum.
(A) Sebagian mahasiswa tidak diizinkan
memotret di dalam museum.
48. Semua yang lahir di negara C adalah 50. Semua musik memiliki tempo cepat.
penduduk. Beberapa yang memiliki tempo cepat
Sebagian penduduk C tinggal beberapa adalah pop.
bulan di negara D. (A) Semua musik merupakan pop.
(A) Semua yang tinggal beberapa bulan di (B) Semua musik pop bertempo cepat.
negara D lahir di negara C. (C) Sebagian pop bukan merupakan
(B) Sebagian yang tinggal beberapa bulan musik.
di negara D lahir di negara C. (D) Semua yang bertempo cepat adalah
(C) Semua yang tidak tinggal beberapa pop.
bulan di negara D lahir di negara C. (E) Sebagian musik memiliki tempo yang
(D) Semua yang tinggal beberapa bulan di cepat.
negara D tidak lahir di negara C.
(E) Sebagian yang hanya tinggal beberapa
bulan di negara D tidak lahir di negara
C.

49. Semua sarung bermotif kotak-kotak.


Sebagian kebaya adalah sarung.
(A) Semua sarung adalah kebaya.
(B) Semua kebaya bermotif kotak-kotak.
(C) Sebagian kebaya bermotif kotak-kotak.
(D) Semua yang bermotif kotak-kotak
adalah kebaya.
(E) Sebagian yang bermotif kotak-kotak
adalah sarung.

Anda mungkin juga menyukai